Sei sulla pagina 1di 118

Revise your past 1.

5 years of current affairs with our most comprehensive Test Series on Current Affairs

DO NOT OPEN THIS TEST BOOK UNTIL YOU ARE TOLD TO DO SO


TEST BOOKLET
GENERAL STUDIES
PAPER – I

CIVILSTAP CURRENT AFFAIRS TEST SERIES

Time Allowed: Two Hours Maximum Marks: 200

INSTRUCTIONS

1. IMMEDIATELY AFTER THE COMMENCEMENT OF THE EXAMINATION, YOU SHOULD CHECK


THAT THIS TEST BOOKLET DOES NOT HAVE ANY UNPRINTED OR TORN OR MISSING PAGES OR
ITEMS, ETC. IF SO, GET IT REPLACED BY A COMPLETE TEST BOOKLET.
2. Please note that it is the candidate's responsibility to encode and fill in the Roll Number and
Test Booklet Series Code A, B, C or D carefully and without any omission or discrepancy at
the appropriate places in the OMR Answer Sheet. Any omission will render the Answer Sheet
liable for rejection.
3. You have to enter your Roll Number on the Test
Booklet in the Box provide alongside.
DO NOT write anything else on the Test Booklet.
4. This Test Booklet contains 100 items (questions). Each item is printed in English only. Each item
comprises four responses (answers). You will select the response which you want to mark on the
Answer Sheet. IN case you feel that there is more than one correct response which you want to mark
on the Answer Sheet. In case you feel that there is more than one correct response, mark the
response which you consider the best. In any case, choose ONLY ONE response for each item.
5. You have to mark all your responses ONLY on the separate Answer Sheet provided. See directions
in the Answer Sheet.
6. All the items carry equal marks.
7. Before you proceed to mark in the Answer Sheet response to various items in the Test Booklet, you
have to fill in some particulars in the Answer Sheet as per instructions sent to you with your
Admission Certificate.
8. After you have completed filing in all your responses on the Answer Sheet and the examination has
concluded, you should hand over to the invigilator only the Answer Sheet. You are permitted to
take away with you the Test Booklet.
9. Sheets for rough work are appended in the Test Booklet at the end.
10. Penalty for wrong answers:
THERE WILL BE PENALTY FOR WRONG ANSWERS MARKED BY A CONDIDATE IN THE
OBJECTIVE TYPE QUESTION PAPERS.
i. There are four alternatives for the answer to every question. For each question for which a wrong answer
has been given by the candidate, one third of the marks assigned to that question will be deducted as
penalty.
ii. If a candidate gives more than one answer, it will be treated as a wrong answer even if one of the given
answers happens to be correct and there will be same penalty as above to that question.
iii. If a question is left blank, i.e., no answer is given by the candidate, there will be no penalty for that
question.

DO NOT OPEN THIS TEST BOOK UNTIL YOU ARE TOLD TO DO SO

Have a querry?
Drop us a mail at – hello@civilstap.com or call us at 8146207241
1
Revise your past 1.5 years of current affairs with our most comprehensive Test Series on Current Affairs
TEST - I

1. With reference to the National


5. Which of the following statements
Knowledge Network (NKN),
is/are correct regarding ‘The
consider the following statements:
Indian Council for Cultural
1. It is a multi-gigabit pan-India Relations (ICCR)’?
resource sharing network which
aims to connect all universities, 1. It is an organization coming
research institutions, libraries under the Ministry of Tourism
across the country 2. It was founded in 1950 by
2. India has started to extend its Independent India’s first
KNK to research and educational Education Minister
networks in foreign countries Select the correct answer using the
Which of the statements given above code given below:
is/are correct? (a) 1 only
(a) 1 only (b) 2 only
(b) 2 only (c) Both 1 and 2
(c) Both 1 and 2 (d) Neither 1 nor 2
(d) Neither 1 nor 2 6. With reference to the Electoral
2. What is ‘DS Faraday’, a term Bond Scheme, consider the
recently seen in the news? following statements:
(a) Wheat variety 1. Any Indian resident or
(b) Electromagnetic wave incorporated body (such as a
(c) Object in the outer space company) is eligible to purchase
(d) New element in the periodic table an electoral bond
2. The bonds will have to
3. With reference to ‘Pradhan Mantri mandatorily carry the name of the
Swasthya Suraksha Yojana purchaser
(PMSSY), which of the following 3. The bonds can be encashed by
statement is/are correct? parties only through a designated
account with an authorized bank
1. It is a health insurance scheme
Which of the statement(s) given
launched by the Ministry of
above is/are incorrect?
Health and Family Welfare
(a) 3 only
2. One of the components of PMSSY
(b) 1 and 2 only
is the setting up of AIIMS like
(c) 2 and 3 only
institutions in India
(d) 1 only
Select the correct answer using the
code given below. 7. Which of the following statements
(a) 1 only is/are correct regarding the New
(b) 2 only Delhi International Arbitration
(c) Both 1 and 2 Centre (NDIAC)?
(d) Neither 1 nor 2 1. It is a statutory body established
4. What is ‘GAVI’, a term sometimes to conduct arbitration, mediation,
seen in the news? and conciliation proceedings
2. It subsumes the International
(a) a global vaccine alliance Centre for Alternative Dispute
(b) a web-portal connecting all the Resolution
schools in India 3. It will be headed by the Union
(c) an indigenous satellite launch Minister of External Affairs
vehicle Which of the statements given above
(d) a government portal for defence is/are correct?
procurement (a) 2 and 3 only
Have a querry?
Drop us a mail at – hello@civilstap.com or call us at 8146207241
2
Revise your past 1.5 years of current affairs with our most comprehensive Test Series on Current Affairs

(b) 1 only 11. With reference to India’s FDI


(c) 1 and 2 only Policy in Single Brand Retail
(d) 1, 2 and 3 Trading, consider the following
statements:
8. Which of the following statements
is/are correct regarding the e- 1. 100% FDI in Single Brand Retail
Samvad portal? Trading is allowed under the
automatic route
1. It is an online platform for NGOs 2. A Single Brand Retail Chain is
and Civil Societies to interact with expected to sell all its products
the Ministry of Health and Family under only one label across its
Welfare stores in India
2. It would help the government in 3. In cases, where the FDI exceeds
the formulation of effective 51%, 30% of the value of goods
policies and measures based on purchased must be from India
the feedbacks/suggestions Which of the above statements
provided by the NGOs is/are correct?
Select the correct answer using the (a) Only 1
code given below: (b) 1 and 3
(a) 1 only (c) 1 and 2
(b) 2 only (d) 1, 2, 3
(c) Both 1 and 2
(d) Neither 1 nor 2 12. The term ‘The National
Register of Citizens’ seen in the
9. What is 'Parker Solar Probe',
news is related to which of the
recently in the news?
following states in India?
(a) Mankind’s first mission to touch
(a) Jammu and Kashmir
the Sun launched by NASA
(b) Manipur
(b) Mankind’s first mission to enter
(c) Assam
the outer space launched by
(d) Telangana
NASA
(c) Mission to test the presence of 13. Recently, our scientists have
water on Mars launched by NASA discovered a new frog species
(d) Mankind’s first mission to touch which has been named after
the Sun launched by ISRO wildlife scientist Mewa Singh. This
10. The term ‘Retrotransposons’ species belongs to a genus
seen in the news refers to? endemic to the Western Ghats.
This species has been discovered
(a) Small genetic sequences that can
in which of the following wildlife
lead to mutation in their host’s
sanctuary?
genome
(b) Small genetic sequences that act (a) Malabar Wildlife Sanctuary
as scissors to cut the sections of (b) Tansa Sanctuary
their host’s genome (c) Cotigao Sanctuary
(c) Small genetic sequences that (d) Dandeli Sanctuary
have the ability to replicate and
position themselves in new 14. The term, ‘Channel Stuffing’ in
locations in their host’s genome business refers to which of the
(d) Small genetic sequences that are following?
responsible for divisions of cells (a) A business practice wherein a
in their host’s genome company resorts to unfair means

Have a querry?
Drop us a mail at – hello@civilstap.com or call us at 8146207241
3
Revise your past 1.5 years of current affairs with our most comprehensive Test Series on Current Affairs

to sell its goods/services to the consider the following


end consumers statements:
(b) A business practice wherein a 1. It was launched in the year 2017
company resorts to international by the Ministry of Finance
channels to market its 2. LIC has been given the sole
goods/services privilege to operate the scheme
(c) A business practice wherein a 3. It provides social security during
company takes more credit than old age and to protect elderly
it can repay for the production of persons aged 55 years and above
goods/services in a year Which of the statement(s) given
(d) A business practice wherein a above is/are correct?
company produces and delivers (a) 1 only
more goods to retailers than what (b) 1 and 2 only
would be sold successfully to end (c) 1 and 3 only
consumers (d) 1, 2 and 3
15. The terms, ‘Ethereum’, 18. With reference to the ‘National
‘Ripple’, ‘Monero’, ‘Dash’ often Tobacco Control Programme’,
heard in news are types of which consider the following statement:
of the following?
1. It was launched in the year 2007-
(a) Supercomputers 08 by the Ministry of Health and
(b) Cryptocurrencies Family Welfare
(c) Software Programming 2. One of the objectives of the
Languages programme is to ensure effective
(d) Digital Payment Systems implementation of the provisions
16. With reference to the ‘National under COPTA, 2003
Clean Air Programme’, consider 3. The National Tobacco Control Cell
the following statements: (NTCC) is responsible for overall
policy formulation, planning,
1. The goal of the NCAP is to meet implementation, monitoring and
the prescribed annual average evaluation of the different
ambient air quality standards at activities envisaged under the
all locations in the country in a programme
stipulated timeframe (long-term) Which of the statement(s) given
2. This will be a mid-term, five-year above is/ are correct?
action plan to begin with keeping (a) 1 and 2 only
2019 as the first year (b) 2 and 3 only
3. The target of NCAP is 20-30% (c) 2 only
reduction of PM2.5 and PM10 (d) 1, 2 and 3
concentration by 2024 keeping
2019 as the base year for 19. ‘Mission Raftar’ seen in the
comparison of concentration news relates to which of the
Which of the statement(s) given following?
above is/are correct? (a) Doubling the rate at which the
(a) 3 only Indian economy is growing
(b) 1 and 3 (b) Increasing the proportion of
(c) 1 and 2 passengers travelling by air route
(d) 1, 2 and 3 (c) Speeding up India’s progress
17. With reference to Pradhan towards achieving the SDGs
Mantri Vaya Vandana Yojana, (d) Doubling of average speed of
freight trains

Have a querry?
Drop us a mail at – hello@civilstap.com or call us at 8146207241
4
Revise your past 1.5 years of current affairs with our most comprehensive Test Series on Current Affairs

20. Consider the following (a) 1 only


statements in respect of the (b) 2 only
‘Khelo India Programme’: (c) Both 1 and 2
1. It has been introduced to revive (d) Neither 1 nor 2
sports culture in India at the 23. ‘Walong tri-junction’ was in
grass-root level and to gorge tie- the news recently. It is a junction
ups with various international between which of the following
sports organizations
countries?
2. The revamped Khelo India
Programme aims to achieve the (a) India, China, Pakistan
twin objectives of mass (b) India, Bhutan, China
participation and Promotion of (c) India, China, Myanmar
excellence in sports (d) China, Russia, Mongolia
3. The Khelo India logo has been 24. Consider the following
designed by Ogilvy India
statements:
Which of the above statement(s)
is/are correct? 1. Transiting Exoplanet Survey
(a) 3 only Satellite (TESS) is a satellite
(b) 1 and 2 only launched by the European Space
(c) 2 and 3 only Agency to catalog exoplanets.
(d) 1, 2 and 3 2. Exoplanets are planets that
remain undiscovered in our solar
21. Consider the following system.
statements about the Indian Which of the above statement(s)
Regional Navigation Satellite is/are correct?
System (IRNSS): (a) Only 1
(b) Only 2
1. It has three satellites in
(c) Both 1 and 2
geosynchronous orbit and four
(d) Neither 1 nor 2
satellites in geostationary orbit.
2. It can service regions extending 25. Consider the following
up to 1500 km around India's statements about the Brahmos
boundary. Missile:
3. It has a total of nine satellites of
1. It has been indigenously
which six are currently in orbit.
developed by India.
Which of the above statement(s)
2. It is the fastest cruise missile in
is/are correct?
the world.
(a) 1 and 2 only
3. It travels at a speed faster than
(b) 1 and 3 only
the speed of sound.
(c) 2 only
Which of the above is/are correct?
(d) 1, 2 and 3
(a) 1 and 2 only
22. Consider the following (b) 2 and 3 only
statements: (c) 1 and 3 only
(d) All of the above
1. Neutrinos are tiny elementary
particles that are part of atoms. 26. Consider the following
2. The India-based Neutrino statements about Pink Moon:
Observatory is proposed to be set
1. The full moon in April is called
up in Theni district of Tamil
Pink Moon.
Nadu.
2. It is called so because it is Pink in
Which of the above statement(s)
colour.
is/are correct?

Have a querry?
Drop us a mail at – hello@civilstap.com or call us at 8146207241
5
Revise your past 1.5 years of current affairs with our most comprehensive Test Series on Current Affairs

Which of the above is/are correct? 30. With reference to the "Student
(a) 1 only Rural Entrepreneurship
(b) 2 only Awareness Development Yojana
(c) Both 1 and 2 (READY), consider the following
(d) Neither 1 nor 2 statements:
27. Consider the following 1. It is an initiative of Indian
statements about North East Council of Agricultural Research
Centre for Technology Application 2. It has been started for the rural
and Reach (NECTAR): youth employed in agriculture
sector
1. It is an autonomous society under Which of the statements given above
the Ministry of Development of is/are correct?
North Eastern Region. (a) 1 only
2. It has a vision to expand the (b) 2 only
benefits of technology for (c) Both 1 and 2
inclusive economic development (d) Neither 1 nor 2
of the North East.
Which of the above is/are correct? 31. With reference to the "Central
(a) 1 only Adoption Resource Authority
(b) 2 only (CARA), consider the following
(c) Both 1 and 2 statements:
(d) Neither 1 nor 2 1. It is designated as the Central
28. Consider the following Authority to deal with inter-
statements about the Fugitive country adoptions
Economic Offenders Act, 2018: 2. It is a statutory body
3. It has allowed individuals in a
1. It covers economic offences of at live-in relationship to adopt
least Rs. 50 crores. children within India
2. Counterfeiting currency is one of Which of the statement(s) given
the offences listed in the Act. above is/are correct?
Which of the above is/are correct? (a) 1 only
(a) 1 only (b) 2 and 3 only
(b) 2 only (c) 1 and 3 only
(c) Both 1 and 2 (d) 1, 2 and 3
(d) Neither 1 nor 2
32. With reference to the "India-
29. Consider the following
based Neutrino Observatory",
statements about Pectinophora
consider the following
gossypiella:
statements:
1. It is commonly known as Pink
1. The Tata Institute of
bollworm.
Fundamental Research is the
2. It is a pest that attacks cotton
nodal institution for this project
crops.
2. 1500 crore has been allocated for
Which of the above statement(s)
this project
is/are correct?
Which of the statements given above
(a) 1 only
is/are correct?
(b) 2 only
(a) 1 only
(c) Both 1 and 2
(b) 2 only
(d) Neither 1 nor 2
(c) Both 1 and 2
(d) Neither 1 nor 2

Have a querry?
Drop us a mail at – hello@civilstap.com or call us at 8146207241
6
Revise your past 1.5 years of current affairs with our most comprehensive Test Series on Current Affairs

33. "Regional Aviation Which of the statement/s given


Partnership" sometimes above is/are correct?
mentioned in the news, is related (a) 1 only
to which of the following (b) 2 only
organization? (c) Both 1 and 2
(a) BRICS (d) Neither 1 nor 2
(b) ASEAN 37. With reference to the "Swachh
(c) G7 Survekshan Grameen, 2018",
(d) BIMSTEC consider the following
34. With reference to the statements:
"Chandrayaan-2", consider the 1. It has been launched in only 100
following statements: districts of the country
1. It is only a lunar orbiter, 2. Citizen's feedback is one of the
developed by India parameters for ranking
2. It is India's second lunar Which of the statement/s given
exploration mission above is/are correct?
3. It will be launched to the moon (a) 1 only
by GSLV Mark III (b) 2 only
Which of the statement(s) given (c) Both 1 and 2
above is/are correct? (d) Neither 1 nor 2
(a) 1 and 2 only 38. Consider the following
(b) 2 and 3 only statements:
(c) 3 only
(d) 1, 2 and 3 1. Golden jackal is listed as
"Endangered" on the IUCN Red
35. The term "Generic Drug" is List
sometimes seen in the news. 2. IUCN is an organ of the United
Which of the following statement Nations
best describes this term? Which of the statement(s) given
above is/are correct?
(a) These are the drugs, which are
(a) 1 only
not approved by the CDSCO
(b) 2 only
(b) These are inferior drug to brand-
(c) Both 1 and 2
name drug as they are being
(d) Neither 1 nor 2
manufactured in poor-quality
facilities 39. With reference to the "Assam
(c) These are the cheaper and more Witch Hunting (Prohibition,
effective alternative to brand- Prevention and Protection) Act,
name drugs consider the following
(d) These are the copies of the statements:
brand-name drugs that have
exactly the same use and effects 1. It prescribes a prison term of up
to seven years and up to ₹5 lakh
36. With reference to the "Indian in fine for calling a person witch
Council of World Affairs (ICWA)", 2. Every offense under the Act has
consider the following been made “cognizable, non-
statements: bailable and non-compoundable
1. It is an institution of national Which of the statement(s) given
importance above is/are correct?
2. Vice-President of India is the ex- (a) 1 only
officio President of ICWA (b) 2 only

Have a querry?
Drop us a mail at – hello@civilstap.com or call us at 8146207241
7
Revise your past 1.5 years of current affairs with our most comprehensive Test Series on Current Affairs

(c) Both 1 and 2 (a) 1 and 2 only


(d) Neither 1 nor 2 (b) 2 and 3 only
(c) 1 only
40. With reference to the (d) 1,2 and 3
Genetically Modified (GM) Crops,
consider the following 43. Consider the following pairs:
statements: Satellites Objective
1. India ranks 2nd in global sometimes
cultivation of GM crops mentioned in
2. BT Cotton is the only GM crop news
approved for commercial 1. GRACE-FO Monitor
cultivation in India changes in sea
Which of the statement(s) given level rise and ice
above is/are correct? melt
(a) 1 only 2. ICESat-2 Measuring ice
(b) 2 only sheet elevation
(c) Both 1 and 2 and sea ice
(d) Neither 1 nor 2 thickness
3. ASNARO-2 Earth-Imaging
41. Which one of the following best Which of the pair(s) given above
describes the term "Net is/are correctly matched?
Neutrality" sometimes seen in (a) 1 and 2 only
news? (b) 2 and 3 only
(a) It is a method to ensure that all (c) 1 and 3 only
telecom companies are charging (d) 1, 2 and 3
uniform prices for internet
access 44. Section 497 of Indian Penal
(b) It is a principle that Internet Code is related to:
Service Providers should not (a) Cyber Crime
discriminate and treat all (b) Child Labour
internet communications equally (c) Domestic Violence
(c) It is an initiative by Indian (d) Adultery
government to make uniform
Data Privacy policies for all 45. "Nano-technology" has
(d) It is a principle that there should applications in which of the
not be discrimination on the following?
placement of websites on search 1. Precise drug delivery
engines 2. Faster recharge of batteries
3. Improved air quality
42. With reference to the "Ease of 4. Cancer treatment
doing business ranking of states, 5. Better packaging of food
2018", consider the following Select the correct answer using the
statements: code given below:
1. It has been released by the (a) 1, 3 and 4 only
Ministry of Corporate Affairs (b) 2, 3 and 5 only
2. It is based on the scores of states (c) 1 and 4 only
in Business Reform Action Plan (d) 1, 2, 3, 4 and 5
3. Andhra Pradesh has topped the
list 46. With reference to the
Which of the statement/s given "Salicornia Plant", mentioned
above is/are correct?

Have a querry?
Drop us a mail at – hello@civilstap.com or call us at 8146207241
8
Revise your past 1.5 years of current affairs with our most comprehensive Test Series on Current Affairs

often in the news, consider the consider the following


following statements: statements:
1. It grows in salty marshes in the 1. It has been launched under the
mangrove wetlands Pradhan Mantri Awas Yojana-
2. It is used in the treatment of Urban in India
hypertension and diabetes 2. It has been launched by the
3. India is the largest producer of International Federation for
this plant Housing and Planning
Which of the statement/s given Which of the statement/s given
above is/are correct? above is/are correct?
(a) 1 and 2 only (a) 1 only
(b) 2 and 3 only (b) 2 only
(c) 1 and 3 only (c) Both 1 and 2
(d) 1, 2 and 3 (d) Neither 1 nor 2
47. In the Indian context, what is 50. With reference to the
the implication of becoming the provisions made under the DNA
"Institutions of Eminence"? Technology (Use and Application)
1. They would be given significant Regulation Bill, 2018, consider
autonomy in operations the following statements:
2. Grant of 1000 crore would be 1. It provides for the establishment
provided to them over 5 years of regional DNA data banks
3. IIT Kharagpur has been declared 2. Written consent by individuals is
as the Institution of Eminence required to collect DNA samples
Which of the statement/s given from them
above is/are correct? Which of the statements/s given
(a) 1 and 2 only above is/are correct?
(b) 2 and 3 only (a) 1 only
(c) 1 and 3 only (b) 2 only
(d) 1, 2 and 3 (c) Both 1 and 2
(d) Neither 1 nor 2
48. With reference to "DigiLocker",
consider the following 51. With reference to the Water
statements: and Power Consultancy Services
1. It is available only for Indian (India) Limited, consider the
citizens following statements:
2. It is a part of Digital India 1. It is a body under the Ministry of
Initiative Water Resources, River
3. Ministry of Railways has Development and Ganga
approved digital Aadhaar and Rejuvenation
Driving Licence from DigiLocker 2. It is a Navratna company
as proof of identity of passengers Which of the statement/s given
Which of the statement(s) given above is/are correct?
above is/are correct? (a) 1 only
(a) 1 and 2 only (b) 2 only
(b) 2 and 3 only (c) Both 1 and 2
(c) 1 and 3 only (d) Neither 1 nor 2
(d) 1, 2 and 3
52. Consider the following
49. With reference to the "Global statements about Coal Mine
Housing Technology challenge",

Have a querry?
Drop us a mail at – hello@civilstap.com or call us at 8146207241
9
Revise your past 1.5 years of current affairs with our most comprehensive Test Series on Current Affairs

Surveillance & Management Which of the above statements


System: is/are correct?
1. It is a GIS application through (a) 1 only
which the location of sites for (b) 2 only
unauthorized mining can be (c) Both 1 and 2
detected (d) Neither 1 nor 2
2. It has been launched by the 56. With reference to ‘North
Ministry of Electronics & Eastern Council’, consider the
Information Technology
following statements:
Which of the statements/s given
above is/are correct? 1. It is a statutory body with
(a) 1 only governors and chief ministers of
(b) 2 only all the eight north eastern states
(c) Both 1 and 2 as its members.
(d) Neither 1 nor 2 2. Union Home-Minister is ex-officio
chairman of this council.
53. With reference to the Which of the statement(s) given
Recapitalization of Regional Rural above is/are correct?
Banks Scheme, which of the (a) 1 only
following statement(s) is/are (b) 2 only
correct? (c) Both 1 and 2
1. It will enable RRBs to maintain (d) Neither 1 nor 2
the minimum prescribed CRAR 57. With reference to ‘Swajal
of 9% Scheme’, consider the following
2. It has been extended upto the statements:
year 2022
Select the correct answer using the 1. It has been launched by the
codes given below: Ministry of Water Resources,
(a) 1 only River Development and Ganga
(b) 2 only Rejuvenation.
(c) Both 1 and 2 2. Under this scheme, Government
(d) Neither 1 nor 2 aims to provide villages with
piped water supply powered by
54. HSV1 virus is sometimes solar energy.
mentioned in the news with Which of the statement(s) given
reference to which of the following above is/are correct?
diseases? (a) 1 only
(a) AIDS (b) 2 only
(b) Swine flu (c) Both 1 and 2
(c) Herpes (d) Neither 1 nor 2
(d) Alzheimer’s 58. ‘Pinaka’ sometimes seen in
55. Consider the following news refers to:
statements about the Indian (a) A multi-barrel rocket launcher
Regional Navigation Satellite (b) An application for grievance
System (IRNSS): redressal
(c) A card payment scheme
1. IRNSS 1I is the eighth satellite in
(d) A portal that enables NGOs to
the satellite system.
enrol centrally
2. IRNSS will help in land navigation
only.

Have a querry?
Drop us a mail at – hello@civilstap.com or call us at 8146207241
10
Revise your past 1.5 years of current affairs with our most comprehensive Test Series on Current Affairs

59. Which of the following 62. Which of the following


statement/s is/are correct in statement(s) is/are correct in
context of ‘World Blood Donors context of ‘Nipah Virus Infection’?
day’? 1. It is transmitted to humans
1. It has been instituted by United through animals
Nations Human Rights Council 2. It was first identified in 1999 in
(UNHRC) Malaysia and Singapore
2. It is one of the eight official global 3. Chickens are the natural host of
public health campaigns marked this virus
by WHO Select the correct answer using the
Select the correct answer using the code given below:
code given below: (a) 1 and 2 only
(a) 1 only (b) 2 and 3 only
(b) 2 only (c) 1 and 3 only
(c) Both 1 and 2 (d) 1, 2 and 3
(d) Neither 1 nor 2
63. ‘Credit Enhancement Fund’
60. With reference to ‘Nuclear launched by the Union
Suppliers Group’, consider the Government aims to:
following statements: (a) Provide education loan for
1. It is a multi-lateral export control students of economically weaker
regime. sections
2. It was founded in response to the (b) Facilitate Infrastructure
Indian nuclear test in 1974. investments by insurance and
3. In 2018, Estonia became the first pension funds
Baltic state to chair the NSG. (c) Facilitate farmers with credit
Which of the statement(s) given availability
above is/are correct? (d) Facilitate infrastructure for
(a) 1 and 2 Women Safety
(b) 2 and 3
(c) 1 and 3 64. Which of the following pair/s
(d) 1, 2 and 3 is/are correctly matched?
Temples Places
61. With reference to “SCOs 1. Sabarimala Kerala
Qingdao Declaration 2018”, which
of the following statement/s
Temple
is/are correct? 2. Raghavendra Tamil
Swamy Temple Nadu
1. It calls for implementing three-
year plan to combat terrorism, 3. Brahma Sarovar Haryana
separatism and extremism. Temple
2. India is the only country to Select the correct answer using the
refuse to endorse One Belt, One codes given below:
Road (OBOR) project as part of (a) 1 and 2 only
this declaration. (b) 2 and 3 only
Select the correct answer using the (c) 1 and 3 only
codes given below: (d) 1, 2 and 3
(a) 1 only
(b) 2 only 65. With reference to ‘Composite
(c) Both 1 and 2 Water Management Index’, which
(d) Neither 1 nor 2 of the following statement/s
is/are correct?

Have a querry?
Drop us a mail at – hello@civilstap.com or call us at 8146207241
11
Revise your past 1.5 years of current affairs with our most comprehensive Test Series on Current Affairs

1. It aims to inspire States/UTs for seeking protection for stray


towards efficient and optimal dogs
utilization of water and recycling. (b) It was a march organised to seek
2. It is released by Ministry of protection from the menace of
Drinking Water and Sanitation. stray dogs
Select the correct answer using the (c) It was a march by Anti-Brexit
code given below: protesters for seeking new
(a) 1 only referendum on BREXIT
(b) 2 only (d) It was a march by the people in
(c) Both 1 and 2 United Kingdom to hold a new
(d) Neither 1 nor 2 referendum on the issue of
Scotland
66. Consider the following pairs
and identify the correct one/s: 69. In context of Index of Eight
Regions / Cities seen in Country Core Industries, which of the
news following is/are correct?
Gaza Israel 1. Between the period 2012-13 to
Caracas Venezuela 2017-18, there has been a steady
Sulawesi China annual growth in the overall
Select the correct code: output of the eight core
(a) 1 only industries.
(b) 1 and 2 only 2. Among the eight core industries,
(c) 2 only ‘Petroleum Refinery production’
(d) None of the above has the maximum and ‘fertilizer
production’ the minimum weight.
67. Operation Samudra Maitri was Select the correct code:
in news recently, which of the (a) 1 only
following correctly describes it? (b) 2 only
(a) It is the name of joint naval (c) Both 1 and 2
exercise held recently between (d) Neither 1 or 2
India and Sri Lanka
70. In context of Index of Eight
(b) It is the name of maritime
exercise held recently between Core Industries, which of the
India and Bangladesh following is/are incorrect?
(c) It was the relief operation 1. Between the period from 2012-13
launched by India to provide to 2017-18, the rate of growth in
relief assistance to earthquake the overall output of the eight
and tsunami victims in Indonesia core industries has been uniform.
(d) It was a rescue operation 2. In the same time period, the
conducted by Indian navy to overall output of the eight core
rescue a Bangladeshi industries has been increasing
commercial vessel in the Indian steadily.
Ocean Region Select the correct code:
(a) 1 only
68. “Wooferendum march” was (b) 2 only
seen in news recently. Which of (c) Both 1 and 2
the following correctly describes (d) Neither 1 or 2
it?
71. Recently China conducted a
(a) It was a march campaign
maiden test flight of an aircraft
organised by the animal activists
called as AG600. Consider the

Have a querry?
Drop us a mail at – hello@civilstap.com or call us at 8146207241
12
Revise your past 1.5 years of current affairs with our most comprehensive Test Series on Current Affairs

following statements regarding it 1. It is the premier workspace for


and identify the correct one/s: Electronics and IT corporates to
1. It has been indigenously designed organise employee engagements
and built by China for social work.
2. It has been code named as 2. It comprises of an ‘admin portal’
‘Kunlong’ and a ‘mobile app’.
3. It has been designed to be the Which of the above is/are correct?
world’s largest amphibious (a) 1 only
aircraft (b) 2 only
Select the correct code: (c) Both 1 and 2
(a) 1 only (d) Neither 1 or 2
(b) 1 and 2 only 75. Consider the following
(c) 1 and 3 only statements regarding “Main Nahin
(d) All of the above Hum App”:
72. Consider the following pairs 1. It is one of the components under
and identify the correct one/s: the ‘Self4Society Platform’.
2. It provides an IT tool to track the
Cyclones Affected
progress of various government
recently in news Region
initiatives.
Leslie Arabian
Which of the above is / are correct?
Peninsula
(a) 1 only
Titli Eastern
(b) 2 only
Coast of
(c) Both 1 and 2
India
(d) Neither 1 nor 2
Luban Iberian
Peninsula 76. An antibody known as
Select the correct code: “p95HER2-TCB” was seen in news
(a) 1 only recently. It has been developed to
(b) 1 and 2 only treat which of the following
(c) 2 only (a) Tuberculosis
(d) All of the above (b) AIDS
73. The term “ASMR” was seen in (c) Breast Cancer
news recently. Which of the (d) Nipah infection
following correctly describes it? 77. Cyclone Titli was in news
(a) It is an anti-aircraft missile recently, which of the following
technology. state/s was/were impacted by it?
(b) It is a type of missile defence 1. Odisha
system intended to shield a 2. Andhra Pradesh
country against incoming 3. West Bengal
missile. 4. Tamil Nadu
(c) It is an experience characterized Select the correct code:
by a tingling sensation on the (a) 1 only
skin that typically begins on the (b) 2 and 3 only
scalp and moves down the back (c) 1, 2 and 3 only
of the neck and upper spine. (d) All of the above
(d) None of the above
78. In context of Ease of Doing
74. Consider the following
Business Index, which of the
statements regarding
“Self4Society” platform:

Have a querry?
Drop us a mail at – hello@civilstap.com or call us at 8146207241
13
Revise your past 1.5 years of current affairs with our most comprehensive Test Series on Current Affairs

following statement/s is/are 81. With reference to ‘Seva Bhoj


correct? yojana’, which of the following
1. It ranks countries of the world statement/s is/are correct?
across 10 indicators. 1. It has been introduced by the
2. India jumped 77 ranks in Ease of Ministry of Consumer Affairs,
Doing Business Index 2018. Food and Public Distribution
Select the correct code: 2. The scheme envisages
(a) 1 only reimbursing the Central
(b) 2 only Government share of Central
(c) Both 1 and 2 Goods and Services Tax (CGST)
(d) Neither 1 or 2 and Integrated Goods and
Service Tax (IGST).
79. Consider the following 3. Under this Yojana, eligible
statements regarding “World religious institution shall first
Economic Outlook” and identify register with Darpan Portal.
the incorrect one/s: Select the correct answer using the
1. It is an annual flagship report of code given below:
the World Bank. (a) 1 and 3 only
2. The report of 2018 was released (b) 2 only
during annual meet of World (c) 2 and 3 only
Bank and IMF held at Bali, (d) 1, 2 and 3
Indonesia.
Select the correct code: 82. Which of the following sailing
(a) 1 only vessel is related to the ‘Navika
(b) 2 only Sagar Parikrama’?
(c) Both 1 and 2 (a) INSV Tarini
(d) Neither 1 or 2 (b) INSV Mhadei
(c) INSV Tarangini
80. National Institute of Mental (d) INSV Sudarshini
Health Rehabilitation was
sometimes seen in news. Which of 83. With reference to ‘Benami
the following statement/s Transaction Informants Reward
regarding it is/are correct? Scheme’, which of the following
1. It will function under Department statement/s is/are correct?
of Health Research. 1. It has been launched by the
2. It will serve as institution of Ministry of Corporate Affairs.
excellence for capacity building in 2. Its objective is to obtain people’s
human resource and research in participation in the efforts to
area of mental health unearth black money and to
rehabilitation. reduce tax evasion.
3. Union Cabinet has decided to set 3. A person can get a reward up to
it up in Sehore District, Madhya Rs. 1 Lakh for giving specific
Pradesh. information.
Select the correct code: Select the correct answer using the
(a) 1 only code given below:
(b) 1 and 3 only (a) 1 and 3 only
(c) 3 only (b) 2 only
(d) 2 and 3 only (c) 2 and 3 only
(d) 1, 2 and 3
84. Consider the following pairs:

Have a querry?
Drop us a mail at – hello@civilstap.com or call us at 8146207241
14
Revise your past 1.5 years of current affairs with our most comprehensive Test Series on Current Affairs

Islands Country (a) 1 only


Sylt Denmark (b) 2 only
Socotra Yemen (c) Both 1 and 2
Corsica France (d) Neither 1 nor 2
Which of the pairs given above are
88. In context of ‘Public Credit
correctly matched?
(a) 1 and 3 only Registry’, which of the following
(b) 2 and 3 only statement/s is/are correct?
(c) 1 only 1. It is an information repository
(d) 1, 2 and 3 that collates all loan information
of individuals and corporate
85. Ensemble Prediction System’ borrowers.
launched by the Ministry of Earth 2. It has been set up on the
Sciences pertains to? recommendations of a committee
(a) Traffic estimation and prediction headed by Viral Acharya.
(b) Weather forecast 3. It has been set up by the Reserve
(c) Effective heart disease prediction Bank of India.
(d) Tsunami prediction Select the correct answer using the
code given below:
86. In context of ‘Kashmir Super (a) 1 and 2 only
50’, which of the following (b) 2 and 3 only
statement/s is/are correct? (c) 1 and 3 only
1. It is a joint initiative by Indian (d) 1, 2 and 3
Army, Center for Social
89. With reference to ‘Dam
Responsibility and Leadership
Rehabilitation & Improvement
(CSRL) and PETRONET LNG
Limited Project (DRIP), which of the
2. It has been launched to transform following statement/s is/are
the educational status of children correct?
from economically weaker 1. It has been launched under
sections in the Kashmir region Ministry of Drinking water and
Select the correct answer using the sanitation
code given below: 2. It was launched in 2012 by
(a) 1 only Central Water Commission
(b) 2 only Select the correct answer using the
(c) Both 1 and 2 code given below:
(d) Neither 1 nor 2 (a) 1 only
(b) 2 only
87. With reference to ‘Lithium-ion (c) Both 1 and 2
Batteries’, which of the following (d) Neither 1 nor 2
statement/s is/are correct?
1. These are common rechargeable 90. Which of the following gives
batteries for portable electronics, ‘Global Peace Index’ ranking to
with a high energy density, the countries of the world?
no memory effect and low self- (a) World Economic Forum
discharge. (b) UN Human Rights Council
2. A group of students at IIT, Delhi (c) Institute for Economics and Peace
has developed an indigenous (d) International Monetary Fund
technology of Lithium-ion cells.
Select the correct answer using the
code given below:

Have a querry?
Drop us a mail at – hello@civilstap.com or call us at 8146207241
15
Revise your past 1.5 years of current affairs with our most comprehensive Test Series on Current Affairs

91. Consider the following (a) World Wildlife Fund for


statements regarding Eco conservation of endangered
Sensitive Zones: species
1. These are also called Ecologically (b) Wildlife Crime Control Bureau for
Vulnerable Areas. work done in combating
2. These are regulated by the transboundary environmental
respective state governments. crime
3. These zones are mentioned in the (c) Food and Agriculture
Environment (Protection) Act, Organization for its efforts in
1986 defeating hunger
4. Elephant Corridors come under (d) Both a and b
such zones 95. Consider the following
Which of the above statement(s) statements regarding Model Code
is/are incorrect? of Conduct:
(a) 1 and 3 only
(b) 4 only 1) It has been given a statutory
(c) 4 only status.
(d) All of the above 2) It can come into action before
polls are announced.
92. Consider the following Which of the above statements
statements regarding “Kamorta”: is/are correct?
1. It is a class of Indian Navy anti- (a) 1 only
submarine warfare stealth (b) 2 only
corvettes to be built in India (c) None is correct
2. It is a village in the Nicobar (d) Both are correct
district of Andaman and Nicobar 96. The Regional Integrated Multi
Islands. Hazard Early Warning System
Which of the above statement(s) is/
(RIMES) was in news recently.
are correct?
Consider the following statements
(a) 1 only
(b) 2 only regarding RIMES:
(c) Both 1 and 2 1. It is an inter governmental body
(d) Neither 1 or 2 registered under the United
Nations.
93. The Public Sector 2. It seeks to establish an early
Undertakings (PSU) index is: warning system for cyclones and
(a) listed on BSE and consists of all earthquakes only.
PSU stocks in the BSE 500 index Which of the above statement/s
(b) listed on NSE and consists of all is/are correct?
PSU stocks in the NIFTY 50 index (a) 1 only
(c) listed on BSE and consists of all (b) 2 only
PSU stocks in the NIFTY 50 index (c) Both 1 and 2
(d) listed on BSE and consists of all (d) Neither 1 nor 2
PSU stocks in the BSE 30 index
97. Consider the following
94. United Nations Environment statements related to Particularly
Programme (UNEP) has awarded Vulnerable Tribal Groups (PVTGs):
Asia Environment Enforcement 1. These are the tribes within the ST
Award 2018 to which of the category which are more
following? vulnerable among the tribal
groups.

Have a querry?
Drop us a mail at – hello@civilstap.com or call us at 8146207241
16
Revise your past 1.5 years of current affairs with our most comprehensive Test Series on Current Affairs

2. These were earlier known as 100. RFID tags are being


Primitive Tribal Groups increasingly used these days for
3. These are identified and approved identifying and tracking objects.
by the Central Ministry of Tribal What is the full form of RFID?
Affairs (a) Radio Frequency Identification
Which of the above statement/s Device
is/are correct? (b) Real Feedback Identification
(a) 1 only Device
(b) 1 and 2 only (c) Radio Feedback Identification
(c) 1 and 3 only Device
(d) 2 only (d) None of the above
98. Consider the following
statements regarding
“Constitution Day”:
1. It is celebrated in order to mark
the coming into effect of the
Constitution of India.
2. It is celebrated on 26th Nov every
year.
Which of the above statement/s
is/are correct?
(a) 1 only
(b) 2 only
(c) Both 1 and 2
(d) Neither 1 nor 2
99. Consider the following
statements:
1. The 73rd Constitutional
Amendment Act, 1992 extends to
the whole of India.
2. Election Commission of India
has the power to oversee the
establishment of electoral rolls as
well as organize elections for
legislative assembly and council
in Jammu and Kashmir.
Which of the above statement/s
is/are incorrect?
(a) 1 only
(b) 2 only
(c) Both 1 and 2
(d) Neither 1 nor 2

Have a querry?
Drop us a mail at – hello@civilstap.com or call us at 8146207241
17
Revise your past 1.5 years of current affairs with our most comprehensive Test Series on Current Affairs

Have a querry?
Drop us a mail at – hello@civilstap.com or call us at 8146207241
18
Revise your past 1.5 years of current affairs with our most comprehensive Test Series on Current Affairs

ABHYAAS – 2019 – CURRENT AFFAIRS TEST


SERIES

SOLUTION PDF
TEST 1

• Each test contains 100 MCQs framed from the most important sources namely The
Hindu, The Indian Express, LiveMint, PIB and PRS
• Current affairs of national and international importance are covered from Jan 2018
to May 2019 for UPSC Prelims 2019 to be held on 2nd June 2019.
• All tests can be attempted online as well as offline (through downloadable question
paper)Each question is accompanied with detailed explanation
• You can get in touch with us over call and whatsapp – 8146207241 as well as email
– hello@civilstap.com
• Visit our website – www.civilstap.com

Have a querry?
Drop us a mail at – hello@civilstap.com or call us at 8146207241
19
Revise your past 1.5 years of current affairs with our most comprehensive Test Series on Current Affairs

{QUESTION NUMBER} 1
{TAGS} Government Schemes and programmes
With reference to the National Knowledge Network (NKN), consider the following
statements:
1. It is a multi-gigabit pan-India resource sharing network which aims to connect all
universities, research institutions, libraries across the country
2. India has started to extend its KNK to research and educational networks in
foreign countries
Which of the statements given above is/are correct?
{OPTION 1} 1 only
{OPTION 2} 2 only
{OPTION 3} Both 1 and 2
{OPTION 4} Neither 1 nor 2
{RIGHT ANSWER} 3
{EXPLANATION}
Recent Context:
• In January 2018, India kicked off the process of appointing a telecom company
that will connect and extend its state-of-the art National Knowledge Network
(NKN) to research and education networks in six South Asian Association of
Regional Cooperation member states — Afghanistan, Bangladesh, Bhutan,
Maldives, Nepal and Sri Lanka.
• Pakistan is the only SAARC nation that has been left out of this initiative.
• In March 2019, India has decided to extend its National Knowledge Network to
Bangladesh.

About NKN:
• It is aimed at digitally connecting all national universities, colleges and research
establishments to create “country-wide virtual classrooms”.
• For example, it enables collaboration among researchers from different
educational networks such as TEIN4, GARUDA, CERN and Internet2.
• It also aims to connect laboratories, healthcare and agricultural institutions
across the country.
• The leading mission-oriented agencies in the fields of nuclear, space and
defence research are also part of NKN.
• This project was initiated in the year 2009.
• Currently, this programme is a component of umbrella, ‘Digital India
Programme’.

Have a querry?
Drop us a mail at – hello@civilstap.com or call us at 8146207241
20
Revise your past 1.5 years of current affairs with our most comprehensive Test Series on Current Affairs

It provides three types of services:


• Generic services like Internet, Intranet, e-Mail, Messaging Gateways, SMS
Gateway etc.
• Community Services like Shared Storage, e-Mail List Software Application
(LISTSERV), International Collaborations with EU-India Grid, Global Ring Network
for Advanced Applications Development (GLORIAD) etc.
• Special Services like Virtual Private Network Stitching Services.

So, in one line we can sum up that KNK is a revolutionary step towards creating
a knowledge society without boundaries.

https://economictimes.indiatimes.com/news/politics-and-nation/india-excludes-
pakistan-from-a-saarc-initiative/articleshow/62345991.cms

{QUESTION NUMBER} 2
{TAGS} Science and Tech
What is ‘DS Faraday’, a term recently seen in the news?
{OPTION 1} Wheat variety
{OPTION 2} Electromagnetic wave
{OPTION 3} Object in the outer space
{OPTION 4} New element in the periodic table
{RIGHT ANSWER} 1
{EXPLANATION} Recent Context:
Australian scientists have developed the world’s first ‘speed breeding’ technique
that can boost the production of the crop by up to three times.
What is DS Faraday?
• Using this ‘speed breeding’ technique, the scientists have developed a high
protein, milling wheat variety, DS Faraday which is tolerant to pre-harvest
sprouting.

About the speed breeding technique:


• This technique has been inspired from NASA’s experiments to grow wheat in
space.
• The NASA experiments involved using continuous light on wheat which triggered
early reproduction in the plants.
• The scientists used this idea in specially modified glasshouses to grow six
generations of wheat, chickpea and barley plants and four generations of canola

Have a querry?
Drop us a mail at – hello@civilstap.com or call us at 8146207241
21
Revise your past 1.5 years of current affairs with our most comprehensive Test Series on Current Affairs

plants in a single year – as opposed to two or three generations in a regular


glasshouse, or a single generation in the field.

https://economictimes.indiatimes.com/news/science/nasa-inspired-speed-breeding-
method-boosts-wheat-production/articleshow/62336506.cms
{QUESTION NUMBER} 3
{TAGS} Government Schemes and programmes
With reference to ‘Pradhan Mantri Swasthya Suraksha Yojana (PMSSY)’, which
of the following statement is/are correct?
1. It is a health insurance scheme launched by the Ministry of Health and Family
Welfare
2. One of the components of PMSSY is the setting up of AIIMS like institutions in
India
Select the correct answer using the code given below.
{OPTION 1} 1 only
{OPTION 2} 2 only
{OPTION 3} Both 1 and 2
{OPTION 4} Neither 1 nor 2
{RIGHT ANSWER} 2
{EXPLANATION} Recent Context:
The Union Cabinet approved setting up of an AIIMS in Bilaspur in Himachal Pradesh
in January 2018 under the Pradhan Mantri Swasthya Surakhsa Yojna (PMSSY).
The Statement No.1 is incorrect.
The Pradhan Mantri Swasthya Suraksha Yojana (PMSSY) was announced with
objectives of correcting regional imbalances in the availability of affordable/
reliable tertiary healthcare services and also to augment facilities for quality
medical education in the country.
PMSSY has two components:
(i) Setting up of AIIMS like Institutions
(ii) Upgradation of Government Medical College Institutions

When was the scheme launched?


In his Independence Day speech on 15th August 2003 the then Hon’ble Prime
Minister had announced the setting up of six AIIMS type hospitals under Pradhan
Mantri Swasthya Suraksha Yojana (PMSSY). The scheme was approved in
March 2006.

What is the total number of AIIMS?

Have a querry?
Drop us a mail at – hello@civilstap.com or call us at 8146207241
22
Revise your past 1.5 years of current affairs with our most comprehensive Test Series on Current Affairs

Phase and Year of AIIMS like institutions Total


announcement in number
Budget of
AIIMS
Phase I (2006) Bihar (Patna), Chhattisgarh 6
(Raipur), Madhya Pradesh
(Bhopal), Orissa
(Bhubaneswar), Rajasthan
(Jodhpur) and Uttaranchal
(Rishikesh)
Phase II (2009) West Bengal (Shifted to Phase 1
IV), Uttar Pradesh (Rae
Bareli)
Phase III (2013) No new AIIMS 0
Phase IV (2014-15) Andhra Pradesh (Mangalagiri), 4
Maharashtra (Nagpur), Uttar
Pradesh (Gorakhpur), West
Bengal (Kalyani)
Phase V (2015-16) Jammu, Kashmir, Punjab, Tamil 7
Nadu, Himachal Pradesh,
Assam and Bihar
Phase V A (2016-17) No new AIIMS 0
Phase VI (2017-18) Gujarat and Jharkhand 2
Phase VI A Telangana 1
Phase VII (2019-20) Haryana 1
https://www.thehindu.com/news/national/other-states/cabinet-nod-for-aiims-in-
himachal-pradesh/article22359283.ece

{QUESTION NUMBER} 4
{TAGS} International News
What is ‘GAVI’, a term sometimes seen in the news?
{OPTION 1} a global vaccine alliance
{OPTION 2} a web-portal connecting all the schools in India
{OPTION 3} an indigenous satellite launch vehicle
{OPTION 4} a government portal for defence procurement
{RIGHT ANSWER} 1
{EXPLANATION} Recent Context:

Have a querry?
Drop us a mail at – hello@civilstap.com or call us at 8146207241
23
Revise your past 1.5 years of current affairs with our most comprehensive Test Series on Current Affairs

In January 2018, The Global Alliance for Vaccines and Immunization (GAVI) has
created a $50-million new cold chain fund and it plans to tap the huge potential
in India, which has recently embarked on Mission Indradhanush to push vaccines
for all children in India.
What is GAVI?
Gavi, based in Geneva, Switzerland, is an international organisation that was
created in 2000 to bring together the best of what key UN agencies,
governments, the vaccine industry, private sector and civil society had to offer
in order to improve childhood immunisation coverage in poor countries and
to accelerate access to new vaccines.
https://www.thehindubusinessline.com/companies/gavi-to-help-modernise-vaccine-
storage-in-india-with-50million-fund/article8082674.ece

{QUESTION NUMBER} 5
{TAGS} Indian institutions and bodies in news
Which of the following statements is/are correct regarding ‘The Indian Council for
Cultural Relations (ICCR)’?
1. It is an organization coming under the Ministry of Tourism
2. It was founded in 1950 by Independent India’s first Education Minister
Select the correct answer using the code given below:
{OPTION 1} 1 only
{OPTION 2} 2 only
{OPTION 3} Both 1 and 2
{OPTION 4} Neither 1 nor 2
{RIGHT ANSWER} 2
{EXPLANATION} Recent context:
In January 2018, the President Ram Nath Kovind appointed Rajya Sabha MP Vinay
Sahasrabuddhe as the President of the Indian Council of Cultural Relations
(ICCR).
About ICCR:
The Indian Council for Cultural Relations (ICCR) was founded in 1950 by Maulana
Abul Kalam Azad, independent India’s first Education Minister.
It is an autonomous organisation of the Government of India, tasked with
strengthening India’s external cultural relations through exchanges with other
countries and their people.
https://www.business-standard.com/article/news-ians/bjp-s-sahasrabuddhe-
appointed-iccr-president-117122901082_1.html

Have a querry?
Drop us a mail at – hello@civilstap.com or call us at 8146207241
24
Revise your past 1.5 years of current affairs with our most comprehensive Test Series on Current Affairs

{QUESTION NUMBER} 6
{TAGS} Government Schemes and programmes
With reference to the Electoral Bond Scheme, consider the following statements:
1. Any Indian resident or incorporated body (such as a company) is eligible to
purchase an electoral bond
2. The bonds will have to mandatorily carry the name of the purchaser
3. The bonds can be encashed by parties only through a designated account with an
authorized bank
Which of the statement(s) given above is/are incorrect?
{OPTION 1} 3 only
{OPTION 2} 1 and 2 only
{OPTION 3} 2 and 3 only
{OPTION 4} 1 only
{RIGHT ANSWER} 2
{EXPLANATION} Recent context: The Scheme of electoral bonds was
announced as part of the Union Budget 2017-18. It was notified by the Ministry
of Finance in January 2018.
Under the scheme, a new instrument called an electoral bond will be issued to
make donations to political parties.
Firstly, what is an electoral bond?
• It is a bond issued in the nature of promissory note which shall be a bearer
banking instrument.
• It neither carries the name of the buyer nor the payee.
• A bearer instrument is one where no ownership information is recorded and the
holder of the instrument is presumed to be the owner.
• The one (eligible political party in this case) in possession of the electoral bond is
entitled to encash the same.

Let us have a look at the features of the scheme:


• Any Indian citizen or an incorporated body (such as a company) will be eligible
to purchase an electoral bond.
• A person can buy electoral bonds, either single or jointly with other individuals.
The purchaser will have to make the payment from his bank account and comply
with the Know Your Customer norms of the bank. The bonds will not carry the
name of the purchaser.

Have a querry?
Drop us a mail at – hello@civilstap.com or call us at 8146207241
25
Revise your past 1.5 years of current affairs with our most comprehensive Test Series on Current Affairs

• These bonds will be issued by certain branches of the State Bank of India in
multiples of: (i) Rs 1,000, (ii) Rs 10,000, (iii) Rs one lakh, (iv) Rs ten lakh, and (v) Rs
one crore.
• Electoral bonds may only be used to make donations to registered political
parties (A party should be registered under section 29A of the Representation of
the People Act) which secured at least 1% of the votes in the last Lok Sabha or
State Assembly election. These bonds can be encashed by parties only through
a designated account with an authorised bank.
• Electoral bonds will be available for 10 days each in January, April, July and
October. Bonds will be available for purchase for another 30 days in the year of
the Lok Sabha elections.
• The bonds will have to be encashed within 15 days of issue and will not be eligible
for trading.
• The contribution made by an individual/company through an electoral bond will
be exempt from income tax as per Sections 80GGC/80GGB of the Income Tax
Act. Further, the contribution received by any eligible political party in the form
of electoral bonds will be exempt from income tax as per Section 13A of the
Income Tax act.

Important point to note:


Donations below Rs.2000 can still be received in cash by the political parties.

{QUESTION NUMBER} 7
{TAGS} Indian institutions and bodies in news
Which of the following statements is/are correct regarding the New Delhi International
Arbitration Centre (NDIAC)?
1. It is a statutory body established to conduct arbitration, mediation, and
conciliation proceedings
2. It subsumes the International Centre for Alternative Dispute Resolution
3. It will be headed by the Union Minister of External Affairs
Which of the statements given above is/are correct?
{OPTION 1} 2 and 3 only
{OPTION 2} 1 only
{OPTION 3} 1 and 2 only
{OPTION 4} 1, 2 and 3
{RIGHT ANSWER} 3
{EXPLANATION} Recent Context:

Have a querry?
Drop us a mail at – hello@civilstap.com or call us at 8146207241
26
Revise your past 1.5 years of current affairs with our most comprehensive Test Series on Current Affairs

• The New Delhi International Arbitration Centre Bill, 2018 was introduced in Lok
Sabha in January 2018.
• But the bill lapsed as it could not get clearance from upper house before the
expiry of the budget session.
• Thus, to bring this bill into effect, the President of India, Ram Nath Kovind
promulgated the New Delhi International Arbitration Centre (NDIAC) Ordinance
in March 2019.
• NDIAC would subsume the International Centre for Alternative Dispute
Resolution set-up in the year 1995, which is a society registered under the
Societies Registration Act.

What are going to be the benefits of this ordinance?


• The benefits of institutionalized arbitration will accrue to Government and its
agency and to the parties to a dispute.
• This shall be to the advantage of the public and the public institutions in terms
of quality of expertise and costs incurred and will facilitate India becoming a hub
for Institutional Arbitration.

What are the objectives of the Ordinance?


The NDAIC shall be established with an aim to:
✓ bring targeted reforms to develop itself as a flagship institution for
conducting international and domestic arbitration;
✓ provide facilities and administrative assistance for conciliation mediation and
arbitral proceedings;
✓ maintain panels of accredited arbitrators, conciliators and mediators both at
national and international level or specialists such as surveyors and investigators;
✓ facilitate conducting of international and domestic arbitrations and conciliation
in the most professional manner;
✓ provide cost effective and timely services for the conduct of arbitrations and
conciliations at Domestic and International level;
✓ promote studies in the field of alternative dispute resolution and related
matters, and to promote reforms in the system of settlement of disputes; and
✓ co-operate with other societies, institutions and organisations, national or
international for promoting alternative dispute resolution.

Members of NDIAC:

Have a querry?
Drop us a mail at – hello@civilstap.com or call us at 8146207241
27
Revise your past 1.5 years of current affairs with our most comprehensive Test Series on Current Affairs

• New Delhi International Arbitration Centre (NDIAC) will be headed by a


chairperson who has been a Judge of the Supreme Court or a Judge of a High
Court or an eminent person, having special knowledge and experience in the
conduct or administration of arbitration law or management, to be appointed
by the Central Government in consultation with the Chief Justice of India.
• There will be two Full time or Part time Members from amongst eminent
persons having substantial knowledge and experience in institutional
arbitration, both domestic and international.
• Also, one representative of a recognised body of commerce and industry shall
be chosen on rotational basis as Part time Member.
• Secretary, Department of Legal Affairs, Financial Adviser nominated by the
Department of Expenditure and Chief Executive Officer, NDIAC shall be ex-
officio Members.
{QUESTION NUMBER} 8
{TAGS} Government Schemes and programmes
Which of the following statements is/are correct regarding the e-Samvad portal?
1. It is an online platform for NGOs and Civil Societies to interact with the Ministry
of Health and Family Welfare
2. It would help the government in the formulation of effective policies and
measures based on the feedbacks/suggestions provided by the NGOs
Select the correct answer using the code given below:
{OPTION 1} 1 only
{OPTION 2} 2 only
{OPTION 3} Both 1 and 2
{OPTION 4} Neither 1 nor 2
{RIGHT ANSWER} 2
{EXPLANATION} Recent context:
The e-Samvad portal was launched by the Ministry of Women and Child
Development in January 2018.
What is the use of this portal?
• It is an online platform for NGOs and Civil Societies to interact with the Ministry
of Women & Child Development.
• Through e-Samvad portal, NGOs and Civil Society can provide their feedback,
suggestions, put up grievances, share best practices etc. Senior Officers within
MWCD will be able to view the inputs/suggestions received for their concerned
subject areas and appropriately respond to NGOs.

Have a querry?
Drop us a mail at – hello@civilstap.com or call us at 8146207241
28
Revise your past 1.5 years of current affairs with our most comprehensive Test Series on Current Affairs

• This will help in formulation of effective policies and measures for welfare of
women and children.

{QUESTION NUMBER} 9
{TAGS} Space in news, Science and Tech
What is 'Parker Solar Probe', recently in the news?
{OPTION 1} Mankind’s first mission to touch the Sun launched by NASA
{OPTION 2} Mankind’s first mission to enter the outer space launched by NASA
{OPTION 3} Mission to test the presence of water on Mars launched by NASA
{OPTION 4} Mankind’s first mission to touch the Sun launched by ISRO
{RIGHT ANSWER} 1
{EXPLANATION} Recent context:
The world’s first mission to touch the Sun was launched by NASA in the year
2018.
What is Parker Solar Probe?
• It will perform the closest-ever observations of the Sun’s outer atmosphere,
called the crona, a region of the Sun only seen from the Earth when the Moon
blocks out the Sun’s bright face during total solar eclipses.
• The corona holds the answers to many of scientists’ outstanding questions
about the Sun’s activity and processes.
• It was launched on an unprecedented, seven-year long journey to unlock the
mysteries of the sun’s fiery outer atmosphere and its effects on space weather.
• The probe was launched from Space Launch Complex 37 at Cape Canaveral Air
Force Station in the US (Florida) using the United Launch Alliance Delta IV Heavy
Rocket (Launch vehicle).

{QUESTION NUMBER} 10
{TAGS} Science and Tech
The term ‘Retrotransposons’ seen in the news refers to?
{OPTION 1} Small genetic sequences that can lead to mutation in their host’s
genome
{OPTION 2} Small genetic sequences that act as scissors to cut the sections of their
host’s genome
{OPTION 3} Small genetic sequences that have the ability to replicate and position
themselves in new locations in their host’s genome
{OPTION 4} Small genetic sequences that are responsible for divisions of cells in
their host’s genome

Have a querry?
Drop us a mail at – hello@civilstap.com or call us at 8146207241
29
Revise your past 1.5 years of current affairs with our most comprehensive Test Series on Current Affairs

{RIGHT ANSWER} 3
{EXPLANATION} Recent context:
Researchers from King Abdullah University of Science and Technology (KAUST) in
Saudi Arabia have identified special genes, called retrotransposons, which
could help Symbiodinium adapt more rapidly to heat stress.
What is Symbiodinium?
It is a unicellular alga that provides its coral host with photosynthetic products in
return for nutrients and shelter.
What is special about this gene?
The gene identified improves the heat tolerance of the algae that live symbiotically
with coral species and could potentially help the corals adapt to some warming.
Usually what happens?
Coral Bleaching:
• High sea temperatures can cause the breakdown of the symbiotic relationship
between the algae and corals and lead to the widespread expulsion of
Symbiodinium from host tissues, an event known as coral beaching.
• If bleached corals do not recover, they starve to death, leaving only their white,
calcium-carbonate exoskeleton.
Now coming to these genes, generally what are retrotransposons?
Retrotransposons are small genetic sequences that have the ability to replicate and
position themselves in new locations in their host’s genome.

{QUESTION NUMBER} 11
{TAGS} Government Policies
With reference to India’s FDI Policy in Single Brand Retail Trading, consider the
following statements:
1. 100% FDI in Single Brand Retail Trading is allowed under the automatic route
2. A Single Brand Retail Chain is expected to sell all its products under only one label
across its stores in India
3. In cases, where the FDI exceeds 51%, 30% of the value of goods purchased must
be from India
Which of the above statements is/are correct?
{OPTION 1} Only 1
{OPTION 2} 1 and 3
{OPTION 3} 1 and 2
{OPTION 4} 1, 2, 3
{RIGHT ANSWER} 2

Have a querry?
Drop us a mail at – hello@civilstap.com or call us at 8146207241
30
Revise your past 1.5 years of current affairs with our most comprehensive Test Series on Current Affairs

{EXPLANATION} The second statement is incorrect.


Why?
What is Single Brand Product Retail Trading?
• Here there are certain conditions that have to be met:
• Products to be sold should be of a ‘Single Brand’ only.
• Products should be sold under the same brand internationally i.e. products
should be sold under the same brand in one or more countries other than India.
• ‘Single Brand’ product-retail trading would cover only products which are
branded during manufacturing.

Let us have a look at the rules pertaining to India’s FDI Policy in Single Brand
Retail Trading:

• In respect of proposals involving foreign investment beyond 51%, sourcing of


30% of the value of goods purchased, will be done from India, preferably from
MSMEs, village and cottage industries, artisans and craftsmen, in all sectors.
• The quantum of domestic sourcing will be self-certified by the company, to be
subsequently checked, by statutory auditors, from the duly certified accounts
which the company will be required to maintain.
• This procurement requirement would have to be met, in the first instance, as an
average of five years’ total value of the goods purchased, beginning 1st April of
the year of the commencement of the business i.e. opening of the first store.
• Thereafter, it would have to be met on an annual basis.
• Note: The FDI Policy is released by the Department for Promotion of Industry
and Internal Trade (DPIIT), Ministry of Commerce and Industry.

{QUESTION NUMBER} 12
{TAGS} Places, regions and communities in news
The term ‘The National Register of Citizens’ seen in the news is related to which
of the following states in India?
{OPTION 1} Jammu and Kashmir
{OPTION 2} Manipur
{OPTION 3} Assam
{OPTION 4} Telangana
{RIGHT ANSWER} 3

Have a querry?
Drop us a mail at – hello@civilstap.com or call us at 8146207241
31
Revise your past 1.5 years of current affairs with our most comprehensive Test Series on Current Affairs

{EXPLANATION} The National Register of Citizens:


• The National Register of Citizens (NRC) is a list that contains names of Indian
citizens of Assam. It was last prepared after Census in 1951.
• It was prepared by recording particulars of all the persons enumerated during
that Census.
• Assam, which had faced an influx of people from Bangladesh since the early 20th
century, is the only state having an NRC.
• Since 1951, the process of identification of illegal immigrants in Assam has been
debated and has become a contentious issue in the State’s politics.
• A six-year agitation demanding identification and deportation of illegal
immigrants was launched by the All Assam Students’ Union (AASU) in 1979. It
culminated with the signing of the Assam Accord on August 15, 1985.
• The Government of India has been working to update the list recently.
• The Assam government released the final draft of NRC on July 30, 2018. The list
incorporates names of 2.89 crore people out of 3.29 crore applicants.
• The draft includes the names of Indian citizens who have been residing in Assam
before March 25, 1971.

{QUESTION NUMBER} 13
{TAGS} Biodiversity in news
Recently, our scientists have discovered a new frog species which has been
named after wildlife scientist Mewa Singh. This species belongs to a genus
endemic to the Western Ghats. This species has been discovered in which of
the following wildlife sanctuary?
{OPTION 1} Malabar Wildlife Sanctuary
{OPTION 2} Tansa Sanctuary
{OPTION 3} Cotigao Sanctuary
{OPTION 4} Dandeli Sanctuary
{RIGHT ANSWER} 1
{EXPLANATION} Scientists have discovered a new frog species from
Kozhikode’s Malabar Wildlife Sanctuary.
• The frog species, named Mewa Singh’s Night frog, belongs to a genus endemic to
the Western Ghats.
• The frog has been named after wildlife scientist Mewa Singh, in honour of his
contributions to behavioural ecology and primate studies.
What are the important characteristics of the frog?

Have a querry?
Drop us a mail at – hello@civilstap.com or call us at 8146207241
32
Revise your past 1.5 years of current affairs with our most comprehensive Test Series on Current Affairs

• It belongs to genus Nyctibatrachus (commonly known as night frogs) endemic


only to Western Ghats mountain range.
• The frog’s genetically closest relatives are the Athirappilly night frog (found
south of the Palakkad Gap in Thrissur and Idukki) and the Kempholey night frog
(found in the northern Western Ghats of Kerala and Karnataka).
• These frogs are found only in the Western Ghats mountain range. After the latest
discovery, the total number of night frogs has reached to 36.

{QUESTION NUMBER} 14
{TAGS} Terms in news
The term, ‘Channel Stuffing’ in business refers to which of the following?
{OPTION 1} A business practice wherein a company resorts to unfair means to sell
its goods/services to the end consumers
{OPTION 2} A business practice wherein a company resorts to international
channels to market its goods/services
{OPTION 3} A business practice wherein a company takes more credit than it can
repay for the production of goods/services in a year
{OPTION 4} A business practice wherein a company produces and delivers more
goods to retailers than what would be sold successfully to end consumers
{RIGHT ANSWER} 4
{EXPLANATION} This refers to a business practice wherein a company produces
and delivers more goods to retailers than what could be sold successfully to
end consumers.
• Companies generally resort to channel stuffing to temporarily boost sales and
profits, thus presenting a rosy picture to investors, although they have not yet
received payment in cash.
• The sale of goods is recorded under accounts receivable, and it is likely to be
reversed when the retailer returns the unsold goods back to the company, thus
eventually causing sales and profits to drop.
• Channel stuffing may fail to boost stock prices if investors begin to doubt the
quality of a company’s sales.
• Channel stuffing is not confined to the wholesale and retail trade; it can take
place in the industrial sector, high tech industry, and pharmaceutical industry as
well.
• This fraudulent practice is usually done in an attempt to raise the value of the
stock or prevent its fall upon release of quarterly or annual results.

Have a querry?
Drop us a mail at – hello@civilstap.com or call us at 8146207241
33
Revise your past 1.5 years of current affairs with our most comprehensive Test Series on Current Affairs

{QUESTION NUMBER} 15
{TAGS} Terms in news
The terms, ‘Ethereum’, ‘Ripple’, ‘Monero’, ‘Dash’ often heard in news are types of
which of the following?
{OPTION 1} Supercomputers
{OPTION 2} Cryptocurrencies
{OPTION 3} Software Programming Languages
{OPTION 4} Digital Payment Systems
{RIGHT ANSWER} 2
{EXPLANATION} What is a cryptocurrency?
• This is an algorithm powered currency used as tokens in select online
communities and backed by certain technologies, assets or projects.
• They can be used for mostly peer to peer payments, and a few for direct, real-life
transactions.
• Their value is often determined by demand, supply, and algorithmic
parameters.
• These have their own self-regulating networks and ecosystems, are
decentralized, and aren’t reliant on third parties like banks to function and
execute transactions.
• Bitcoin is currently the most popular cryptocurrency in the market, followed
by Ethereum and a bunch of others like Ripple, Litecoin, Electrone, Binance etc.
• Cryptocurrencies are a type of Virtual Currencies (VCs).
• The point to note here is that VCs don’t have any intrinsic value and are not
backed by any kind of assets.
• The price of these currencies is entirely a matter of speculation resulting in spurt
and volatility in their prices.”
Why is it a cause of concern?
• Virtual currency (VC) transactions are encrypted and are likely being used for
drug trafficking, money laundering, terror-funding and smuggling.
• Besides, VCs are not reliable as they are stored in digital/electronic format,
making them vulnerable to hacking and malware attack.
What is the status of cryptocurrencies in India?
Since July 2018, banks have been barred by the Reserve Bank of India (RBI) from
undertaking any business relation with crypto bourses or traders.
Latest developments:

Have a querry?
Drop us a mail at – hello@civilstap.com or call us at 8146207241
34
Revise your past 1.5 years of current affairs with our most comprehensive Test Series on Current Affairs

There is a committee which is headed by the Economic Affairs Secretary, Subhash


Chandra Garg which is in the process of coming out with the draft regulations
for virtual currencies.

{QUESTION NUMBER} 16
{TAGS} Government Schemes and programmes
With reference to the ‘National Clean Air Programme’, consider the following
statements:
1. The goal of the NCAP is to meet the prescribed annual average ambient air quality
standards at all locations in the country in a stipulated timeframe (long-term)
2. This will be a mid-term, five-year action plan to begin with keeping 2019 as the
first year
3. The target of NCAP is 20-30% reduction of PM2.5 and PM10 concentration by 2024
keeping 2019 as the base year for comparison of concentration
Which of the statement(s) given above is/are correct?
{OPTION 1} 3 only
{OPTION 2} 1 and 3
{OPTION 3} 1 and 2
{OPTION 4} 1, 2 and 3
{RIGHT ANSWER} 3
{EXPLANATION} Recent Context:
• In January 2018, the government had formulated National Clean Air Programme
(NCAP) as a medium-term national level strategy to tackle the increasing air
pollution problem across the country in a comprehensive manner.
• In January 2019, The Ministry of Environment, Forest and Climate Change was
launched this programme.

Why the need?


• Air pollution is one of the biggest global environmental challenges of today.
• Thus to tackle the increasing air pollution problem across the country in a
comprehensive manner, a time bound national level strategy for pan India
implementation has been launched.

Goal of NCAP:
It is to meet the prescribed annual average ambient air quality standards at all
locations in the country in a stipulated timeframe (long-term).
Target:

Have a querry?
Drop us a mail at – hello@civilstap.com or call us at 8146207241
35
Revise your past 1.5 years of current affairs with our most comprehensive Test Series on Current Affairs

• The tentative national level target of 20%–30% reduction of PM2.5 and PM10
concentration by 2024 is proposed under the NCAP.
• This is keeping 2017 as the base year for the comparison of concentration.

Tenure:
• This will be a mid-term, five-year action plan to begin with keeping 2019 as the
first year.
• However, the international experiences and national studies indicate that
significant outcome in terms of air pollution initiatives are visible only in the long-
term, and hence the programme may be further extended to 20–25 years in the
long-term after a mid-term review of the outcomes.

Coverage:
The target has been to use the smart cities framework to launch the NCAP in the
43 smart cities falling in the list of the 102 non-attainment cities.
Approach:
Collaborative, multi-scale and cross-sectoral coordination between the relevant
central ministries, state governments and local bodies.
Implementation of NCAP:
The CPCB (Central Pollution Control Board) shall, in consonance with the Air
(Prevention and control of Pollution) Act, 1981, and in particular with the
provision of Section 16(2)(b) of the Act, execute the nation-wide programme
for the prevention, control, and abetment of air pollution within the
framework of the NCAP.
What are non-attainment cities?
The CPCB has identified a list of polluted cities in which the prescribed NAAQS
(National Ambient Air Quality Standards) are violated. These cities have been
identified based on the ambient air quality data obtained (2011–2015) under
NAMP (National Air Quality Monitoring Programme).
http://pib.nic.in/PressReleseDetail.aspx?PRID=1515772

{QUESTION NUMBER} 17
{TAGS} Government Schemes and programmes
With reference to Pradhan Mantri Vaya Vandana Yojana, consider the following
statements:
1. It was launched in the year 2017 by the Ministry of Finance
2. LIC has been given the sole privilege to operate the scheme

Have a querry?
Drop us a mail at – hello@civilstap.com or call us at 8146207241
36
Revise your past 1.5 years of current affairs with our most comprehensive Test Series on Current Affairs

3. It provides social security during old age and to protect elderly persons aged 55
years and above
Which of the statement(s) given above is/are correct?
{OPTION 1} 1 only
{OPTION 2} 1 and 2 only
{OPTION 3} 1 and 3 only
{OPTION 4} 1, 2 and 3
{RIGHT ANSWER} 2
{EXPLANATION} Recent Context:
The subscriptions for The Pradhan Mantri Vaya Vandana Yojana (PMVVY) were
announced open till 3rd May 2018.
About Pradhan Mantri Vaya Vandana Yojana (PMVVY):
• The scheme was launched in the year 2017 by the Ministry of Finance.
• It is a Pension Scheme announced by the Government of India exclusively for
the senior citizens aged 60 years and above.
• The scheme enables old age income security for senior citizens through provision
of assured pension/return linked to the subscription amount based on
government guarantee to Life Insurance Corporation of India (LIC).

Key benefits of the scheme:


• The scheme provides an assured return of 8% per annum for 10 years.
• The differential return, i.e. the difference between return generated by LIC and
the assured return of 8% per annum would be borne by Government of India as
subsidy on an annual basis.
• Pension is payable at the end of each period during the policy tenure of 10 years
as per the frequency of monthly/quarterly/ half-yearly/yearly as chosen by the
subscriber at the time of purchase.
• Minimum purchase price under the scheme is Rs.1,50,000/- for a minimum
pension of Rs. 1,000/- per month.
• The scheme is exempted from Goods and Services Tax (GST).

Some Important points:


• The scheme has been extended upto March 2020.
• The current investment limit is also increased to Rs. 15 lakh from the existing
limit of Rs. 7.5 lakh per senior citizen.
http://pib.nic.in/PressReleseDetail.aspx?PRID=1515643

Have a querry?
Drop us a mail at – hello@civilstap.com or call us at 8146207241
37
Revise your past 1.5 years of current affairs with our most comprehensive Test Series on Current Affairs

{QUESTION NUMBER} 18
{TAGS} Government Schemes and programmes
With reference to the ‘National Tobacco Control Programme’, consider the following
statement:
1. It was launched in the year 2007-08 by the Ministry of Health and Family Welfare
2. One of the objectives of the programme is to ensure effective implementation of
the provisions under COPTA, 2003
3. The National Tobacco Control Cell (NTCC) is responsible for overall policy
formulation, planning, implementation, monitoring and evaluation of the
different activities envisaged under the programme
Which of the statement(s) given above is/ are correct?
{OPTION 1} 1 and 2 only
{OPTION 2} 2 and 3 only
{OPTION 3} 2 only
{OPTION 4} 1, 2 and 3
{RIGHT ANSWER} 4
{EXPLANATION} Recent Context:
• In January 2018, The Committee on Subordinate Legislation (COSL) 16th Lok
Sabha, on the Cigarettes and Other Tobacco Products (Packaging and Labelling)
Amendment Rules, 2014, in its 11thReport felt a need for framing a “National
Tobacco Control Policy” which should be equitable and pragmatic.
• An Inter-Ministerial Committee of Secretaries has been constituted at the
national level to come out with the same.
• At present, we have the National Tobacco Control Programme (NTCP) in place.

About the National Tobacco Control Programme:


It was launched by the Ministry of Health and Family Welfare in 2007-08.
Objectives of NTCP:
(i) create awareness about the harmful effects of tobacco consumption;
(ii) reduce the production and supply of tobacco products
(iii) ensure effective implementation of the provisions under “The Cigarettes and
Other Tobacco Products (Prohibition of Advertisement and Regulation of Trade
and Commerce, Production, Supply and Distribution) Act, 2003” (COTPA)
(iv) help the people quit tobacco use, and
(v) facilitate implementation of strategies for prevention and control of tobacco
advocated by WHO Framework Convention of Tobacco Control.
Implementation of the programme:

Have a querry?
Drop us a mail at – hello@civilstap.com or call us at 8146207241
38
Revise your past 1.5 years of current affairs with our most comprehensive Test Series on Current Affairs

• NTCP is implemented through a three-tier structure, i.e. (i) National Tobacco


Control Cell (NTCC) at Central level (ii) State Tobacco Control Cell (STCC) at State
level & (iii) District Tobacco Control Cell (DTCC) at District level. There is also a
provision of setting up Tobacco Cessation Services at District level.
• The NTCC at the National level is responsible for overall policy formulation,
planning, implementation, monitoring and evaluation of the different activities
envisaged under the National Tobacco Control Programme (NTCP).
http://pib.nic.in/PressReleseDetail.aspx?PRID=1515667

{QUESTION NUMBER} 19
{TAGS} Government Schemes and programmes
‘Mission Raftar’ seen in the news relates to which of the following?
{OPTION 1} Doubling the rate at which the Indian economy is growing
{OPTION 2} Increasing the proportion of passengers travelling by air route
{OPTION 3} Speeding up India’s progress towards achieving the SDGs
{OPTION 4} Doubling of average speed of freight trains
{RIGHT ANSWER} 4
{EXPLANATION} Recent Context:
• With a view to increasing speed of trains in Indian Railways, ‘Mission Raftaar’
was announced in the Railway Budget 2016-17. It was approved by the NITI Aayog
in 2017.

What is the mission all about?


• The mission envisages a target of doubling of average speed of freight trains and
increasing the average speed of all non-suburban passenger trains by 25
kmph in the next 5 years
• Principal routes have been identified for raising of speed under Mission Raftaar.
These are six routes on Golden Quadrilateral and diagonals namely, Delhi –
Mumbai, Delhi – Howrah, Howrah- Chennai, Chennai – Mumbai, Delhi – Chennai
and Howrah – Mumbai.
• These six routes carry 58% of freight traffic and 52% of coaching traffic with a
share of only 16% of the network.
http://pib.nic.in/PressReleseDetail.aspx?PRID=1515655

{QUESTION NUMBER} 20
{TAGS} Government Schemes and programmes
Consider the following statements in respect of the ‘Khelo India Programme’:

Have a querry?
Drop us a mail at – hello@civilstap.com or call us at 8146207241
39
Revise your past 1.5 years of current affairs with our most comprehensive Test Series on Current Affairs

1. It has been introduced to revive sports culture in India at the grass-root level and
to gorge tie-ups with various international sports organizations
2. The revamped Khelo India Programme aims to achieve the twin objectives of
mass participation and Promotion of excellence in sports
3. The Khelo India logo has been designed by Ogilvy India
Which of the above statement(s) is/are correct?
{OPTION 1} 3 only
{OPTION 2} 1 and 2 only
{OPTION 3} 2 and 3 only
{OPTION 4} 1, 2 and 3
{RIGHT ANSWER} 3
{EXPLANATION} Recent context:
The vibrant Khelo India logo, reflecting the freshness, agility and vitality of New
India was launched at a simple ceremony by the Union Minister of State (I/C)
Youth Affairs and Sports Col Rajyavardhan Rathore at the Jawaharlal Nehru
Stadium in January 2018.
About the Khelo India Programme:
• The Khelo India programme has been introduced to revive the sports culture in
India at the grass-root level by building a strong framework for all sports played
in our country and establishing India as a great sporting nation.
• This programme is expected to help scout young talent from the schools in various
disciplines and groom them as future sports champions.
• Talented players identified in priority sports disciplines at various levels by a High-
Powered Committee will be provided annual financial assistance of Rs. 5 lakh per
annum for 8 years.

Revamped Khelo India Programme:


• The revamped Khelo India Scheme was launched during the financial year 2017-
18 with a view to achieving the twin objectives of mass participation and
promotion of excellence in sports.
• The vision of the revamped Khelo India is to infuse sports culture and achieve
sporting excellence in the country.
• The mission as stated in the revamped scheme is to encourage sports for all thus
allowing the population across gender and all age groups to harness the power
of sports.

What are the facilities/opportunities provided?

Have a querry?
Drop us a mail at – hello@civilstap.com or call us at 8146207241
40
Revise your past 1.5 years of current affairs with our most comprehensive Test Series on Current Affairs

• The identified sports talents will be given the option to join SAI National Sports
Academies, State Sports Academies or Sports Academies established by private
sector.
• Grants-in-aid will be provided for establishment, operation and maintenance of
sports academies in respect of identified disciplines to Sports Authority of India,
State Governments or to private sector or sports person under Public Private
Partnership (PPP) mode for facilitating and supplementing Long Term Athlete
Development (LTAD) programme (for 8 years).
• A system would be developed for rating of academics to facilitate selection of
appropriate academics for support.
• At least one academy for Para Athletes will be supported.

Khelo India Logo:

• The three-stroke Khelo India logo, designed by Ogilvy India, has in-built
modularity, allowing plenty of adaptability and flexibility into endless pictorial
forms.
• The colour of the Indian flag evokes a sense of National pride and team-work.
• The logo also conveys an impression of fitness and competitiveness.
http://pib.nic.in/PressReleseDetail.aspx?PRID=1515776

{QUESTION NUMBER} 21
{TAGS} Science and Tech
Consider the following statements about the Indian Regional Navigation Satellite
System (IRNSS):
1. It has three satellites in geosynchronous orbit and four satellites in geostationary
orbit.
2. It can service regions extending up to 1500 km around India's boundary.
3. It has a total of nine satellites of which six are currently in orbit.
Which of the above statement(s) is/are correct?

Have a querry?
Drop us a mail at – hello@civilstap.com or call us at 8146207241
41
Revise your past 1.5 years of current affairs with our most comprehensive Test Series on Current Affairs

{OPTION 1} 1 and 2 only


{OPTION 2} 1 and 3 only
{OPTION 3} 2 only
{OPTION 4} 1, 2 and 3
{RIGHT ANSWER} 3
{EXPLANATION}
Why in news?
IRNSS 1I satellite was launched by ISRO on April 12, 2018.

What is IRNSS?
• IRNSS stands for Indian Regional Navigation Satellite System.
• It is a set of satellites which together can provide India a regional positioning
system similar to the GPS.
• According to the ISRO website, the system is designed to give position accuracy
better than 20 metres to users in its primary coverage area.
• It can also service regions extending up to 1500 km around India's boundary.
• ISRO has built a total of nine satellites in the IRNSS series of which eight are
currently in orbit.

How many IRNSS satellites are already in orbit?


• There are currently seven IRNSS satellites (1A to 1G) in orbit. IRNSS 1I will be the
eighth in the series.
• A, B, F, G, I are placed in a geosynchronous orbit, which means they seem to be
at a fixed location above the Earth and they orbit along with the Earth.
• C, D, E, are located in geostationary orbit-they seem to be at a fixed location
above the Earth along the equator and orbit along with the Earth.

Specifications of IRNSS-1I:
• The IRNSS-1I weighs 1425 kg at lift-off and is the eighth satellite in the IRNSS
satellite constellation.
• It will be placed in a sub-geosynchronous transfer orbit and at its closest point will
be 284 km above the Earth and at its farthest will be 20,650 km above the Earth.
• Like all other IRNSS satellites, IRNSS-1I will also carry two payloads – navigation
payload and ranging payload - the former to transmit signals for determining
position, velocity and time and the latter for determining the frequency range of
the satellite.

What are the applications of IRNSS?

Have a querry?
Drop us a mail at – hello@civilstap.com or call us at 8146207241
42
Revise your past 1.5 years of current affairs with our most comprehensive Test Series on Current Affairs

• These satellites help not just in land navigation but also in marine and aerial
navigation.
• The data from these satellites can be used to give vehicle drivers visual and voice
navigation assistance.
• They also help in disaster management and in proper time-keeping.

https://www.thehindu.com/sci-tech/science/navigation-satellite-irnss-1i-to-be-
launched-on-april-12/article23494496.ece

{QUESTION NUMBER} 22
{TAGS} Science and Tech
Consider the following statements:
1. Neutrinos are tiny elementary particles that are part of atoms.
2. The India-based Neutrino Observatory is proposed to be set up in Theni district of
Tamil Nadu.
Which of the above statement(s) is/are correct?
{OPTION 1} 1 only
{OPTION 2} 2 only
{OPTION 3} Both 1 and 2
{OPTION 4} Neither 1 nor 2
{RIGHT ANSWER} 2
{EXPLANATION}
Why in news?
In Theni district in Tamil Nadu, the India-based Neutrino Observatory is proposed to
be set up.

What are neutrinos?


• Proton, neutron, and electron are tiny particles that make up atoms. The neutrino
is also a tiny elementary particle, but it is not part of the atom. Such particles
are also found to exist in nature.
• Neutrino has a very tiny mass, no charge and spins half. It interacts very weakly
with other matter particles. So weakly that every second trillions of neutrinos fall
on us and pass through our bodies unnoticed.
• Neutrinos come from the sun (solar neutrinos) and other stars, cosmic rays that
come from beyond the solar system, and from the Big Bang from which our
Universe originated. They can also be produced in the lab.
• Neutrinos come in three types or “flavours” – electron neutrino, tau neutrino
and muon neutrino. They can change from one flavor to another as they travel.

Have a querry?
Drop us a mail at – hello@civilstap.com or call us at 8146207241
43
Revise your past 1.5 years of current affairs with our most comprehensive Test Series on Current Affairs

This process is called neutrino oscillation and is an unusual quantum


phenomenon.

How are atmospheric neutrinos produced in nature?


Atmospheric neutrinos are produced from cosmic rays which consist of protons and
heavy nuclei. These collide with atmospheric molecules such as Nitrogen to
give off pions and muons which further decay to produce neutrinos.

Future applications of neutrino science:


• Properties of the sun
The visible light that reaches us from the sun is emitted from the surface of the
sun. The neutrinos which also take close to this time to reach us from the sun,
known as solar neutrinos, were produced in the core of the sun. Therefore,
they give us information about the interior of the sun. Studying these neutrinos
can help us understand what goes on in the interior of the sun.
• What makes up the universe?
Light coming from distant stars can be studied by astronomers, for example, to
detect new planets. Light is the visible part of the electromagnetic spectrum;
other parts are used in for example radio astronomy. Likewise, if the properties
of neutrinos are understood better, they can be used in astronomy to discover
what the universe is made up of.
• Probing Early Universe
Neutrinos interact very little with the matter around them, so they travel long
distances uninterrupted. Since they take time to cross these distances, they are
in effect uninterrupted for very long times. The extragalactic neutrinos we
observe may be coming from the distant past. These inviolate messengers can
give us a clue about the origin of the universe and the early stages of the
infant universe, soon after the Big Bang.
• Medical Imaging
Apart from direct future uses of neutrinos, there are technological applications
of the detectors that will be used to study them. For instance, X-ray machines,
PET scans, MRI scans, etc., all came out of research into particle detectors.
Hence the INO detectors may have applications in medical imaging.
https://www.thehindu.com/sci-tech/science/what-are-neutrinos-and-how-are-they-
detected/article23546887.ece

{QUESTION NUMBER} 23
{TAGS} Places, regions and communities in news, Defence in news

Have a querry?
Drop us a mail at – hello@civilstap.com or call us at 8146207241
44
Revise your past 1.5 years of current affairs with our most comprehensive Test Series on Current Affairs

‘Walong tri-junction’ was in the news recently. It is a junction between which of the
following countries?
{OPTION 1} India, China, Pakistan
{OPTION 2} India, Bhutan, China
{OPTION 3} India, China, Myanmar
{OPTION 4} China, Russia, Mongolia
{RIGHT ANSWER} 3
{EXPLANATION}
Why in news?
Indian troops deployed along the disputed Sino-India border in the Himalayan range
of the Arunachal sector have increased their patrolling at Walong, a tri-junction
of India, China and Myanmar to prevent a repeat of a Doklam-like standoff.

About Walong Tri-Junction:


• The tri-junction is located around 50 kms from Walong, India’s easternmost
town in Arunachal Pradesh, near the Tibet region.
• Walong is a small cantonment and administrative town in the Anjaw District of
the state of Arunachal Pradesh in northeastern India. Anjaw was carved out of
Lohit District in 2004.
• Walong's approximate position is 28 degrees 06 minutes North, 97 degrees East.
It lies on the west bank of the Lohit River (a tributary of the Brahmaputra),
approximately 20 kms south of the Chinese border. Just north of the border lies
the Tibetan trading town of Rima.

https://www.thehindu.com/news/national/india-steps-up-vigil-at-walong-tri-
junction/article23408702.ece
https://www.livemint.com/Politics/pk7ZEWPLKnYoVCO5o8SWpN/Army-
increases-strength-at-IndiaChinaMyanmar-trijunction.html

Have a querry?
Drop us a mail at – hello@civilstap.com or call us at 8146207241
45
Revise your past 1.5 years of current affairs with our most comprehensive Test Series on Current Affairs

{QUESTION NUMBER} 24
{TAGS} Science and Tech
Consider the following statements:
1. Transiting Exoplanet Survey Satellite (TESS) is a satellite launched by the European
Space Agency to catalog exoplanets.
2. Exoplanets are planets that remain undiscovered in our solar system.

Which of the above statement(s) is/are correct?


{OPTION 1} Only 1
{OPTION 2} Only 2
{OPTION 3} Both 1 and 2
{OPTION 4} Neither 1 nor 2
{RIGHT ANSWER} 4
{EXPLANATION}
Why in news?
On April 18, 2018 NASA launched TESS (Transiting Exoplanet Survey Satellite),
which will monitor more than 200,000 stars.

What are exoplanets?


• All of the planets in our solar system orbit around the Sun. Planets that orbit
around other stars are called exoplanets.
• Exoplanets are very hard to see directly with telescopes. They are hidden by the
bright glare of the stars they orbit.

How can exoplanets be discovered?


One way to search for exoplanets is to look for "wobbly" stars. A star that has
planets doesn’t orbit perfectly around its center. From far away, this off-center
orbit makes the star look like its wobbling.

What is the current exoplanet count?


As of now, the number of known exoplanets stands at over 3,700. Another 4,500
objects are strong contenders to become exoplanets. About 50 are believed to
potentially habitable. They have the right size and the right orbit to support
surface water and, at least theoretically, to support life.

What will TESS do?


The mission will catalog thousands of planet candidates and vastly increase the
current number of known exoplanets. Of these, approximately 300 are
expected to be Earth-sized and super-Earth-sized exoplanets, which are worlds
no larger than twice the size of Earth.

Have a querry?
Drop us a mail at – hello@civilstap.com or call us at 8146207241
46
Revise your past 1.5 years of current affairs with our most comprehensive Test Series on Current Affairs

How does TESS plan to do this?


TESS will look for dips in the visible light of stars, which requires that planets
cross stars in our line of sight. Repetitive, periodic dips can reveal a planet or
planets orbiting a star.

How long is this mission?


TESS will survey the entire sky over a period of two years. The satellite will do this
by breaking it up into 26 different sectors and the cameras on the satellite will
survey each sector for 27 days.
https://www.thehindu.com/sci-tech/science/the-hindu-explains-nasas-transiting-
exoplanet-survey-satellite/article23583276.ece

{QUESTION NUMBER} 25
{TAGS} Science and Tech, Defence in news
Consider the following statements about the Brahmos Missile:
1. It has been indigenously developed by India.
2. It is the fastest cruise missile in the world.
3. It travels at a speed faster than the speed of sound.
Which of the above is/are correct?
{OPTION 1} 1 and 2 only
{OPTION 2} 2 and 3 only
{OPTION 3} 1 and 3 only
{OPTION 4} All of the above
{RIGHT ANSWER} 2
{EXPLANATION}
Why in news?
Brahmos Aerospace CEO and MD Sudhir Mishra said that the Bhrahmos cruise
missile will breach the mach 7 barrier to become a hypersonic missile system in
the next decade.

About Brahmos:
• Brahmos is a two-stage missile with a solid propeller booster engine as its first
stage which brings it to supersonic speed and then gets separated.
• Stealth technology and guidance system with advanced embedded software
provides the missile with special features.
• It is the fastest cruise missile in the world co-developed by India and Russia.

Features:
• The missile has flight range of up to 290-km with supersonic speed all through
the flight, leading to shorter flight time, consequently ensuring lower dispersion

Have a querry?
Drop us a mail at – hello@civilstap.com or call us at 8146207241
47
Revise your past 1.5 years of current affairs with our most comprehensive Test Series on Current Affairs

of targets, quicker engagement time and non-interception by any known weapon


system in the world.
• It operates on ‘Fire and Forget Principle’, adopting varieties of flights on its way
to the target. Its destructive power is enhanced due to large kinetic energy on
impact.
• Its cruising altitude could be up to 15 km and terminal altitude is as low as 10
meters.
• It carries a conventional warhead weighing 200-300 kgs.
• The missile currently travels at mach 2.8 or 2.8 times the speed of sound and will
touch mach 5 in three years.

https://www.thehindu.com/news/national/brahmos-will-breach-mach-7-barrier-in-
next-decade/article23717862.ece

{QUESTION NUMBER} 26
{TAGS} Science and Tech
Consider the following statements about Pink Moon:
1. The full moon in April is called Pink Moon.
2. It is called so because it is Pink in colour.
Which of the above is/are correct?
{OPTION 1} 1 only
{OPTION 2} 2 only
{OPTION 3} Both 1 and 2
{OPTION 4} Neither 1 nor 2
{RIGHT ANSWER} 1
{EXPLANATION} Every year, the full moon in April is called the ‘pink moon’,
but it is only pink in name. It is to represent ground phlox. Phlox are ground-
cover flowering plants and usually bloom around springtime. These plants are
found mostly in North America. The April full moon is also known as the
Sprouting Grass moon, the Egg Moon and the Fish Moon, all Native American
names for the phenomenon.
https://www.thehindu.com/sci-tech/science/the-hindu-explains-what-is-a-pink-
moon/article23730955.ece

{QUESTION NUMBER} 27
{TAGS} Indian institutions and bodies in news
Consider the following statements about North East Centre for Technology
Application and Reach (NECTAR):
1. It is an autonomous society under the Ministry of Development of North Eastern
Region.

Have a querry?
Drop us a mail at – hello@civilstap.com or call us at 8146207241
48
Revise your past 1.5 years of current affairs with our most comprehensive Test Series on Current Affairs

2. It has a vision to expand the benefits of technology for inclusive economic


development of the North East.
Which of the above is/are correct?
{OPTION 1} 1 only
{OPTION 2} 2 only
{OPTION 3} Both 1 and 2
{OPTION 4} Neither 1 nor 2
{RIGHT ANSWER} 2
{EXPLANATION}
Why in news?
Despite ₹1,290 crore in funds, the research body (NECTAR) remains distant and
redundant for the Northeast.

What is NECTAR:
NECTAR is an autonomous society, set up under Department of Science &
Technology, Government of India with its headquarter at Shillong,
Meghalaya.

Purpose:
In order to assist the northeastern region, NECTAR will be ensuring applications
of appropriate technologies for development in the areas of biodiversity
concerns, watershed management, telemedicine, horticulture, infrastructure
planning & development, planning and monitoring, and tele-schooling using
cutting-edge solutions, employment generation etc. through utilization of local
products/resources and associated skill development.

VISION:
To be the leading Centre to foster, nurture and ensure the delivery, sustenance
and use of technology applications for public and social good; and to reach
and expand the benefits of technology among people, communities,
institutions and governments for equitable and inclusive social and economic
development of the North Eastern Region of our country.

MISSION:
NECTAR is conceived as a collaborative centre of excellence to resolve the last mile
problem in the delivery, induction, management, use and extension of
technology applications which serve public good and promote social and
economic development of the North Eastern region in the broadest possible
terms.

https://www.thehindu.com/news/national/other-states/when-nectar-turned-poison-
for-bamboo/article23796544.ece

Have a querry?
Drop us a mail at – hello@civilstap.com or call us at 8146207241
49
Revise your past 1.5 years of current affairs with our most comprehensive Test Series on Current Affairs

{QUESTION NUMBER} 28
{TAGS} Government Rules and Regulations
Consider the following statements about the Fugitive Economic Offenders Act, 2018:
1. It covers economic offences of at least Rs. 50 crores.
2. Counterfeiting currency is one of the offences listed in the Act.
Which of the above is/are correct?
{OPTION 1} 1 only
{OPTION 2} 2 only
{OPTION 3} Both 1 and 2
{OPTION 4} Neither 1 nor 2
{RIGHT ANSWER} 2
{EXPLANATION}
Why in news?
The Fugitive Economic Offenders Act, 2018 came into force on 21st April, 2018.

What is it?
The Fugitive Economic Offenders Act, 2018 is an Act of the Parliament of India that
seeks to confiscate properties and assets of economic offenders that evade
prosecution by remaining outside the jurisdiction of Indian courts. Economic
offences with a value of more than Rs. 100 crores, which are listed in the
schedule of the Fugitive Economic Offenders Act, come under the purview of this
law.

Who is a fugitive economic offender?


A fugitive economic offender has been defined as a person against whom an arrest
warrant has been issued for committing an offence listed in the schedule, and the
value of the offence is at least Rs. 100 crore. Further, the person has:
(i) left the country to avoid facing prosecution, or
(ii) refuses to return to face prosecution.

Which are the offences listed in the Act?


Some of the offences listed in the Act are:
(i) counterfeiting government stamps or currency,
(ii) cheque dishonour,
(iii) money laundering, and
(iv) transactions defrauding creditors.

Salient Features:
• A person may be declared an FEO if he leaves the country and refuses to return
to face prosecution. He may be asked to appear at a specified place at least six
weeks after notice.

Have a querry?
Drop us a mail at – hello@civilstap.com or call us at 8146207241
50
Revise your past 1.5 years of current affairs with our most comprehensive Test Series on Current Affairs

• A person’s property may be attached for 180 days. Attached properties may
include those believed to be proceeds of crime and benami properties.
• An FEO’s property may be confiscated and vested in the central government, free
of encumbrances. The central government may dispose this property only after
90 days. The Special Court may exempt certain properties from confiscation
where any other person has a genuine interest.
• Authorities may search premises and persons on the belief that a person may be
an FEO, or has proceeds of crime, among others. They may also seize documents.

https://www.thehindu.com/news/national/president-nod-for-fugitive-economic-
offenders-bill/article24608225.ece

{QUESTION NUMBER} 29
{TAGS} Science and Tech
Consider the following statements about Pectinophora gossypiella:
1. It is commonly known as Pink bollworm.
2. It is a pest that attacks cotton crops.
Which of the above statement(s) is/are correct?
{OPTION 1} 1 only
{OPTION 2} 2 only
{OPTION 3} Both 1 and 2
{OPTION 4} Neither 1 nor 2
{RIGHT ANSWER} 3
{EXPLANATION}
Why in news?
Pink Bollworm attacks are a perpetual threat to the cotton farmers in India.

What is Pectinophora gossypiella?


Pectinophora gossypiella is the scientific name for Pink bollworm. It is an insect
known for being a pest in cotton farming. The adult is a small, thin, gray moth
with fringed wings. The larva is a dull white caterpillar with eight pairs of legs
with conspicuous pink banding along its dorsum. The larva reaches one half inch
in length.

Where is it found?
The pink bollworm is native to Asia, but has become an invasive species in most of
the world's cotton-growing regions. It reached the cotton belt in the southern
United States by the 1920s. It is a major pest in the cotton fields of the southern
California deserts.
How can it be controlled?

Have a querry?
Drop us a mail at – hello@civilstap.com or call us at 8146207241
51
Revise your past 1.5 years of current affairs with our most comprehensive Test Series on Current Affairs

• Allowing cattle grazing of the left over green bolls on the plant at the end of crop
season. Timely crop termination to maintain closed season.
• Clean up/ destruction of cotton stubbles immediate to harvest.
• Avoiding stacking of cotton stalks for fuel purpose over long periods.
• Deep ploughing to expose the pupae of the surviving larvae.

https://www.thehindubusinessline.com/specials/india-file/cotton-a-seedy-
story/article23953027.ece

{QUESTION NUMBER} 30
{TAGS} Government Schemes and programmes
With reference to the "Student Rural Entrepreneurship Awareness Development
Yojana (READY), consider the following statements:
1. It is an initiative of Indian Council of Agricultural Research
2. It has been started for the rural youth employed in agriculture sector
Which of the statements given above is/are correct?
{OPTION 1} 1 only
{OPTION 2} 2 only
{OPTION 3} Both 1 and 2
{OPTION 4} Neither 1 nor 2
{RIGHT ANSWER} 1
{EXPLANATION}
Recent context:
▪ Union Minister of Agriculture and Farmers’ Welfare Shri Radha Mohan Singh
addresses 90th Foundation Day ceremony of Indian Council of Agricultural
Research
▪ The Agriculture Minister said the skill development of students in the Student
READY (Rural Entrepreneurship Awareness Development Yojana) programme has
now been included for one complete year to attract youth to agriculture
education.

Student READY Programme:


▪ Student READY (Rural Entrepreneurship Awareness Development Yojana)
programme is an initiative of Indian Council of Agricultural Research.
▪ It aims to ensure employment to graduates of Agriculture and allied subjects and
to develop entrepreneurs for emerging knowledge-intensive agriculture.
▪ This envisages the introduction of the programme in all the Agricultural
Universities as an essential prerequisite for the award of degree to ensure hands
on experience and practical training depending on the requirements of
respective discipline and local demands.
▪ This programme includes five components

Have a querry?
Drop us a mail at – hello@civilstap.com or call us at 8146207241
52
Revise your past 1.5 years of current affairs with our most comprehensive Test Series on Current Affairs

1. Experiential Learning
2. Rural Awareness Works Experience
3. In-Plant Training / Industrial attachment
4. Hands-on training (HOT) / Skill development training
5. Students Projects
http://pib.nic.in/PressReleseDetail.aspx?PRID=1538731

{QUESTION NUMBER} 31
{TAGS} Indian institutions and bodies in news
With reference to the "Central Adoption Resource Authority (CARA), consider the
following statements:
1. It is designated as the Central Authority to deal with inter-country adoptions
2. It is a statutory body
3. It has allowed individuals in a live-in relationship to adopt children within India
Which of the statement(s) given above is/are correct?
{OPTION 1} 1 only
{OPTION 2} 2 and 3 only
{OPTION 3} 1 and 3 only
{OPTION 4} 1, 2 and 3
{RIGHT ANSWER} 4
{EXPLANATION}
Recent Context:
Ministry of Women and Child Development directs state governments to ensure
registration of all child care institutions and linking to CARA

About CARA:
▪ Central Adoption Resource Authority (CARA) is an autonomous and statutory
body of Ministry of Women and Child Development in the Government of India.
▪ It was set up in 1990.
▪ It functions as the nodal body for the adoption of Indian children and is
mandated to monitor and regulate in-country and inter-country adoptions.
CARA is designated as the Central Authority to deal with inter-country adoptions
in accordance with the provisions of the Hague Convention on Inter-country
Adoption, 1993, ratified by Government of India in 2003.
▪ CARA primarily deals with the adoption of orphaned, abandoned and surrendered
children through its associated and recognized adoption agencies.
▪ In 2018, CARA has allowed individuals in a live-in relationship to adopt children
from and within India.
http://pib.nic.in/PressReleseDetail.aspx?PRID=1538747

{QUESTION NUMBER} 32
{TAGS} Science and Tech

Have a querry?
Drop us a mail at – hello@civilstap.com or call us at 8146207241
53
Revise your past 1.5 years of current affairs with our most comprehensive Test Series on Current Affairs

With reference to the "India-based Neutrino Observatory", consider the following


statements:
1. The Tata Institute of Fundamental Research is the nodal institution for this project
2. ₹1500 crore has been allocated for this project
Which of the statements given above is/are correct?
{OPTION 1} 1 only
{OPTION 2} 2 only
{OPTION 3} Both 1 and 2
{OPTION 4} Neither 1 nor 2
{RIGHT ANSWER} 3
{EXPLANATION} Recent context:
Centre has given its nod for building country’s first India-based Neutrino Observatory
(INO) at Bodi West Hills in Theni district of Tamil Nadu.

Where is the India neutrino observatory being planned?


At Pottipuram village, in Theni district, near the Tamil Nadu-Kerala border.

What does it entail?


▪ It is an underground project and will comprise a complex of caverns.
▪ The main cavern, which will house the huge neutrino detector [50-kilo tonne
magnetised iron calorimeter], will be 130 m long, 26 m wide, and 30 m high.
▪ Two smaller caverns will be used for setting up experiments for neutrino double
detector and dark matter. Approach to this complex will be by a 2-km-long tunnel.
What is a neutrino?
Neutrinos are the smallest particles that form the universe.

Who is in-charge of the project?


The Tata Institute of Fundamental Research is the nodal institution. The observatory
is to be built jointly with the Department of Atomic Energy and the Department
of Science and Technology.

Why are the locals opposing it?


Locals fear that the excavation and blasts needed to bore the tunnel in the mountains
will endanger the biodiversity of the Western Ghats. Some of the concerns
voiced range from radiation, structural damage to the mountain to emission of
hazardous chemicals.

What do scientists say?


Scientists have junked all these claims as baseless and unfounded.
https://www.thehindubusinessline.com/news/science/all-you-want-to-know-about-
the-neutrino-controversy/article23554745.ece
http://www.pib.nic.in/Pressreleaseshare.aspx?PRID=1538983

Have a querry?
Drop us a mail at – hello@civilstap.com or call us at 8146207241
54
Revise your past 1.5 years of current affairs with our most comprehensive Test Series on Current Affairs

{QUESTION NUMBER} 33
{TAGS} International News
"Regional Aviation Partnership" sometimes mentioned in the news, is related to
which of the following organization?
{OPTION 1} BRICS
{OPTION 2} ASEAN
{OPTION 3} G7
{OPTION 4} BIMSTEC
{RIGHT ANSWER} 1
{EXPLANATION}
Recent Context:
Cabinet has approved MOU Amongst BRICS (Brazil, Russia, India, China and
South Africa) Nations on Regional Aviation Partnership.

▪ The objective of MoU is to benefit BRICS countries from establishing of


institutional framework to cooperate in field of civil aviation.
▪ The MoU signifies important landmark in civil aviation relations between India
and other BRICS member states.
▪ It has potential to spur greater trade, investment, tourism and cultural
exchanges amongst BRICS nations.
Among the areas of cooperation, following areas have been identified:
1. Public Policies and best practices in regional services;
2. Regional Airports;
3. Airport infrastructure management and air navigation services
4. Technical cooperation between regulatory agencies
5. Innovation
6. Environment Sustainability; including deliberation of global initiatives
7. Qualification and Training
8. Other fields as mutually determined
http://pib.nic.in/PressReleseDetail.aspx?PRID=1539070
https://www.business-standard.com/article/news-cm/cabinet-approves-mou-
amongst-brics-nations-on-regional-aviation-partnership-118071801058_1.html

{QUESTION NUMBER} 34
{TAGS} Science and Tech, Space in news
With reference to the "Chandrayaan-2", consider the following statements:
1. It is only a lunar orbiter, developed by India
2. It is India's second lunar exploration mission
3. It will be launched to the moon by GSLV Mark III
Which of the statement(s) given above is/are correct?
{OPTION 1} 1 and 2 only
{OPTION 2} 2 and 3 only

Have a querry?
Drop us a mail at – hello@civilstap.com or call us at 8146207241
55
Revise your past 1.5 years of current affairs with our most comprehensive Test Series on Current Affairs

{OPTION 3} 3 only
{OPTION 4} 1, 2 and 3
{RIGHT ANSWER} 2
{EXPLANATION} Recent Context:
Indian Space Research Organisation (ISRO) is planning to deploy a rover on the
lunar surface through Chandrayaan-2 mission.

About Chandrayaan-2:
▪ Chandrayaan-2 is India's second lunar exploration mission after Chandrayaan-1.
▪ It has been developed by the Indian Space Research Organisation (ISRO).
▪ The mission is planned to be launched to the Moon by a Geosynchronous
Satellite Launch Vehicle Mark III (GSLV Mk III).
▪ It includes a lunar orbiter, lander and rover, all developed by India.
▪ Chandrayaan-2 is scheduled to launch in April 2019.
http://pib.nic.in/PressReleseDetail.aspx?PRID=1538997
https://en.wikipedia.org/wiki/Chandrayaan-2

Have a querry?
Drop us a mail at – hello@civilstap.com or call us at 8146207241
56
Revise your past 1.5 years of current affairs with our most comprehensive Test Series on Current Affairs

{QUESTION NUMBER} 35
{TAGS} Terms in news, Science and Tech
The term "Generic Drug" is sometimes seen in the news. Which of the following
statement best describes this term?
{OPTION 1} These are the drugs, which are not approved by the CDSCO
{OPTION 2} These are inferior drug to brand-name drug as they are being
manufactured in poor-quality facilities
{OPTION 3} These are the cheaper and more effective alternative to brand-name
drugs
{OPTION 4} These are the copies of the brand-name drugs that have exactly the same
use and effects
{RIGHT ANSWER} 4
{EXPLANATION} Recent Context:
▪ At a summit organized by the Organization of Pharmaceutical Producers of India
(OPPI), it was recognized that India is one of the key players in the field of
generic medicines.
▪ By 2020, India is likely to be among the top three pharmaceutical markets by
incremental growth and 6th largest market globally in absolute size.
➢ Generic Drugs:
▪ Generic drugs are copies of brand-name drugs that have exactly the same
dosage, intended use, effects, side effects, route of administration, risks, safety,
and strength as the original drug. In other words, their pharmacological effects
are exactly the same as those of their brand-name counterparts.
▪ In India, CDSCO approves all type of drugs including the Generic Drugs.
http://pib.nic.in/PressReleseDetail.aspx?PRID=1538588

{QUESTION NUMBER} 36
{TAGS} Indian institutions and bodies in news
With reference to the "Indian Council of World Affairs (ICWA)", consider the
following statements:
1. It is an institution of national importance
2. Vice-President of India is the ex-officio President of ICWA
Which of the statement/s given above is/are correct?
{OPTION 1} 1 only
{OPTION 2} 2 only
{OPTION 3} Both 1 and 2
{OPTION 4} Neither 1 nor 2
{RIGHT ANSWER} 3
{EXPLANATION}
Recent Context:
President and Vice President of India has appointed Dr. T.C.A Raghavan as the
Director General of ICWA.

Have a querry?
Drop us a mail at – hello@civilstap.com or call us at 8146207241
57
Revise your past 1.5 years of current affairs with our most comprehensive Test Series on Current Affairs

About ICWA:
▪ The Indian Council of World Affairs (ICWA) is a New Delhi based Indian think-
tank.
▪ Established in 1943, it is devoted exclusively for the study of international
relations and foreign affairs.
▪ By an Act of Parliament, it has been declared an institution of national
importance in 2001.
▪ The Vice President of India is the ex-officio President of ICWA, while the Minister
of External Affairs is its Vice-President.
▪ It is housed in Sapru House, the name being derived from Sir Tej Bahadur Sapru,
who was also the founder President of the Council.
▪ ICWA is a member of the United Nations Academic Impact.
http://pib.nic.in/newsite/PrintRelease.aspx?relid=180545

{QUESTION NUMBER} 37
{TAGS} Government Schemes and programmes
With reference to the "Swachh Survekshan Grameen, 2018", consider the following
statements:
1. It has been launched in only 100 districts of the country
2. Citizen's feedback is one of the parameters for ranking
Which of the statement/s given above is/are correct?
{OPTION 1} 1 only
{OPTION 2} 2 only
{OPTION 3} Both 1 and 2
{OPTION 4} Neither 1 nor 2
{RIGHT ANSWER} 2
{EXPLANATION} Recent Context:
Ministry of Drinking Water and Sanitation launched the Swachh Survekshan
Grameen 2018.

Swachh Survekshan Grameen, 2018


▪ It is a nationwide survey of rural India to rank the cleanest and dirtiest States
and districts based on qualitative and quantitative evaluation.
▪ A random selection of 6980 villages across 698 districts will be surveyed for the
ranking.
▪ Total 34,000 public places namely schools, anganwadis, public health centres,
haat/bazaars/religious places in these villages will be visited for the survey.
▪ The rankings will be based on three basic parameters:
1. Direct observation of public places by independent surveyors (30% weightage)
2. Service-level progress using data from the Swachh Bharat Mission’s information
system (35%)
3. Citizens’ feedback (35%)

Have a querry?
Drop us a mail at – hello@civilstap.com or call us at 8146207241
58
Revise your past 1.5 years of current affairs with our most comprehensive Test Series on Current Affairs

▪ The top-performing states and districts were awarded on 2nd October 2018.
https://www.thehindu.com/news/national/survey-launched-to-rank-states-on-rural-
cleanliness/article24414126.ece

{QUESTION NUMBER} 38
{TAGS} Biodiversity in news
Consider the following statements:
1. Golden jackal is listed as "Endangered" on the IUCN Red List
2. IUCN is an organ of the United Nations
Which of the statement(s) given above is/are correct?
{OPTION 1} 1 only
{OPTION 2} 2 only
{OPTION 3} Both 1 and 2
{OPTION 4} Neither 1 nor 2
{RIGHT ANSWER} 4
{EXPLANATION} Recent Context:
Destruction of mangrove cover in the Bandar Reserve Forest is forcing the golden
jackal out of its habitat, triggering a conflict with the local communities.
➢ Golden Jackal
▪ The golden jackal (Canis aureus) is a wolf-like canid that is native to Southeast
Europe, Southwest Asia, South Asia, and regions of Southeast Asia.
▪ Golden jackals are abundant in valleys and beside rivers and their tributaries,
canals, lakes, and seashores. They are rare in foothills and low mountains.
▪ It is listed as Least Concern on the IUCN Red List due to its widespread
distribution and high density in areas with plenty of available food and optimum
shelter.
➢ IUCN
▪ The International Union for Conservation of Nature (IUCN) is an international
organization working in the field of nature conservation and sustainable use of
natural resources.
▪ It is involved in data gathering and analysis, research, field projects, advocacy, and
education.
▪ IUCN's mission is to "influence, encourage and assist societies throughout the
world to conserve nature and to ensure that any use of natural resources is
equitable and ecologically sustainable".
▪ IUCN has a membership of over 1400 governmental and non-governmental
organizations.
▪ The organization is best known to the wider public for compiling and publishing
the IUCN Red List of Threatened Species, which assesses the conservation status
of species worldwide.

Have a querry?
Drop us a mail at – hello@civilstap.com or call us at 8146207241
59
Revise your past 1.5 years of current affairs with our most comprehensive Test Series on Current Affairs

▪ IUCN was established in 1948. It was previously called the International Union for
the Protection of Nature (1948–1956) and the World Conservation Union (1990–
2008).
▪ IUCN has observer status at the United Nations.
https://www.thehindu.com/news/national/andhra-pradesh/golden-jackal-faces-threat-in-
its-habitat/article24428393.ece

{QUESTION NUMBER} 39
{TAGS} Important Bills, Acts and Amendments
With reference to the "Assam Witch Hunting (Prohibition, Prevention and Protection)
Act, consider the following statements:
1. It prescribes a prison term of up to seven years and up to ₹5 lakh in fine for calling
a person witch
2. Every offense under the Act has been made “cognizable, non-bailable and non-
compoundable
Which of the statement(s) given above is/are correct?
{OPTION 1} 1 only
{OPTION 2} 2 only
{OPTION 3} Both 1 and 2
{OPTION 4} Neither 1 nor 2
{RIGHT ANSWER} 3
{EXPLANATION} Recent Context:
President has given assent to the Assam Witch Hunting (Prohibition, Prevention and
Protection) Bill, 2015 making it an act.
Provisions of the Act:
▪ It has made every offense under the Act “cognizable, non-bailable and non-
compoundable.”
▪ The Act prescribes a prison term of up to seven years and up to ₹5 lakh in fine for
calling a person witch.
▪ It also has provisions to come with Section 302 of the IPC (punishment for
murder) if someone is killed after being branded a witch.
▪ The punishment for leading a person to suicide may be extended to life
imprisonment and up to ₹5 lakh in fine.
▪ Another important person behind the legislation is the Director-General of
Police Kuladhar Saikia. As Deputy Inspector-General in Kokrajhar, he launched
Project Prahari in 2001, which blended normal policing with social campaigns to
check the menace.
https://www.thehindu.com/news/national/other-states/kovind-clears-assam-bill-
against-witch-hunt/article24436379.ece

{QUESTION NUMBER} 40
{TAGS} Science and Tech

Have a querry?
Drop us a mail at – hello@civilstap.com or call us at 8146207241
60
Revise your past 1.5 years of current affairs with our most comprehensive Test Series on Current Affairs

With reference to the Genetically Modified (GM) Crops, consider the following
statements:
1. India ranks 2nd in global cultivation of GM crops
2. BT Cotton is the only GM crop approved for commercial cultivation in India
Which of the statement(s) given above is/are correct?
{OPTION 1} 1 only
{OPTION 2} 2 only
{OPTION 3} Both 1 and 2
{OPTION 4} Neither 1 nor 2
{RIGHT ANSWER} 2
{EXPLANATION}
Recent context:
▪ As per the report of International Service for the Acquisition of Agri-biotech
Applications, 2018, India ranks 5th in global cultivation of GM crops.

▪ Bt. cotton is the only GM crop approved for commercial cultivation in the
Country.
▪ The approval of any new genetically modified crop is given on a case-to-case
basis after thorough scientific evaluation of health and environment safety as per
applicable guidelines made under Environment (Protection) Act, 1986 and Rules,
1989.
▪ In India, the GM crops that are under regulatory consideration — apart from the
already commercialised Bt/insect-resistant cotton — include glyphosate-tolerant
cotton and biotech hybrid mustard.

Have a querry?
Drop us a mail at – hello@civilstap.com or call us at 8146207241
61
Revise your past 1.5 years of current affairs with our most comprehensive Test Series on Current Affairs

https://indianexpress.com/article/india/indias-genetically-modified-crop-area-fifth-
largest-in-world-5255662/

{QUESTION NUMBER} 41
{TAGS} Government Rules and Regulations, Science and Tech, Terms in news
Which one of the following best describes the term "Net Neutrality" sometimes seen
in news?
{OPTION 1} It is a method to ensure that all telecom companies are charging uniform
prices for internet access
{OPTION 2} It is a principle that Internet Service Providers should not discriminate
and treat all internet communications equally
{OPTION 3} It is an initiative by Indian government to make uniform Data Privacy
policies for all
{OPTION 4} It is a principle that there should not be discrimination on the placement
of websites on search engines
{RIGHT ANSWER} 2
{EXPLANATION} Recent Context:
▪ Telecom Commission has approved the Net Neutrality.
▪ Net Neutrality means that telecom and Internet service providers must treat all
data on Internet equally, and not discriminate or charge differently by user,
content, site, platform, or application. They cannot engage in practices such as
blocking, slowing down or granting preferential speeds to any content.
▪ Telecom Commission has said, “Internet access services should be governed by
a principle that restricts any form of discrimination or interference in the
treatment of content, including practices like blocking, degrading, slowing down
or granting preferential speeds or treatment to any content.”
▪ To implement Net neutrality, the telecom commission had recommended that
the terms of license agreements that govern the provision of Internet services
in India be amended “to incorporate the principles of non-discriminatory
treatment of content along with the appropriate exclusions and exceptions.”
▪ India’s decision to uphold Net neutrality assumes greater significance, given that
in the U.S., the rules on Net neutrality were repealed.
https://www.thehindu.com/sci-tech/technology/internet/telecom-commission-
approves-net-neutrality-new-telecom-policy/article24391077.ece

Have a querry?
Drop us a mail at – hello@civilstap.com or call us at 8146207241
62
Revise your past 1.5 years of current affairs with our most comprehensive Test Series on Current Affairs

{QUESTION NUMBER} 42
{TAGS} Rankings
With reference to the "Ease of doing business ranking of states, 2018", consider the
following statements:
1. It has been released by the Ministry of Corporate Affairs
2. It is based on the scores of states in Business Reform Action Plan
3. Andhra Pradesh has topped the list
Which of the statement/s given above is/are correct?
{OPTION 1} 1 and 2 only
{OPTION 2} 2 and 3 only
{OPTION 3} 1 only
{OPTION 4} 1,2 and 3
{RIGHT ANSWER} 2
{EXPLANATION} Recent Context:
▪ Andhra Pradesh has topped the third edition of the government’s ‘Ease of Doing
Business’ index for States with a final score of 98.42%.
▪ A.P. got 99.73% score in reform evidence and 86.50% in feedback score. While
Telangana stood second in the final score, Haryana bagged the third position.

▪ The Department of Industrial Policy and Promotion (DIPP), Ministry of


Commerce and Industry, in partnership with the World Bank Group, released
the Business Reform Action Plan (BRAP) for implementation by States/UTs.
▪ It includes 405 recommendations for reforms on regulatory processes, policies,
practices and procedures spread across 12 reform areas.
1. labour regulation enablers
2. contract enforcement
3. registering property
4. inspection reform enablers
5. single window system
6. land availability and allotment
7. construction permit enablers
8. environmental registration enablers
9. obtaining utility permits
10.paying taxes
11.access to information and transparency enablers
12.sector-specific reforms spanning the lifecycle of a typical business
▪ DIPP carry out a comprehensive business-to-government (B2G) feedback
exercise whereby the States and UTs will take feedback from businesses on the
quality of implementation of the reforms claimed.
▪ The feedback scores are used to generate a ranking of States/UTs in terms of
reform implementation.
http://pib.nic.in/newsite/PrintRelease.aspx?relid=180491

Have a querry?
Drop us a mail at – hello@civilstap.com or call us at 8146207241
63
Revise your past 1.5 years of current affairs with our most comprehensive Test Series on Current Affairs

https://www.thehindubusinessline.com/opinion/reading-ease-of-doing-business-
rankings/article24400572.ece

{QUESTION NUMBER} 43
{TAGS} Space in news, Science and Tech
Consider the following pairs:
Satellites sometimes mentioned in Objective
news
1. GRACE-FO Monitor changes in sea level rise
and ice melt
2. Monitor changes in sea level Measuring ice sheet elevation and
sea ice thickness
3. ICESat-2 Earth-Imaging

Which of the pair(s) given above is/are correctly matched?


{OPTION 1} 1 and 2 only
{OPTION 2} 2 and 3 only
{OPTION 3} 1 and 3 only
{OPTION 4} 1, 2 and 3
{RIGHT ANSWER} 4
{EXPLANATION} Recent context:
A twin spacecraft (GRACE-FO) to monitor the changes in sea level rise, ice melt
and drought on Earth was launched aboard a SpaceX rocket, along with five
communication satellites.
The Gravity Recovery and Climate Experiment Follow-On (GRACE-FO) is a joint
mission by NASA and the German Research Centre for Geosciences (GFZ).

ICESat-2 (Ice, Cloud, and land Elevation Satellite 2), part of NASA's Earth
Observing System, is a satellite mission for measuring ice sheet elevation and
sea ice thickness, as well as land topography, vegetation characteristics, and
clouds.

ASNARO-2 is a Japanese Synthetic Aperture Radar (SAR) Earth-imaging


satellite developed by NEC Corporation and USEF.
It was launched on January 2018.
https://www.thehindubusinessline.com/news/science/two-sportscar-sized-satellites-
in-orbit-to-measure-earths-water/article23966601.ece

Have a querry?
Drop us a mail at – hello@civilstap.com or call us at 8146207241
64
Revise your past 1.5 years of current affairs with our most comprehensive Test Series on Current Affairs

{QUESTION NUMBER} 44
{TAGS} Government Rules and Regulations
Section 497 of Indian Penal Code is related to:
{OPTION 1} Cyber Crime
{OPTION 2} Child Labour
{OPTION 3} Domestic Violence
{OPTION 4} Adultery
{RIGHT ANSWER} 4
{EXPLANATION} Recent context:
▪ Supreme Court has scrapped Section 497 of the Indian Penal code, which says
that adultery is a crime.
▪ Supreme Court has said that matrimonial sanctity is an issue but the penal
provision on adultery is apparently violative of the right to equality under the
constitution.
▪ Chief Justice has said that how married couples deal with adultery is "absolutely
a matter of privacy at its pinnacle".
Conclusion: Adultery is not a crime.

https://www.thehindu.com/news/national/adultery-not-a-criminal-offence-as-sc-
strikes-down-section-497-of-ipc/article25055245.ece

Have a querry?
Drop us a mail at – hello@civilstap.com or call us at 8146207241
65
Revise your past 1.5 years of current affairs with our most comprehensive Test Series on Current Affairs

{QUESTION NUMBER} 45
{TAGS} Science and Tech
"Nano-technology" has applications in which of the following?
1. Precise drug delivery
2. Faster recharge of batteries
3. Improved air quality
4. Cancer treatment
5. Better packaging of food
Select the correct answer using the code given below:
{OPTION 1} 1, 3 and 4 only
{OPTION 2} 2, 3 and 5 only
{OPTION 3} 1 and 4 only
{OPTION 4} 1, 2, 3, 4 and 5
{RIGHT ANSWER} 4
{EXPLANATION} Recent Context:
▪ Scientists have developed Nano-bubbles that can deliver drugs in the body
when triggered by standard X-rays and may pave the way for a new range of
cancer treatments.
▪ Initial testing has shown this technique to be highly efficient in killing bowel
cancer cells.

Other Applications of Nano-technology:


▪ Precise Drug Delivery: Customized nanoparticles the size of molecules can
deliver drugs directly to diseased cells in your body. When it is perfected, this
method would greatly reduce the damage treatment such as chemotherapy etc.
▪ Food packaging: Companies are developing nanomaterials for food packaging
that will make a difference in not only the taste of food but also in food safety
and the health benefits that food delivers.
▪ Fuel Cells: Nanotechnology is being used to reduce the cost of catalysts used in
fuel cells to produce hydrogen ions from fuel such as methanol and to improve
the efficiency of membranes used in fuel cells to separate hydrogen ions from
other gases such as oxygen
▪ Batteries: Companies have developed nanotech solar cells that can be
manufactured at a significantly lower cost than conventional solar cells.
Companies are currently developing batteries using nanomaterials. One such
battery will be good as new after sitting on the shelf for decades. Another
battery can be recharged significantly faster than conventional batteries.
▪ Space: Nanotechnology may hold the key to making space flight more practical.
Advancements in nanomaterials make lightweight spacecraft and a cable for the
space elevator possible. By significantly reducing the amount of rocket fuel

Have a querry?
Drop us a mail at – hello@civilstap.com or call us at 8146207241
66
Revise your past 1.5 years of current affairs with our most comprehensive Test Series on Current Affairs

required, these advances could lower the cost of reaching orbit and traveling in
space.
▪ Fuels: Nanotechnology can address the shortage of fossil fuels such as diesel
and gasoline by making the production of fuels from low-grade raw materials
economical, increasing the mileage of engines, and making the production of fuels
from normal raw materials more efficient.
▪ Better Air Quality: Nanotechnology can improve the performance of catalysts
used to transform vapours escaping from cars or industrial plants into harmless
gasses.
▪ Textile Industry: Making composite fabric with Nano-sized particles or fibres
allows improvement of fabric properties without a significant increase in weight,
thickness, or stiffness, as might have been the case with previously used
techniques.
▪ Chemical Sensors: Nanotechnology can enable sensors to detect very small
amounts of chemical vapours.
▪ https://www.thehindu.com/sci-tech/health/nano-bubbles-triggered-by-x-ray-
can-target-cancer/article24435368.ece
▪ http://www.understandingnano.com/nanotech-applications.html

{QUESTION NUMBER} 46

Have a querry?
Drop us a mail at – hello@civilstap.com or call us at 8146207241
67
Revise your past 1.5 years of current affairs with our most comprehensive Test Series on Current Affairs

{TAGS} Biodiversity in news, Science and Tech


With reference to the "Salicornia Plant", mentioned often in the news, consider the
following statements:
1. It grows in salty marshes in the mangrove wetlands
2. It is used in the treatment of hypertension and diabetes
3. India is the largest producer of this plant
Which of the statement/s given above is/are correct?
{OPTION 1} 1 and 2 only
{OPTION 2} 2 and 3 only
{OPTION 3} 1 and 3 only
{OPTION 4} 1, 2 and 3
{RIGHT ANSWER} 1
{EXPLANATION}
Recent Context:
The government of Andhra Pradesh is intensifying the efforts to tap commercial
benefits from the Salicornia Plant through cultivation as well as extraction
of the substitute to salt with low-sodium content.
Salicornia Plant:
▪ This plant grows in salty marshes in the mangrove wetlands.
▪ The plant can be used as a substitute for salt with low sodium content.
▪ The patients suffering from hypertension, diabetes and gastric related ailments
prefer the Salicornia salad and salt.
▪ The production of the salt substitute has come down drastically in Gujarat with
the scarcity of the Salicornia. Thus, India is forced to import it from Israel and
Scandinavian countries.
https://www.thehindu.com/news/national/andhra-pradesh/state-gears-up-to-tap-
benefits-of-salicornia-in-krishna/article24490394.ece

{QUESTION NUMBER} 47
{TAGS} Indian institutions and bodies in news
In the Indian context, what is the implication of becoming the "Institutions of
Eminence"?
1. They would be given significant autonomy in operations
2. Grant of 1000 crore would be provided to them over 5 years
3. IIT Kharagpur has been declared as the Institution of Eminence
Which of the statement/s given above is/are correct?
{OPTION 1} 1 and 2 only
{OPTION 2} 2 and 3 only
{OPTION 3} 1 and 3 only
{OPTION 4} 1, 2 and 3
{RIGHT ANSWER} 1
{EXPLANATION} Recent Context:

Have a querry?
Drop us a mail at – hello@civilstap.com or call us at 8146207241
68
Revise your past 1.5 years of current affairs with our most comprehensive Test Series on Current Affairs

▪ The Government has shortlisted Six Institutions of Eminence (IoEs) including 3


from Public Sector and 3 from Private Sector.
▪ Public Sector:
▪ Indian Institute of Science, Bangalore, Karnataka
▪ Indian Institute of Technology, Bombay, Maharashtra
▪ Indian Institute of Technology, Delhi
▪ Private Sector:
▪ Jio Institute (Reliance Foundation), Pune under Green Field Category
▪ Birla Institute of Technology & Sciences, Pilani, Rajasthan
▪ Manipal Academy of Higher Education, Manipal, Karnataka
▪ It is expected that the above selected Institutions will come up in the top 500 of
the world ranking in 10 years and in top 100 of the world ranking eventually
over time.
To achieve the top world ranking, these Institutions shall be provided with-
1. Greater autonomy to admit foreign students up to 30% of admitted students
2. Recruit foreign faculty upto 25% of faculty strength
3. Offer online courses upto 20% of its programmes
4. Enter into an academic collaboration with top 500 in the world ranking
Institutions without permission of UGC
5. Free to fix and charge fees from foreign students without restriction
6. Flexibility of course structure in terms of the number of credit hours and years
to take a degree
7. Complete flexibility in fixing of curriculum and syllabus, among others.
8. Each public Institution selected, as ‘Institution of Eminence’ will get financial
assistance up to Rs. 1000 Crore over the period of five years under this scheme.

Have a querry?
Drop us a mail at – hello@civilstap.com or call us at 8146207241
69
Revise your past 1.5 years of current affairs with our most comprehensive Test Series on Current Affairs

9.
https://www.thehindubusinessline.com/opinion/columns/slate/all-you-wanted-
to-know-aboutinstitution-of-eminence/article24496811.ece

{QUESTION NUMBER} 48
{TAGS} Government Schemes and programmes
With reference to "DigiLocker", consider the following statements:
1. It is available only for Indian citizens
2. It is a part of Digital India Initiative
3. Ministry of Railways has approved digital Aadhaar and Driving Licence from
DigiLocker as proof of identity of passengers
Which of the statement(s) given above is/are correct?
{OPTION 1} 1 and 2 only
{OPTION 2} 2 and 3 only
{OPTION 3} 1 and 3 only
{OPTION 4} 1, 2 and 3
{RIGHT ANSWER} 4
{EXPLANATION} Recent Context:
▪ Ministry of Railways has decided that while undertaking journey in a train, if a
passenger shows the Aadhaar/Driving Licence from the ‘Issued Documents’ section

Have a querry?
Drop us a mail at – hello@civilstap.com or call us at 8146207241
70
Revise your past 1.5 years of current affairs with our most comprehensive Test Series on Current Affairs

by logging into his/her DigiLocker account, the same should be considered as valid
proof of identity.
▪ It is, however, clarified that the documents uploaded by the user himself/herself
(i.e. the documents in ‘Uploaded Documents’ section) will not be considered as a
valid proof of identity.
DigiLocker
▪ DigiLocker is a platform for issuance and verification of documents and
certificates in a digital format, thereby eliminating the use of physical
documents. Indian citizens who sign up for a DigiLocker account get a dedicated
cloud storage space that is linked to their Aadhaar or UIDAI number.
▪ The DigiLocker facility is available only for Indian citizens. To open a DigiLocker
account, you must have an Aadhaar card (Aadhaar number). To activate the
account, it is mandatory to link your DigiLocker account to your Aadhaar (UIDAI)
number.
▪ DigiLocker is part of the government's Digital India initiative. PM Modi launched
the project in 2015.
Benefits of DigiLocker:
▪ Citizens can access their digital documents anytime, anywhere and share it
online for their ease and convenience. This helps in saving time, paperwork, and
helps people get their work done on the move.
▪ It reduces the administrative overhead of government departments by
minimizing the use of paper, thereby making it a hassle-free and more
environment-friendly way of getting things done.
▪ DigiLocker makes it easier to validate the authenticity of documents as they are
issued directly by the registered issuers. Since the concerned departments verify
documents, it helps a user get many other tasks done, for eg: documents required
for various banking purposes, or at airports and railway stations, where IDs are
needed.
▪ Self-uploaded documents can be digitally signed using the eSign facility (which
is similar to the process of self-attestation of documents). Documents can
thereby be shared as and when required, without it having to be manually
delivered or collected.
https://www.livemint.com/Technology/vt7Sx2J2DTaoyVpMqsRxiM/What-is-
DigiLocker-and-how-to-use-it-to-carry-all-documents.html

{QUESTION NUMBER} 49
{TAGS} Government Schemes and programmes
With reference to the "Global Housing Technology challenge", consider the
following statements:
1. It has been launched under the Pradhan Mantri Awas Yojana-Urban in India
2. It has been launched by the International Federation for Housing and Planning

Have a querry?
Drop us a mail at – hello@civilstap.com or call us at 8146207241
71
Revise your past 1.5 years of current affairs with our most comprehensive Test Series on Current Affairs

Which of the statement/s given above is/are correct?


{OPTION 1} 1 only
{OPTION 2} 2 only
{OPTION 3} Both 1 and 2
{OPTION 4} Neither 1 nor 2
{RIGHT ANSWER} 1
{EXPLANATION} Recent Context:
▪ The Ministry of Housing and Urban Affairs has launched the Global Housing
Technology Challenge to make the construction of houses cost-effective and
innovative.

▪ The aim of the initiative — under the Pradhan Mantri Awas Yojana Urban
(PMAY-U) — is to fast-track the construction of affordable housing and meet the
target of constructing 1.2 crore houses by 2022.
▪ The challenge will also focus on identifying and mainstreaming proven
demonstrable technologies for lighthouse projects and spotting potential future
technologies for incubation and acceleration support through ASHA (Affordable
Sustainable Housing Accelerators) — India.
▪ It will invite ideas from across the globe for alternative technologies that go
beyond the brick-and-mortar building model used widely in Indian construction.
▪ The four parameters in the global challenge would be time, cost, quality, and
sustainability. The technology will have to be better than the existing ones on all
these four fronts.
https://www.thehindubusinessline.com/news/real-estate/ministry-launches-global-
housing-technology-challenge-to-bring-changes-in-
construction/article25992855.ece
https://economictimes.indiatimes.com/news/economy/policy/government-launches-
global-housing-technology-challenge-to-build-houses-in-shorter-
time/articleshow/67528562.cms

{QUESTION NUMBER} 50
{TAGS} Important Bills, Acts and Amendments
With reference to the provisions made under the DNA Technology (Use and
Application) Regulation Bill, 2018, consider the following statements:
1. It provides for the establishment of regional DNA data banks
2. Written consent by individuals is required to collect DNA samples from them
Which of the statements/s given above is/are correct?
{OPTION 1} 1 only
{OPTION 2} 2 only
{OPTION 3} Both 1 and 2
{OPTION 4} Neither 1 nor 2
{RIGHT ANSWER} 3

Have a querry?
Drop us a mail at – hello@civilstap.com or call us at 8146207241
72
Revise your past 1.5 years of current affairs with our most comprehensive Test Series on Current Affairs

{EXPLANATION} Recent context:


▪ The DNA Technology (Use and Application) Regulation Bill, 2018 has been
passed which allows the use of technology to establish the identity of persons
in matters of crime, parentage dispute, emigration or immigration and
transplantation of human organs.
▪ The Bill regulates the use of DNA technology for establishing the identity of
persons.
▪ The Bill establishes a National DNA Data Bank and Regional DNA Data Banks.

▪ Every Data Bank will maintain the following indices:


1.crime scene index,
2.suspects or undertrials index,
3.offenders’ index,
4.missing persons’ index, and
5.unknown deceased persons’ index.
▪ The Bill establishes a DNA Regulatory Board. Every DNA laboratory that analyses
a DNA sample to establish the identity of an individual has to be accredited by the
Board.
▪ Written consent by individuals is required to collect DNA samples from them.
Consent is not required for offences with punishment of more than seven years
of imprisonment or death.
▪ The Bill requires consent of the individual when DNA profiling is used in criminal
investigations and identifying missing persons. However, consent requirements
have not been specified in case of DNA profiling for civil matters.
▪ The Bill provides for the removal of DNA profiles of suspects on the filing of a
police report or court order, and of undertrials based on a court order. Profiles
in the crime scene and missing persons’ index will be removed on a written
request.

Have a querry?
Drop us a mail at – hello@civilstap.com or call us at 8146207241
73
Revise your past 1.5 years of current affairs with our most comprehensive Test Series on Current Affairs

{QUESTION NUMBER} 51
{TAGS} Indian institutions and bodies in news
With reference to the Water and Power Consultancy Services (India) Limited,
consider the following statements:
1. It is a body under the Ministry of Water Resources, River Development and Ganga
Rejuvenation
2. It is a Navratna company
Which of the statement/s given above is/are correct?
{OPTION 1} 1 only
{OPTION 2} 2 only
{OPTION 3} Both 1 and 2
{OPTION 4} Neither 1 nor 2
{RIGHT ANSWER} 1
{EXPLANATION} Recent context:
Water and Power Consultancy Services Limited celebrated its 50th foundation
day on 26th June 2018.
▪ The Government of India incorporated WAPCOS, as a Public Sector Enterprise in
1969.
▪ It is a techno-commercial organization under the aegis of Ministry of Water
Resources, River Development and Ganga Rejuvenation.

▪ The company has contributed in the development of over 15 Million Ha irrigation


potential; more than 200 projects in ports and inland navigation; over 500
projects in water supply and sanitation, rural and urban development, roads and
highway engineering.
▪ In the Hydropower sector, WAPCOS has completed almost 52 Hydro-Power
Projects in 19 countries with an installed capacity of more than 20,500 MW; over
105 Hydro Power Projects in India with an installed capacity of more than 9,000
MW.
▪ WAPCOS was conferred with Mini-Ratna status in 2010.

Have a querry?
Drop us a mail at – hello@civilstap.com or call us at 8146207241
74
Revise your past 1.5 years of current affairs with our most comprehensive Test Series on Current Affairs

{QUESTION NUMBER} 52
{TAGS} Science and tech, Government Rules and Regulations
Consider the following statements about Coal Mine Surveillance & Management
System:
1. It is a GIS application through which the location of sites for unauthorized mining
can be detected
2. It has been launched by the Ministry of Electronics & Information Technology
Which of the statements/s given above is/are correct?
{OPTION 1} 1 only
{OPTION 2} 2 only
{OPTION 3} Both 1 and 2
{OPTION 4} Neither 1 nor 2
{RIGHT ANSWER} 1
{EXPLANATION} Recent Context: Recently, The Ministry of Coal has
launched the Coal Mine Surveillance & Management System (CMSMS) and
‘Khan Prahari’ mobile application.

▪ The basic objective of CMSMS is reporting, monitoring and taking suitable


action on unauthorized coal mining activities.
▪ The CMSMS is a Web-based GIS application through which the location of sites
for unauthorized mining can be detected.
▪ The basic platform used in the system is of Ministry of Electronics & Information
Technology’s (MeiTY) map, which provides village-level information.
▪ The leasehold boundary of all the coal mines are displayed on this map.
▪ The system will use satellite data to detect changes by which unauthorized mining
activity extending beyond the allotted lease area can be detected and suitable
action can be taken on it.
▪ The ‘Khan Prahari’ app is for reporting any activity taking place related to illegal
coal mining like rat-hole mining, pilferage etc.
▪ One can upload geo-tagged photographs of the incident along with textual
information directly to the system.

{QUESTION NUMBER} 53
{TAGS} Government Schemes and programmes
With reference to the Recapitalization of Regional Rural Banks Scheme, which of the
following statement(s) is/are correct?
1. It will enable RRBs to maintain the minimum prescribed CRAR of 9%
2. It has been extended upto the year 2022
Select the correct answer using the codes given below:
{OPTION 1} 1 only
{OPTION 2} 2 only
{OPTION 3} Both 1 and 2
{OPTION 4} Neither 1 nor 2

Have a querry?
Drop us a mail at – hello@civilstap.com or call us at 8146207241
75
Revise your past 1.5 years of current affairs with our most comprehensive Test Series on Current Affairs

{RIGHT ANSWER} 1
{EXPLANATION} Recent context:
▪ Recently, the government has approved the extension of recapitalization
scheme for Regional Rural Banks for next 3 years upto 2019-20 with an aim to
strengthen their lending capacity.
▪ This will enable the RRBs to maintain the minimum prescribed capital to risk-
weighted assets ratio (CRAR) of 9 percent.
▪ A strong capital structure and minimum required level of CRAR will ensure
financial stability of RRBs, which will enable them to play a greater role in financial
inclusion and meeting the credit requirements of rural areas.
▪ The scheme was started in FY2010-11 and has been extended twice in the year
2012-13 and 2015-16. The last extension was up to March 31, 2017.
▪ The identification of RRBs that require recapitalization was decided in
consultation with the National Bank for Agriculture and Rural Development
(NABARD).
▪ RRBs were set up with the objective to provide credit and other facilities,
especially to the small and marginal farmers, agricultural labourers, artisans and
small entrepreneurs in rural areas.

{QUESTION NUMBER} 54
{TAGS} Disease in news
HSV1 virus is sometimes mentioned in the news with reference to which of the
following diseases?
{OPTION 1} AIDS
{OPTION 2} Swine flu
{OPTION 3} Herpes
{OPTION 4} Alzheimer’s
{RIGHT ANSWER} 3
{EXPLANATION} Recent context: Scientists have found evidence of a link
between herpes infection and Alzheimer’s disease as well as the potential of
antivirals in reducing the risk of the neurodegenerative disease.
▪ In the study, when people who suffered from severe herpes infection were
treated aggressively with antiviral drugs, the relative risk of dementia was
reduced by a factor of 10.
▪ Herpes Simplex, commonly known as Herpes is an infectious disease caused by
a Herpes Simplex Virus. It appears in various parts of the body.
▪ They have been classified into two types namely:
1. HSV – 1
2. HSV – 2

{QUESTION NUMBER} 55
{TAGS} Space in news, Science and Tech

Have a querry?
Drop us a mail at – hello@civilstap.com or call us at 8146207241
76
Revise your past 1.5 years of current affairs with our most comprehensive Test Series on Current Affairs

Consider the following statements about the Indian Regional Navigation Satellite
System (IRNSS):
1. IRNSS 1I is the eighth satellite in the satellite system.
2. IRNSS will help in land navigation only.
Which of the above statements is/are correct?
{OPTION 1} 1 only
{OPTION 2} 2 only
{OPTION 3} Both 1 and 2
{OPTION 4} Neither 1 nor 2
{RIGHT ANSWER} 1
{EXPLANATION}
Why in news?
IRNSS 1I satellite was launched by ISRO on April 12, 2018.

What is IRNSS?
• IRNSS stands for Indian Regional Navigation Satellite System.
• It is a set of satellites which together can provide India a regional positioning
system similar to the GPS.
• According to the ISRO website, the system is designed to give position accuracy
better than 20 metres to users in its primary coverage area.
• It can also service regions extending up to 1500 km around India's boundary.
• ISRO has built a total of nine satellites in the IRNSS series of which eight are
currently in orbit.

How many IRNSS satellites are already in orbit?


• There are currently seven IRNSS satellites (1A to 1G) in orbit. IRNSS 1I will be the
eighth in the series.
• A, B, F, G, I are placed in a geosynchronous orbit, which means they seem to be
at a fixed location above the Earth and they orbit along with the Earth.
• C, D, E, are located in geostationary orbit-they seem to be at a fixed location
above the Earth along the equator and orbit along with the Earth.

Specifications of IRNSS-1I:
• The IRNSS-1I weighs 1425 kg at lift-off and is the eighth satellite in the IRNSS
satellite constellation.
• It will be placed in a sub-geosynchronous transfer orbit and at its closest point will
be 284 km above the Earth and at its farthest will be 20,650 km above the Earth.
• Like all other IRNSS satellites, IRNSS-1I will also carry two payloads – navigation
payload and ranging payload - the former to transmit signals for determining

Have a querry?
Drop us a mail at – hello@civilstap.com or call us at 8146207241
77
Revise your past 1.5 years of current affairs with our most comprehensive Test Series on Current Affairs

position, velocity and time and the latter for determining the frequency range of
the satellite.

What are the applications of IRNSS?


• These satellites help not just in land navigation but also in marine and aerial
navigation.
• The data from these satellites can be used to give vehicle drivers visual and voice
navigation assistance.
• They also help in disaster management and in proper time-keeping.

https://www.thehindu.com/sci-tech/science/navigation-satellite-irnss-1i-to-be-
launched-on-april-12/article23494496.ece

{QUESTION NUMBER} 56
{TAGS} Indian institutions and bodies in news
With reference to ‘North Eastern Council’, consider the following statements:
1. It is a statutory body with governors and chief ministers of all the eight north
eastern states as its members.
2. Union Home-Minister is ex-officio chairman of this council.
Which of the statement(s) given above is/are correct?
{OPTION 1} 1 only
{OPTION 2} 2 only
{OPTION 3} Both 1 and 2
{OPTION 4} Neither 1 nor 2
{RIGHT ANSWER} 3
{EXPLANATION} Recent context:
▪ Recently, The Union Cabinet has approved the proposal of Ministry of
Development of North Eastern Region (DoNER) for the nomination of the Union
Home Minister as ex-officio Chairman of North Eastern Council (NEC).
▪ The Cabinet also approved that Minister of State (Independent Charge), Ministry
of DoNER would serve as Vice Chairman of the Council.

About North Eastern Council:


▪ It is an apex level body for securing balanced and coordinated development
and facilitating coordination with the States.
▪ It was established under the North Eastern Council Act, 1971
▪ Subsequent to the Amendment of 2002, NEC has been mandated to function as a
regional planning body for the North Eastern Area and while
formulating a regional plan for this area, shall give priority to the schemes and
projects benefiting two or more states provided that in the case of Sikkim, the
Council shall formulate specific projects and schemes for that State.

Have a querry?
Drop us a mail at – hello@civilstap.com or call us at 8146207241
78
Revise your past 1.5 years of current affairs with our most comprehensive Test Series on Current Affairs

{QUESTION NUMBER} 57
{TAGS} Government Schemes and programmes
With reference to ‘Swajal Scheme’, consider the following statements:
1. It has been launched by the Ministry of Water Resources, River Development and
Ganga Rejuvenation.
2. Under this scheme, Government aims to provide villages with piped water supply
powered by solar energy.
Which of the statement(s) given above is/are correct?
{OPTION 1} 1 only
{OPTION 2} 2 only
{OPTION 3} Both 1 and 2
{OPTION 4} Neither 1 nor 2
{RIGHT ANSWER} 2
{EXPLANATION} Recent context:
In June 2018, A National Consultation on the National Rural Drinking Water
Programme (NRDWP) and Swajal was held in the Capital today to discuss the
reforms needed in NRDWP and to outline a road map for the Swajal scheme.

About Swajal scheme:


▪ Swajal is community-owned drinking water scheme for sustained drinking
water supply.
▪ Under it, 90% of project cost is taken care by the government and remaining 10%
is contributed by beneficiary community.
▪ The operations and management of the project is taken care by the local
villagers.
▪ Under this scheme, government aims to provide villages with piped water supply
powered by solar energy.
▪ Ministry of Drinking water and sanitation has launched Swajal scheme in 115
aspirational districts of India to provide clean water.

{QUESTION NUMBER} 58
{TAGS} Defence in news
‘Pinaka’ sometimes seen in news refers to:
{OPTION 1} A multi-barrel rocket launcher
{OPTION 2} An application for grievance redressal
{OPTION 3} A card payment scheme
{OPTION 4} A portal that enables NGOs to enrol centrally
{RIGHT ANSWER} 1
{EXPLANATION} Recent context:
The indigenous Pinaka rocket system of the Defence Research and Development
Organisation (DRDO), is being evolved into a precision-guided missile, with
enhanced range and accuracy to hit its target.

Have a querry?
Drop us a mail at – hello@civilstap.com or call us at 8146207241
79
Revise your past 1.5 years of current affairs with our most comprehensive Test Series on Current Affairs

About Pinaka:
▪ Pinaka is a multiple rocket launcher produced in India and developed by
the Defence Research and Development Organisation (DRDO) for the Indian
Army.
▪ The system has a maximum range of 40 km for Mark-I and 75 km for Mark-
II, and can fire a salvo of 12 HE rockets in 44 seconds. The system is mounted on
a truck for mobility.
▪ Pinaka saw service during the Kargil War, where it was successful in
neutralising enemy positions on the mountain tops. It has since been inducted into
the Indian Army in large numbers.

{QUESTION NUMBER} 59
{TAGS} International News
Which of the following statement/s is/are correct in context of ‘World Blood Donors
day’?
1. It has been instituted by United Nations Human Rights Council (UNHRC)
2. It is one of the eight official global public health campaigns marked by WHO
Select the correct answer using the code given below:
{OPTION 1} 1 only
{OPTION 2} 2 only
{OPTION 3} Both 1 and 2
{OPTION 4} Neither 1 nor 2
{RIGHT ANSWER} 2
{EXPLANATION} Recent context:
▪ Every year on 14 June, countries around the world celebrate World Blood Donor
Day (WBDD).
▪ The theme for 2018 was ‘Be there for someone else. Give blood. Share life’. The
host was Greece.
About World Blood Donors Day:
▪ The World Blood Donors day was instituted by WHO in 2004, serves to raise
awareness of the need for safe blood and blood products, and to thank blood
donors for their voluntary, life-saving gifts of blood.
▪ World Blood Donor Day is one of eight official global public health campaigns
marked by the World Health Organization (WHO), along with World Health
Day, World Tuberculosis Day, World Immunization Week, World Malaria
Day, World No Tobacco Day, World Hepatitis Day, and World AIDS Day.

{QUESTION NUMBER} 60
{TAGS} International News
With reference to ‘Nuclear Suppliers Group’, consider the following statements:
1. It is a multi-lateral export control regime.
2. It was founded in response to the Indian nuclear test in 1974.
3. In 2018, Estonia became the first Baltic state to chair the NSG.

Have a querry?
Drop us a mail at – hello@civilstap.com or call us at 8146207241
80
Revise your past 1.5 years of current affairs with our most comprehensive Test Series on Current Affairs

Which of the statement(s) given above is/are correct?


{OPTION 1} 1 and 2
{OPTION 2} 2 and 3
{OPTION 3} 1 and 3
{OPTION 4} 1, 2 and 3
{RIGHT ANSWER} 1
{EXPLANATION} Recent context:
In June 2018, The Nuclear Suppliers Group (NSG) plenary meeting took place in
Jurmala, Latvia after Latvia assumed chairmanship of NSG for 2018-19.

About NSG:
▪ The Nuclear Suppliers Group (NSG) is a multilateral export control regime and a
group of nuclear supplier countries that seek to prevent nuclear proliferation by
controlling the export of materials, equipment and technology that can be used
to manufacture nuclear weapons.
▪ The NSG was founded in response to the Indian nuclear test in May 1974 and
first met in November 1975. The test demonstrated that certain non-weapons
specific nuclear technology could be readily turned to weapons development.
▪ Initially it had 7 members, but as of 2019, NSG has 48 participating governments.
▪ In 2018, Latvia became the first Baltic state to chair the NSG.

{QUESTION NUMBER} 61
{TAGS} International News
With reference to “SCOs Qingdao Declaration 2018”, which of the following
statement/s is/are correct?
1. It calls for implementing three-year plan to combat terrorism, separatism and
extremism.
2. India is the only country to refuse to endorse One Belt, One Road (OBOR) project
as part of this declaration.
Select the correct answer using the codes given below:
{OPTION 1} 1 only
{OPTION 2} 2 only
{OPTION 3} Both 1 and 2
{OPTION 4} Neither 1 nor 2
{RIGHT ANSWER} 3
{EXPLANATION} Recent Context:
▪ SCO summit has adopted ‘Qingdao declaration’ calling for 3-year plan to
combat terrorism.
▪ Shanghai Cooperation Organization (SCO) leaders taking part in a summit
in China’s Qingdao have signed the Qingdao Declaration.
▪ The signing ceremony involved the leaders of China, Russia, Kazakhstan,
Kyrgyzstan, Tajikistan, Uzbekistan, India and Pakistan.

Have a querry?
Drop us a mail at – hello@civilstap.com or call us at 8146207241
81
Revise your past 1.5 years of current affairs with our most comprehensive Test Series on Current Affairs

▪ It also calls for implementation of the treaty on long-term good neighborliness,


friendship and co-operation.
▪ The leaders adopted a total of 17 documents at the summit.
▪ India refused to endorse China’s ambitious One Belt, One Road (OBOR)
project as part of Qingdao Declaration at the 18th SCO summit.
▪ At the restricted session of the SCO Summit, Indian Prime Minister Modi floated
concept of SECURE:
‘S’ for security for citizens,
‘E’ for economic development,
‘C’ for connectivity in the region,
‘U’ for unity,
‘R’ for respect of sovereignty and integrity,
‘E for environment protection.

{QUESTION NUMBER} 62
{TAGS} Disease in news
Which of the following statement(s) is/are correct in context of ‘Nipah Virus
Infection’?
1. It is transmitted to humans through animals
2. It was first identified in 1999 in Malaysia and Singapore
3. Chickens are the natural host of this virus
Select the correct answer using the code given below:
{OPTION 1} 1 and 2 only
{OPTION 2} 2 and 3 only
{OPTION 3} 1 and 3 only
{OPTION 4} 1, 2 and 3
{RIGHT ANSWER} 1
{EXPLANATION} Recent Context:
Recently, Kerala government has lifted high alert issued in the wake of outbreak of
deadly Nipah virus in May 2018 in northern Kozhikode and Malappuram district
in Kerala.

About Nipah Virus:


▪ Nipah virus is a zoonotic virus (it is transmitted from animals to humans) and
can also be transmitted through contaminated food or directly between people.
▪ In infected people, it causes a range of illnesses from asymptomatic (subclinical)
infection to acute respiratory illness and fatal encephalitis. The virus can also cause
severe disease in animals such as pigs, resulting in significant economic losses for
farmers.
▪ It was first identified in 1999 during outbreak among pig farmers in Malaysia and
Singapore. It gets its name from a Malaysian village called ‘Sunghai Nipah’,
where pig farmers became ill with encephalitis.

Have a querry?
Drop us a mail at – hello@civilstap.com or call us at 8146207241
82
Revise your past 1.5 years of current affairs with our most comprehensive Test Series on Current Affairs

▪ Fruit Bats are the natural hosts of the virus. Infected bats sheds virus in their
excretion and secretion.

{QUESTION NUMBER} 63
{TAGS} Important Funds
‘Credit Enhancement Fund’ launched by the Union Government aims to:
{OPTION 1} Provide education loan for students of economically weaker sections
{OPTION 2} Facilitate Infrastructure investments by insurance and pension funds
{OPTION 3} Facilitate farmers with credit availability
{OPTION 4} Facilitate infrastructure for Women Safety
{RIGHT ANSWER} 2
{EXPLANATION} Recent context:
▪ Union govt. is going to launch Rs.500 crore ‘Credit Enhancement Fund’ to
facilitate infrastructure investments by insurance and pension funds.
▪ The fund will help in raising credit rating of bonds issued by infrastructure
companies and facilitate investment from long-term investors
▪ The Asian Infrastructure and Investment Bank will take a 10 per cent equity
stake in a ₹500-crore credit enhancement fund that seeks to raise investments in
operational and stabilized infrastructure projects
▪ The initial corpus of the fund, to be sponsored by IIFCL (India Infrastructure
Finance Company).

{QUESTION NUMBER} 64
{TAGS} Places, regions and communities in news
Which of the following pair/s is/are correctly matched?
Temples Places
1. Sabarimala Temple Kerala
2. Raghavendra Swamy Temple Tamil Nadu
3. Brahma Sarovar Temple Haryana
Select the correct answer using the codes given below:
{OPTION 1} 1 and 2 only
{OPTION 2} 2 and 3 only
{OPTION 3} 1 and 3 only
{OPTION 4} 1, 2 and 3
{RIGHT ANSWER} 3
{EXPLANATION} Recent context:
Recently, Union ministry of Drinking water and sanitation has selected 10 more
iconic places under the phase 3 of Swachh Bharat Mission.
Launched in 2016, The Phase I iconic places are:
▪ Ajmer Sharif Dargah, CST Mumbai ,Golden Temple, Kamakhya Temple,
MaikarnikaGhat, Meenakshi Temple, Shri Mata Vaishno Devi, Shree Jagannath
Temple, The Taj Mahal and Tirupati Temple.
▪ Phase II of Swachh Iconic Places launched in Nov 2017 were:

Have a querry?
Drop us a mail at – hello@civilstap.com or call us at 8146207241
83
Revise your past 1.5 years of current affairs with our most comprehensive Test Series on Current Affairs

Gangotri, Yamunotri, Mahakaleshwar Temple, Charminar, Convent and Church of


St. Francis of Assissi, Kalady, Gommateswara, BaidyanathDham, Gaya Tirth and
Somnath temple.
▪ Ten new iconic sites under phase 3 are:
Raghavendra Swamy Temple (Kurnool, Andhra Pradesh); Hazardwari Palace
(Murshidabad, West Bengal); Brahma Sarovar Temple (Kurukshetra, Haryana);
VidurKuti (Bijnor, Uttar Pradesh); Mana village (Chamoli, Uttarakhand); Pangong
Lake (Leh-Ladakh, J&K); Nagvasuki Temple (Allahabad, Uttar Pradesh);
ImaKeithal/market (Imphal, Manipur); Sabarimala Temple (Kerala); and
Kanvashram (Uttarakhand).

{QUESTION NUMBER} 65
{TAGS} Indices
With reference to ‘Composite Water Management Index’, which of the following
statement/s is/are correct?
1. It aims to inspire States/UTs towards efficient and optimal utilization of water and
recycling.
2. It is released by Ministry of Drinking Water and Sanitation.
Select the correct answer using the code given below:
{OPTION 1} 1 only
{OPTION 2} 2 only
{OPTION 3} Both 1 and 2
{OPTION 4} Neither 1 nor 2
{RIGHT ANSWER} 1
{EXPLANATION} Recent context:
▪ In February 2018, NITI Aayog had released a report on “Healthy States,
Progressive India” which covered the ranking of States/ UTs in various health
parameters.
▪ As a step further in direction and keeping in view the criticality of water for life,
NITI Aayog has prepared a report on Composite Water Management Index
(CWMI).
▪ In 2018 report, Gujarat is number one, followed by Madhya Pradesh, Andhra
Pradesh, Karnataka and Maharashtra.
▪ In North Eastern and Himalayan States, Tripura has been adjudged number
1, followed by Himachal Pradesh, Sikkim and Assam.

About Composite Water Management Index:


▪ It has been launched and developed by NITI Aayog comprising 9 broad sectors
with 28 different indicators covering various aspects of ground water, restoration
of water bodies, irrigation, farm practices, drinking water, policy and governance.

Have a querry?
Drop us a mail at – hello@civilstap.com or call us at 8146207241
84
Revise your past 1.5 years of current affairs with our most comprehensive Test Series on Current Affairs

▪ For the purposes of analysis, the reporting states were divided into two special
groups – ‘North Eastern and Himalayan states’ and ‘Other States’, to account for
the different hydrological conditions across these groups.
▪ It is an important tool to assess and improve the performance of States/ Union
Territories in efficient management of water resources.
▪ This has been done through a first of its kind water data collection exercise in
partnership with Ministry of Water Resources, Ministry of Drinking Water &
Sanitation and all the States/ Union Territories.

{QUESTION NUMBER} 66
{TAGS} Places, regions and communities in news
Consider the following pairs and identify the correct one/s:
Regions / Cities seen in news Country
1. Gaza Israel
2. Caracas Venezuela
3. Sulawesi China
Select the correct code:

{OPTION 1} 1 only
{OPTION 2} 1 and 2 only
{OPTION 3} 2 only
{OPTION 4} None of the above
{RIGHT ANSWER} 3

{EXPLANATION}

Pair 1 is incorrectly matched:


• The Gaza Strip or simply Gaza, is a self-governing Palestinian territory.
• It is situated on the eastern coast of the Mediterranean Sea that borders Egypt on
the southwest and Israel on the east and north.
• Gaza and the West Bank are claimed by the State of Palestine and often remain in
news.

Have a querry?
Drop us a mail at – hello@civilstap.com or call us at 8146207241
85
Revise your past 1.5 years of current affairs with our most comprehensive Test Series on Current Affairs

• The Gaza Strip has been in news due to the ongoing tensions between Palestine ad
Israel.
• In October 2018, Palestinian officials claimed that Israeli forces have shot dead six
Palestinians, four of them in a single incident in one of the deadliest attacks along
the security fence separating Gaza and Israel.

Pair 2 is correctly matched:


• Caracas is the capital city of Venezuela.
• It has been in news due to the recent incidents of a failed drone attack on the
Venezuelan President – Nicolas Manduro.
• The incident led to a political turmoil in Venezuela including its capital city of Caracas
and made the international headlines.
Venezuela shares its land boundary with:
• Columbia in the west
• Guyana in the east
• Brazil in south and south-east
Venezuela shares its coast with:
• Caribbean Sea in the north west
• North Atlantic Ocean in the north east

Have a querry?
Drop us a mail at – hello@civilstap.com or call us at 8146207241
86
Revise your past 1.5 years of current affairs with our most comprehensive Test Series on Current Affairs

Pair 3 is incorrectly matched:


• Sulawesi region is situated in Indonesia and not China.
• It has been in news because of a shallow earthquake of magnitude 7.5 (Richter scale)
that had struck in neck of Minahasa Peninsula, Indonesia on 28 September 2018.
• Its epicentre was located in mountainous Donggala Regency, Central Sulawesi.
• This event was preceded by sequence of foreshocks, largest of which was a
magnitude 6.1 tremor and followed by localised tsunami, sweeping shore-lying
houses and buildings on its way.
• Sulawesi Earthquake was the deadliest earthquake to strike Indonesia since 2006
Yogyakarta earthquake, as well as deadliest earthquake worldwide in 2018.
• India launched an operation known as ‘Operation Samudra Maitri’ to provide relief
assistance to Indonesia.

{QUESTION NUMBER} 67
{TAGS} International Relations
Operation Samudra Maitri was in news recently, which of the following correctly
describes it?
{OPTION 1} It is the name of joint naval exercise held recently between India and
Sri Lanka
{OPTION 2} It is the name of maritime exercise held recently between India and
Bangladesh
{OPTION 3} It was the relief operation launched by India to provide relief assistance
to earthquake and tsunami victims in Indonesia
{OPTION 4} It was a rescue operation conducted by Indian navy to rescue a
Bangladeshi commercial vessel in the Indian Ocean Region
{RIGHT ANSWER} 3
{EXPLANATION}
• On September 28th -2018, Indonesia was hit by an earthquake and a tsunami
• ‘Operation Samudra Maitri’ for humanitarian assistance to Indonesia was launched
by India after a telephonic conversation between Prime Minister Narendra Modi and
Indonesian President Joko Widodo.

{QUESTION NUMBER} 68
{TAGS} International Relations
“Wooferendum march” was seen in news recently. Which of the following correctly
describes it?
{OPTION 1} It was a march campaign organised by the animal activists for seeking
protection for stray dogs
{OPTION 2} It was a march organised to seek protection from the menace of stray
dogs
{OPTION 3} It was a march by Anti-Brexit protesters for seeking new referendum
on BREXIT

Have a querry?
Drop us a mail at – hello@civilstap.com or call us at 8146207241
87
Revise your past 1.5 years of current affairs with our most comprehensive Test Series on Current Affairs

{OPTION 4} It was a march by the people in United Kingdom to hold a new


referendum on the issue of Scotland
{RIGHT ANSWER} 3
{EXPLANATION}

Wooferendum March – London 2018


• With a campaign known as “Wooferendum March” – the Britons found a new and a
unique way of seeking attention to the question of BREXIT.
• With tails wagging and raucous barking, hundreds of dogs and their owners rallied
in London in October 2018 to howl (seek support) for a new people’s vote on Brexit,
in what organisers dubbed a “wooferendum march”.
• Although the campaigners managed to gain the attention but the British Prime
Minister Theresa May has ruled out a new Brexit referendum, saying it would betray
the result of the 2016 vote and destroy trust in politicians.
• However, the British politicians from all parties have joined a growing movement for
Britons to have the chance to say whether they have changed their minds.

{QUESTION NUMBER} 69
{TAGS} Trends of important parameters
In context of Index of Eight Core Industries, which of the following is/are correct?
1. Between the period 2012-13 to 2017-18, there has been a steady annual growth in
the overall output of the eight core industries.
2. Among the eight core industries, ‘Petroleum Refinery production’ has the maximum
and ‘fertilizer production’ the minimum weight.
Select the correct code:
{OPTION 1} 1 only
{OPTION 2} 2 only
{OPTION 3} Both 1 and 2
{OPTION 4} Neither 1 or 2
{RIGHT ANSWER} 3
{EXPLANATION}
Statement 1 is correct
• From the figure it can be seen that the overall Index of Industrial Production of the
Eight Core Industries has been increasing steadily between the period from 2012-13
to 2017-18.

Have a querry?
Drop us a mail at – hello@civilstap.com or call us at 8146207241
88
Revise your past 1.5 years of current affairs with our most comprehensive Test Series on Current Affairs

Statement 2 is correct:
• The weights allotted to the respective industries are as follows:
Industry Weight
Allotted
Fertilizers 2.63 %
production
Cement production 5.37 %
Natural Gas 6.88 %
Production
Crude Oil 8.98 %
Production
Coal Production 10.33 %
Steel Production 17.92 %
Electricity 19.85 %
Generation
Petroleum Refinery 28.04 %
Production
• From this table we can infer that fertilizer production has been allotted the minimum
weight, while the petroleum and refinery production has been allotted the maximum
weight.
From examination point of view exact weights need not be remembered, but the trend
and the order of the weights is important.

Have a querry?
Drop us a mail at – hello@civilstap.com or call us at 8146207241
89
Revise your past 1.5 years of current affairs with our most comprehensive Test Series on Current Affairs

{QUESTION NUMBER} 70
{TAGS} Trends of important parameters
In context of Index of Eight Core Industries, which of the following is/are incorrect?
1. Between the period from 2012-13 to 2017-18, the rate of growth in the overall
output of the eight core industries has been uniform.
2. In the same time period, the overall output of the eight core industries has been
increasing steadily.
Select the correct code:
{OPTION 1} 1 only
{OPTION 2} 2 only
{OPTION 3} Both 1 and 2
{OPTION 4} Neither 1 or 2
{RIGHT ANSWER} 1
{EXPLANATION}

Statement 1 is incorrect
• From the figure, it can be seen that between the period from 2012-13 to 2017-18,
the rate of growth in the overall output of the eight core industries has not been
uniform.
• However, as far as the output from the eight core industries is concerned between
the same time period, it is steadily increasing which is evident from the data given
below.

Have a querry?
Drop us a mail at – hello@civilstap.com or call us at 8146207241
90
Revise your past 1.5 years of current affairs with our most comprehensive Test Series on Current Affairs

• Between the period 2012-13 to 2017-18, the steady increase in the overall index of
the eight core industries indicates that the overall output of the eight core industries
has been increasing in the same period.
Since the question has asked for the incorrect statement therefore the answer is
option a.

{QUESTION NUMBER} 71
{TAGS} Science and Tech, Defence in news
Recently China conducted a maiden test flight of an aircraft called as AG600.
Consider the following statements regarding it and identify the correct one/s:
1. It has been indigenously designed and built by China
2. It has been code named as ‘Kunlong’
3. It has been designed to be the world’s largest amphibious aircraft
Select the correct code:
{OPTION 1} 1 only
{OPTION 2} 1 and 2 only
{OPTION 3} 1 and 3 only
{OPTION 4} All of the above
{RIGHT ANSWER} 4
{EXPLANATION}

Have a querry?
Drop us a mail at – hello@civilstap.com or call us at 8146207241
91
Revise your past 1.5 years of current affairs with our most comprehensive Test Series on Current Affairs

https://www.thehindu.com/news/international/china-built-worlds-largest-
amphibious-plane-completes-maiden-flight-test/article25269405.ece
Recent Context:
Recently in October 2018, China-built world’s largest amphibious plane
completed maiden flight test.

What is AG600?
• China’s indigenously designed and built amphibious aircraft AG600 successfully
carried out its first take-off and landing tests in October 2018.
Hence statement 1 is correct
• The aircraft is code-named Kunlong and is also known by the name of AVIC AG600
Kunlong.
(Hence statement 2 is correct)
• Designed to be the world’s largest amphibious aircraft, AG600 is powered by four
domestically-built turboprop engines and has a range of 12 hours.
Hence statement 3 is correct
• It will be mainly used for maritime rescue, fighting forest fires and marine
monitoring.

{QUESTION NUMBER} 72
{TAGS} Places, regions and communities in news
Consider the following pairs and identify the correct one/s:
Cyclones recently Affected
in news Region
1. Leslie Arabian
Peninsula
2. Titli Eastern
Coast of
India
3. Luban Iberian
Peninsula
Select the correct code:
{OPTION 1} 1 only
{OPTION 2} 1 and 2 only
{OPTION 3} 2 only
{OPTION 4} All of the above
{RIGHT ANSWER} 3
{EXPLANATION}
1. https://www.thehindu.com/news/international/hurricane-leslie-heading-to-
portugal-spain/article25213125.ece

Have a querry?
Drop us a mail at – hello@civilstap.com or call us at 8146207241
92
Revise your past 1.5 years of current affairs with our most comprehensive Test Series on Current Affairs

2. https://www.thehindu.com/news/national/other-states/rimes-terms-titli-
cyclone-rarest-of-rare/article25593366.ece
3. https://www.financialexpress.com/photos/business-gallery/1346817/cyclone-
titli-wreaks-havoc-in-odisha-andhra-pradesh-see-devastating-pictures/
4. https://earthobservatory.nasa.gov/images/92877/cyclones-luban-and-titli
Recent Context:
The month of October 2018 witnessed three major cyclones in different parts of the
world. These three cyclones were:
A. Cyclone Leslie – Iberian Peninsula

• Also called as ‘Hurricane Leslie’, it was a long-lived tropical cyclone and had a
varying nature – it means that it constantly fluctuated between tropical storm and
Category 1 hurricane intensity.
• A peculiar phenomenon associated with ‘Leslie’ was that it had a tropical origin, but
it made landfall in the Iberian Peninsula as an extratropical cyclone.
• It had a devastating impact on Spain and Portugal.

B. Cyclone Titli – Eastern Coast of India

• In October 2018, the Eastern Coast of India was hit hard by ‘Cyclone Titli’.
• On October 6, a low-pressure area formed in the Andaman Sea.

Have a querry?
Drop us a mail at – hello@civilstap.com or call us at 8146207241
93
Revise your past 1.5 years of current affairs with our most comprehensive Test Series on Current Affairs

• Over the next two days, the disaster entered the Bay of Bengal and became a
depression on October 8.
• Before being named 'Cyclone Titli', it received the designation BOB 08 from the IMD.
• Afterward, the storm rapidly strengthened, becoming a ‘very severe cyclonic storm’
on October 9, with the strength of a Category 2 hurricane on the Saffir–Simpson
Scale (SSHWS).
Saffir-Simpson Scale
The Saffir-Simpson Hurricane Wind Scale is a 1 to 5 rating based on a hurricane's
sustained wind speed. This scale estimates potential property damage.
Hurricanes reaching Category 3 and higher are considered major hurricanes because
of their potential for significant loss of life and damage.

• It tore into coastal areas of Odisha and Andhra Pradesh states with winds of up to
150km/h (93mph).
• It weakened into a depression before entering West Bengal, wreaking havoc in some
parts of South Bengal.

C. Cyclone Luban

• ‘Luban’ developed on October 6 in the central Arabian Sea.


• Twelve cyclones have formed in this region since 2010, but very few reach the
Arabian Peninsula at cyclone strength.
• Luban is therefore a rare cyclonic event that managed to make its way to the Arabian
Peninsula.
• It was categorised as ‘very severe cyclonic storm’.
• On October 14, it made a landfall in Eastern Yemen (in Arabian Peninsula), as a
cyclonic storm.
• As it progressed further, it weakened over the dry mountainous terrain of the
Arabian Peninsula.
• It produced flooding rains in Somalia, Oman and Yemen.

Have a querry?
Drop us a mail at – hello@civilstap.com or call us at 8146207241
94
Revise your past 1.5 years of current affairs with our most comprehensive Test Series on Current Affairs

{QUESTION NUMBER} 73
{TAGS} Science and Tech, Terms in news
The term “ASMR” was seen in news recently. Which of the following correctly
describes it?
{OPTION 1} It is an anti-aircraft missile technology.
{OPTION 2} It is a type of missile defence system intended to shield a country
against incoming missile.
{OPTION 3} It is an experience characterized by a tingling sensation on the skin that
typically begins on the scalp and moves down the back of the neck and upper
spine.
{OPTION 4} None of the above
{RIGHT ANSWER} 3
{EXPLANATION}
https://www.thehindu.com/sci-tech/technology/internet/a-new-digital-
dimension/article25096676.ece
Recent Context:
• ASMR videos made headlines when PayPal came under fire for banning ASMR artists
and freezing their accounts, under the impression (fuelled by trolls) that the content
was sexual.
• The ASMR videos that are gaining popularity on the internet is an example as to
how the internet subculture fuels the scientific research.

What is ASMR?
• ASMR stands for Autonomous Sensory Meridian Response.
• It is an experience characterized by a static-like or tingling sensation on the skin
that typically begins on the scalp and moves down the back of the neck and upper
spine.
Option C is therefore correct.
• The term was first known to be formally coined in 2010 by cybersecurity
professional Jennifer Allen, who runs the ASMR Group on Facebook.
• The research scholars at Shenandoah University USA describe it simply as a variety
of soothing sensations (tingles, relaxation, calmness, sleepiness) due to gentle
stimuli (whispering, soft talking, light touches, methodical sounds).

Why is it becoming popular?


• Users from across the world plug into YouTube, Patreon, and other such sites to
experience ASMR videos that produce these stimuli, called ‘triggers’, to experience
soothing sensations (tingles, relaxation, calmness, sleepiness).

Research on ASMR

Have a querry?
Drop us a mail at – hello@civilstap.com or call us at 8146207241
95
Revise your past 1.5 years of current affairs with our most comprehensive Test Series on Current Affairs

• Modern Life-style is full of stress and causes various lifestyle diseases like insomnia,
hyper tension, anxiety etc. This has caused a need for technological developments
that can soothe a person through gentle stimuli.
• This popular internet experience has encouraged the scientists to explore this field
and many projects have started in this sphere.
• Projects like “Whisperlodge”, which describes itself as the very first in-person
immersive ASMR experience, want to bring ASMR into the real world i.e. take it
beyond social media into the real world.

{QUESTION NUMBER} 74
{TAGS} Science and Tech, Government Schemes and programmes
Consider the following statements regarding “Self4Society” platform:
1. It is the premier workspace for Electronics and IT corporates to organise employee
engagements for social work.
2. It comprises of an ‘admin portal’ and a ‘mobile app’.
Which of the above is/are correct?
{OPTION 1} 1 only
{OPTION 2} 2 only
{OPTION 3} Both 1 and 2
{OPTION 4} Neither 1 or 2
{RIGHT ANSWER} 3
{EXPLANATION}

1) https://www.thehindu.com/sci-tech/technology/gadgets/narendra-modi-to-
launch-app-to-rope-in-volunteers/article25242381.ece
2) https://www.business-standard.com/article/current-affairs/pm-modi-to-launch-
main-nahin-hum-portal-for-it-professionals-on-wednesday-118102301203_1.html

3) http://pib.nic.in/newsite/PrintRelease.aspx?relid=184317

4) https://self4society.mygov.in/faq/

Recent Context:
Prime Minister Shri Narendra Modi recently launched an app and a portal known
as “MainNahin Hum” under the platform called as “Self4Society”.

What is self for society platform?


• The Self4Society platform is the premiere workspace for Electronics and IT
corporates to organise employee engagements (Initiatives) for social work.
(Statement 1 is therefore correct).

Have a querry?
Drop us a mail at – hello@civilstap.com or call us at 8146207241
96
Revise your past 1.5 years of current affairs with our most comprehensive Test Series on Current Affairs

• Through this platform an organisation can create their own initiatives under the
identified National Causes.

Components under Self for Society Platform:

'Self for Society'


Platform

Main Nahin Hum Main Nahin Hum


- Online Portal - Mobile App

• The Self4Society platform comprises of the following:


a. Self4Society Admin portal (self4society.mygov.in) – It is a portal for Electronics
and IT Industry's Corporate HR or CSR representatives, who will create the
organisational profile and upload initiatives.
b. Self4Society App (available on Google Play Store and App Store) – Also known as
‘Main Nahin Hum’, it is an app for the people, who will volunteer for initiatives
and contribute their time to tasks on-the-ground.
Hence statement 2 is also correct.

{QUESTION NUMBER} 75
{TAGS} Science and Technology, Government Schemes and programmes
Consider the following statements regarding “Main Nahin Hum App”:
1. It is one of the components under the ‘Self4Society Platform’.
2. It provides an IT tool to track the progress of various government initiatives.
Which of the above is / are correct?
{OPTION 1} 1 only
{OPTION 2} 2 only
{OPTION 3} Both 1 and 2
{OPTION 4} Neither 1 nor 2
{RIGHT ANSWER} 1
{EXPLANATION}
1) https://www.thehindu.com/sci-tech/technology/gadgets/narendra-modi-to-
launch-app-to-rope-in-volunteers/article25242381.ece

2) https://www.business-standard.com/article/current-affairs/pm-modi-to-
launch-main-nahin-hum-portal-for-it-professionals-on-wednesday-
118102301203_1.html

Have a querry?
Drop us a mail at – hello@civilstap.com or call us at 8146207241
97
Revise your past 1.5 years of current affairs with our most comprehensive Test Series on Current Affairs

3) http://pib.nic.in/newsite/PrintRelease.aspx?relid=184317

4) https://self4society.mygov.in/faq/

Recent Context:
Prime Minister Shri Narendra Modi recently launched an app and a portal known
as “MainNahin Hum” under the platform called as “Self4Service”

Components under Self for Society Platform:

'Self for Society'


Platform

Main Nahin Hum Main Nahin Hum


- Online Portal - Mobile App

• The Self4Society platform comprises of the following:


a. Self4Society Admin portal (self4society.mygov.in) – It is a portal for Electronics
and IT Industry's Corporate HR or CSR representatives, who will create the
organisational profile and upload initiatives.
b. Self4Society App (available on Google Play Store and App Store) – Also known as
‘Main Nahin Hum’, it is an app for the people, who will volunteer for initiatives
and contribute their time to tasks on-the-ground.
Hence statement 1 is correct.

Main Nahin Hum App:


• It provides a platform to the professionals keen on doing volunteer work in their
free time.
(Hence statement 2 is incorrect)
• It has been developed by ‘My Gov’.

About My Gov
• MyGov is a citizen engagement platform founded by the Government of India to promote the active
participation of Indian citizens in their country's governance and development.
• It is aimed at creating a common platform to "crowdsource governance ideas from citizens".
• Its website is managed and hosted by National Informatics Centre (NIC).

{QUESTION NUMBER} 76
{TAGS} Science and Tech, Disease in news

Have a querry?
Drop us a mail at – hello@civilstap.com or call us at 8146207241
98
Revise your past 1.5 years of current affairs with our most comprehensive Test Series on Current Affairs

An antibody known as “p95HER2-TCB” was seen in news recently. It has been


developed to treat which of the following ?
{OPTION 1} Tuberculosis
{OPTION 2} AIDS
{OPTION 3} Breast Cancer
{OPTION 4} Nipah infection
{RIGHT ANSWER} 3
{EXPLANATION}
https://www.thehindu.com/sci-tech/health/new-antibody-for-cancer-
treatment/article25145951.ece
Recent Context:
Recently a team of researchers in Spain, Switzerland and the U.S. have researched on
a specific antibody, called the p95HER2-T cell bispecific antibody (TCB), that
can successfully guide immune cells, known as lymphocytes, directly to
cancerous ones for their targeted killing.

More about p95HER2-T cell:


• Ensuring that the therapeutics only target cancerous cells and not the healthy tissues
is one of the most critical challenge to the cancer treatment.
• The recently developed anti body called as p95HER2-T cell bispecific antibody (TCB)
seeks to address this challenge by exploiting the presence of p95HER2 protein,
which is only located in tumour cells.
• Each antibody molecule has a bipartite structure containing two protein-binding
sites. This means that they can simultaneously attach to immune cells and cancerous
ones as well as take the lymphocytes hand-in-hand directly to the malignant cells
for their subsequent destruction.
• This approach can be used to tackle certain HER2+ breast cancers through its
exclusive targeting of cancerous cells.

{QUESTION NUMBER} 77
{TAGS} Places, regions and communities in news
Cyclone Titli was in news recently, which of the following state/s was/were impacted
by it?
1. Odisha
2. Andhra Pradesh
3. West Bengal
4. Tamil Nadu
Select the correct code:
{OPTION 1} 1 only
{OPTION 2} 2 and 3 only
{OPTION 3} 1, 2 and 3 only
{OPTION 4} All of the above

Have a querry?
Drop us a mail at – hello@civilstap.com or call us at 8146207241
99
Revise your past 1.5 years of current affairs with our most comprehensive Test Series on Current Affairs

{RIGHT ANSWER} 3
{EXPLANATION}

Recent Context:
The Eastern Coast of India was hit by cyclone “Titli” in October 2018.
Origin
• It originated in the first week of October after the development of a low-pressure
area formed in the Andaman Sea.
• Over the next two days, it entered the Bay of Bengal and became a depression on
October 8.
• Afterward, the storm rapidly strengthened, becoming a Very Severe Cyclonic Storm
on October 9, with the strength of a Category 2 hurricane on the Saffir–Simpson
scale (SSHWS).
Saffir-Simpson Scale
The Saffir-Simpson Hurricane Wind Scale is a 1 to 5 rating based on a hurricane's sustained
wind speed. This scale estimates potential property damage. Hurricanes reaching Category
3 and higher are considered major hurricanes because of their potential for significant loss
of life and damage.
Affected Regions

• States affected by cyclone Titli are:


a) Odisha
b) Andhra Pradesh
c) West Bengal
• It tore into coastal areas of Orissa and Andhra Pradesh states with winds of up to
150km/h (93mph).
• It weakened into a depression before entering West Bengal, wreaking havoc in some
parts of South Bengal.

{QUESTION NUMBER} 78
{TAGS} Indices, Rankings
In context of Ease of Doing Business Index, which of the following statement/s is/are
correct?

Have a querry?
Drop us a mail at – hello@civilstap.com or call us at 8146207241
100
Revise your past 1.5 years of current affairs with our most comprehensive Test Series on Current Affairs

1. It ranks countries of the world across 10 indicators.


2. India jumped 77 ranks in Ease of Doing Business Index 2018.
Select the correct code:
{OPTION 1} 1 only
{OPTION 2} 2 only
{OPTION 3} Both 1 and 2
{OPTION 4} Neither 1 or 2
{RIGHT ANSWER} 1
{EXPLANATION}
https://www.thehindu.com/business/Economy/india-jumps-up-in-the-ease-of-doing-
business-index/article25381092.ece
Recent Context:
Recently India has reached 77th rank in the Ease of Doing Business Index, 2018.

Who releases Ease of Doing Business Index?


• It released by World Bank on an annual basis.
• The Index ranks 190 countries across 10 indicators ranged across the lifecycle of a
business from 'starting a business' to 'resolving insolvency'.
Hence statement 1 is correct.
Indicators under Ease of Doing Business Index
The World Bank uses the following indicators to rank the countries:
1. Starting a business
2. Dealing with construction permits
3. Getting electricity
4. Registering property
5. Getting credit
6. Protecting investors
7. Paying taxes
8. Trading across borders
9. Enforcing contracts
10.Resolving insolvency

India’s performance in Ease of Doing Business Report

Have a querry?
Drop us a mail at – hello@civilstap.com or call us at 8146207241
101
Revise your past 1.5 years of current affairs with our most comprehensive Test Series on Current Affairs

• India jumped 23 ranks and has reached rank 77 in the World Bank's Ease of Doing
Business Index 2018. (Hence statement 2 is incorrect).
• India was ranked 100 last year and 130 in 2016 and 2015. When Modi government
took over in 2014, it was ranked 142 among 190 nations.

Performance of other countries:


• New Zealand topped the ranking for the third time consecutively.
• Among the BRICS Nation, the ranking in 2018 is as follows:
BRICS Ranking
Nation
Russia 31
China 46
India 77
South 82
Africa
Brazil 109

• Among India’s Neighbouring States, the rankings of the nations are as follows:
India’s Ranking
Neighbour
Bhutan 81
Sri Lanka 100
Nepal 110
Pakistan 136
Myanmar 171
Bangladesh 176

{QUESTION NUMBER} 79
{TAGS} Reports, Indices, Trends of parameters

Have a querry?
Drop us a mail at – hello@civilstap.com or call us at 8146207241
102
Revise your past 1.5 years of current affairs with our most comprehensive Test Series on Current Affairs

Consider the following statements regarding “World Economic Outlook” and identify
the incorrect one/s:
1. It is an annual flagship report of the World Bank.
2. The report of 2018 was released during annual meet of World Bank and IMF held at
Bali, Indonesia.
Select the correct code:
{OPTION 1} 1 only
{OPTION 2} 2 only
{OPTION 3} Both 1 and 2
{OPTION 4} Neither 1 or 2
{RIGHT ANSWER} 1
{EXPLANATION}
https://www.thehindu.com/business/Economy/global-growth-has-plateaued-at-37-
per-cent-clouds-on-the-horizon-imf/article25163744.ece
Recent Context
International Monetary Fund (IMF) released its annual flagship report in October
2018 during the annual IMF and World Bank meeting in Bali, Indonesia.
Hence statement 1 is incorrect and statement 2 is correct.

Overview of World Economic Outlook Report:


• IMF has projected that the Global growth has plateaued at 3.7 per cent.
• It has projected that the global growth will remain steady over 2018—19 at last
year’s rate of 3.7 per cent. This growth exceeds that achieved in any of the years
between 2012 and 2016.

{QUESTION NUMBER} 80
{TAGS} Indian institutions and bodies in news
National Institute of Mental Health Rehabilitation was sometimes seen in news.
Which of the following statement/s regarding it is/are correct?
1. It will function under Department of Health Research.
2. It will serve as institution of excellence for capacity building in human resource and
research in area of mental health rehabilitation.
3. Union Cabinet has decided to set it up in Sehore District, Madhya Pradesh.
Select the correct code:
{OPTION 1} 1 only
{OPTION 2} 1 and 3 only
{OPTION 3} 3 only
{OPTION 4} 2 and 3 only
{RIGHT ANSWER} 4
{EXPLANATION}
http://www.pib.nic.in/PressReleaseIframePage.aspx?PRID=1548399
Recent Context:

Have a querry?
Drop us a mail at – hello@civilstap.com or call us at 8146207241
103
Revise your past 1.5 years of current affairs with our most comprehensive Test Series on Current Affairs

In October 2018, the Union Cabinet modified its earlier decision to establish
National Institute of Mental Health Rehabilitation in Sehore District
(Madhya Pradesh) instead of Bhopal (Madhya Pradesh).
(Hence, Statement 3 is correct)

About National Institute of Mental Health Rehabilitation (NIMHR)


• It will be established as society under Societies Registration Act, 1860.
• It will function under Department of Empowerment of Persons with Disabilities.
(Hence, Statement 1 is incorrect)
• It will have nine departments and centres and will conduct 12 courses to offer
diploma, certificate, graduate, post graduate, M.Phil degrees in field of mental health
rehabilitation.
• It will serve as institution of excellence for capacity building in human resource and
research in area of mental health rehabilitation. (Hence, statement 2 is correct)
• It will be also recommending body suggesting models and protocols for effective
rehabilitation of persons with mental illness.

Objectives of NIMHR
• Its objective is to provide rehabilitation services to persons with mental illness,
capacity development in field of mental health rehabilitation, policy framing and
advanced research in mental health rehabilitation.

{QUESTION NUMBER} 81
{TAGS} Government Schemes and programmes
With reference to ‘Seva Bhoj yojana’, which of the following statement/s is/are
correct?
1. It has been introduced by the Ministry of Consumer Affairs, Food and Public
Distribution
2. The scheme envisages reimbursing the Central Government share of Central Goods
and Services Tax (CGST) and Integrated Goods and Service Tax (IGST).
3. Under this Yojana, eligible religious institution shall first register with Darpan
Portal.
Select the correct answer using the code given below:
{OPTION 1} 1 and 3 only
{OPTION 2} 2 only
{OPTION 3} 2 and 3 only
{OPTION 4} 1, 2 and 3
{RIGHT ANSWER} 3
{EXPLANATION} Recent Context: Ministry of Culture has launched the ‘Seva
Bhoj Yojana’.

About Seva Bhoj Yojana:

Have a querry?
Drop us a mail at – hello@civilstap.com or call us at 8146207241
104
Revise your past 1.5 years of current affairs with our most comprehensive Test Series on Current Affairs

Launched by- Ministry of culture


Total outlay- Rs. 325.00 Crores for Financial Years 2018-19 and 2019-20.
▪ Purpose- The objective of the scheme is to lessen the financial burden of such
Charitable Religious Institutions who provide Food/Prasad/Langar (Community
Kitchen)/Bhandara free of cost without any discrimination to Public/Devotees.
▪ The scheme envisages to reimburse the Central Government share of Central
Goods and Services Tax (CGST) and Integrated Goods and Service Tax (IGST) on
purchase of raw items such as ghee, edible oil, Atta/Maida/Rava/flour , rice
pulses, sugar, burra/jiggery etc. which go into preparation of
food/Prasad/Langar/Bhandara offered free of cost by religious institutions.
Eligibility:
▪ The Charitable Religious Institutions such as Temples, Gurudwara, Mosque,
Church, Dharmik Ashram, Dargah, Math, Monasteries etc. which have been in
existence for at least five years before applying for financial assistance/grant
▪ who serve free food to at least 5000 people in a month
▪ such institutions covered under Section 10( 23BBA) of the Income Tax Act or
Institutions registered as Society under Societies Registration Act ( XXI of
1860) or as a Public Trust under any law
▪ Religious bodies constituted under any Act or institutions registered under Section
12AA of Income Tax Act shall be eligible for grant under the scheme.
▪ All the eligible institutions should be registered with Darpan portal. The
details of registered institutions will be available on an online portal for the
viewership of public, GST authorities and entity/institution itself.

{QUESTION NUMBER} 82
{TAGS} Defence in news
Which of the following sailing vessel is related to the ‘Navika Sagar Parikrama’?
{OPTION 1} INSV Tarini
{OPTION 2} INSV Mhadei
{OPTION 3} INSV Tarangini
{OPTION 4} INSV Sudarshini
{RIGHT ANSWER} 1
{EXPLANATION} Recent context:
▪ The six women officers comprising the crew of INSV Tarini, who recently
completed a circumnavigation of the globe.
▪ The expedition aboard INSV Tarini was flagged off on September 10, 2017 from
Goa and ended with the return to Goa on May 10, 2018.
▪ The vessel was skippered by Lieutenant Commander Vartika Joshi, and the crew
comprised Lieutenant Commanders Pratibha Jamwal, P Swathi, B Aishwarya and
Lieutenants S Vijaya Devi and Payal Gupta.
▪ During its 254 day voyage, the vessel visited Australia, New Zealand, the Falkland
Islands (UK), South Africa and Mauritius.

Have a querry?
Drop us a mail at – hello@civilstap.com or call us at 8146207241
105
Revise your past 1.5 years of current affairs with our most comprehensive Test Series on Current Affairs

▪ The crew also collated and updated meteorological, ocean and wave data on a
regular basis for accurate weather forecast by the India Meteorological Department
(IMD), and monitored and reported marine pollution on the high seas.

{QUESTION NUMBER} 83
{TAGS} Government Schemes and programmes
With reference to ‘Benami Transaction Informants Reward Scheme’, which of the
following statement/s is/are correct?
1. It has been launched by the Ministry of Corporate Affairs.
2. Its objective is to obtain people’s participation in the efforts to unearth black money
and to reduce tax evasion.
3. A person can get a reward up to Rs. 1 Lakh for giving specific information.
Select the correct answer using the code given below:
{OPTION 1} 1 and 3 only
{OPTION 2} 2 only
{OPTION 3} 2 and 3 only
{OPTION 4} 1, 2 and 3
{RIGHT ANSWER} 2
{EXPLANATION} Recent context:
Income tax Department has launched Benami Transaction Informants Reward
Scheme

About Benami Transaction Informants Reward Scheme:


Introduced by- Income Tax Department
Purpose- Obtaining people’s participation in the Income Tax Department’s
efforts to unearth black money and to reduce tax evasion
Need:-
It was found in many cases that black money was invested in properties in the names
of others, even though benefits were enjoyed by the investor concealing his
beneficial ownership in his tax returns.
▪ This reward scheme is aimed at encouraging people to give information about
Benami transactions and properties as well as income earned on such properties by
such hidden investors and beneficial owners.
▪ Under the Benami Transactions Informants Reward Scheme, 2018, a person can
get reward up to Rs. One crore for giving specific Information.
▪ Information can be given to the Joint or Additional Commissioners of Benami
Prohibition Units (BPUs) in Investigation Directorates of Income Tax Department
about Benami transactions and properties as well as proceeds from such properties
which are actionable under Benami Property Transactions Act, 1988, as amended
by Benami Transactions (Prohibition) Amendment Act, 2016.
▪ Foreigners will also be eligible for such reward.
▪ Identity of the persons giving information will not be disclosed and strict
confidentiality shall be maintained.

Have a querry?
Drop us a mail at – hello@civilstap.com or call us at 8146207241
106
Revise your past 1.5 years of current affairs with our most comprehensive Test Series on Current Affairs

{QUESTION NUMBER} 84
{TAGS} Places, regions and communities in news
Consider the following pairs:
Islands Country
4. Sylt Denmark
5. Socotra Yemen
6. Corsica France

Which of the pairs given above are correctly matched?


{OPTION 1} 1 and 3 only
{OPTION 2} 2 and 3 only
{OPTION 3} 1 only
{OPTION 4} 1, 2 and 3
{RIGHT ANSWER} 2
{EXPLANATION} Recent Context:
▪ Recently, 38 Indians were brought back to the home by the Indian Navy after they
were rescued from the cyclone-hit Socotra Island in Yemen.
▪ The Indians were stranded in Socotra Island after a cyclone (named Mekunu) hit
the area and INS Sunayana had evacuated them in an operation named “Nistar”.
▪ Corsica is an island in the Mediterranean Sea and one of the 18 regions of the
France.
▪ Sylt is an Island in northern Germany, part of Nordfriesland district and well
known for the distinctive shape of its shoreline.

{QUESTION NUMBER} 85
{TAGS} Science and Tech
‘Ensemble Prediction System’ launched by the Ministry of Earth Sciences pertains
to?
{OPTION 1} Traffic estimation and prediction
{OPTION 2} Weather forecast
{OPTION 3} Effective heart disease prediction
{OPTION 4} Tsunami prediction
{RIGHT ANSWER} 2
{EXPLANATION} Recent context: Ministry of Earth Sciences has introduced
Ensemble Prediction system for weather forecasts.

About Ensemble Prediction System:


Introduced by- Ministry of Earth Sciences
Purpose- For generating operational 10-days probabilistic forecasts of weather
▪ It has been developed by 3 bodies-
1. Indian Meteorological Department
2. National Centre for Medium range weather forecasting (NCMRWF)

Have a querry?
Drop us a mail at – hello@civilstap.com or call us at 8146207241
107
Revise your past 1.5 years of current affairs with our most comprehensive Test Series on Current Affairs

3. Indian institute of Tropical Meteorology


▪ This systems would have a resolution of 12 km grid scale, marking a big jump
from the present level of 23 km.
▪ In addition, it will be based on a 20-point ensemble system that would generate
a range of forecasts using slightly varying initial conditions.
▪ The systems would particularly be of help to disaster management authorities
and other stakeholders in making better emergency response decisions by
explicitly accounting for uncertainties in weather forecasts.

{QUESTION NUMBER} 86
{TAGS} Terms in news
In context of ‘Kashmir Super 50’, which of the following statement/s is/are correct?
1. It is a joint initiative by Indian Army, Center for Social Responsibility and
Leadership (CSRL) and PETRONET LNG Limited
2. It has been launched to transform the educational status of children from
economically weaker sections in the Kashmir region
Select the correct answer using the code given below:
{OPTION 1} 1 only
{OPTION 2} 2 only
{OPTION 3} Both 1 and 2
{OPTION 4} Neither 1 nor 2
{RIGHT ANSWER} 3
{EXPLANATION} Recent Context:
▪ A group of 30 students, part of Kashmir Super 50 visited New Delhi and
interacted with General Bipin Rawat, Chief of the Army Staff on 12 Jun 18.
▪ The General shared his experiences while serving in these areas, with the students
and motivated them to work hard & contribute actively in the nation building
process.

About Project Kashmir Super 50:


▪ It is an initiative to transform the educational status of children from
economically weaker sections in the Kashmir region. It was launched on 22
March 2013.
▪ It is a joint initiative by Indian Army, Center for Social Responsibility and
Leadership (CSRL) and PETRONET LNG Limited (PLL).
▪ The Kashmir Super 50 program is an 11 month; completely free residential
coaching for students belonging to the underprivileged section of the society.
▪ The students are prepared for IIT and other premier engineering institutes.
▪ The Students are selected for the Programme through an entrance test and interview
conducted at various colleges throughout the state in the months of April and May.

Have a querry?
Drop us a mail at – hello@civilstap.com or call us at 8146207241
108
Revise your past 1.5 years of current affairs with our most comprehensive Test Series on Current Affairs

{QUESTION NUMBER} 87
{TAGS} Science and Tech
With reference to ‘Lithium-ion Batteries’, which of the following statement/s is/are
correct?
1. These are common rechargeable batteries for portable electronics, with a
high energy density, no memory effect and low self-discharge.
2. A group of students at IIT, Delhi has developed an indigenous technology of
Lithium-ion cells.
Select the correct answer using the code given below:
{OPTION 1} 1 only
{OPTION 2} 2 only
{OPTION 3} Both 1 and 2
{OPTION 4} Neither 1 nor 2
{RIGHT ANSWER} 1
{EXPLANATION} Recent Context:
▪ Central Electro Chemical Research Institute (CECRI), Karaikudi, Tamil Nadu
under Council of Scientific & Industrial Research (CSIR) and RAASI Solar
Power Pvt Ltd have signed a Memorandum of Agreement for transfer of
technology for India’s first Lithium Ion (Li-ion) Battery project.
▪ A group at CSIR-CECRI headed by Dr Gopu Kumar has developed an
indigenous technology of Lithium-ion cells in partnership with CSIR-National
Physical Laboratory (CSIR-NPL) New Delhi, CSIR- Central Glass and
Ceramic Research Institute (CSIR-CGCRI) Kolkata and Indian Institute of
Chemical Technology (CSIR-IICT) Hyderabad.
▪ CSIR-CECRI has set up a demo facility in Chennai to manufacture prototype
Lithium-Ion cells. It has secured global IPRs with potential to enable cost
reduction, coupled with appropriate supply chain and manufacturing technology
for mass production.
About Lithium-ion Battery:
▪ A lithium-ion battery or Li-ion battery (abbreviated as LIB) is a type
of rechargeable battery in which lithium ions move from the negative electrode to
the positive electrode during discharge and back when charging.
▪ Lithium-ion batteries are common rechargeable batteries for portable
electronics, with a high energy density, no memory effect (other than LFP
cells) and low self-discharge. LIBs are also growing in popularity for
military, battery electric vehicle and aerospace applications.

Have a querry?
Drop us a mail at – hello@civilstap.com or call us at 8146207241
109
Revise your past 1.5 years of current affairs with our most comprehensive Test Series on Current Affairs

{QUESTION NUMBER} 88
{TAGS} Indian institutions and bodies in news
In context of ‘Public Credit Registry’, which of the following statement/s is/are
correct?
1. It is an information repository that collates all loan information of individuals and
corporate borrowers.
2. It has been set up on the recommendations of a committee headed by Viral
Acharya.
3. It has been set up by the Reserve Bank of India.
Select the correct answer using the code given below:
{OPTION 1} 1 and 2 only
{OPTION 2} 2 and 3 only
{OPTION 3} 1 and 3 only
{OPTION 4} 1, 2 and 3
{RIGHT ANSWER} 3
{EXPLANATION} Recent Context: RBI has announced to set up PCR for India
with view to address information asymmetry, foster access to credit and
strengthen the credit culture in the economy.

About Public Credit Registry:


▪ Public credit registry is an information repository that collates all loan information
of individuals and corporate borrowers.
▪ A credit repository helps banks distinguish between a bad and a good borrower
and accordingly offer attractive interest rates to good borrowers and higher
interest rates to bad borrowers.
▪ The move is based on the recommendations of a committee, headed by Y.M.
Deosthalee.
▪ PCR will address issues such as information asymmetry, improve access to
credit and strengthen the credit culture among consumers.
▪ It can also address the bad loan problem staring at banks, as corporate debtors
will not be able to borrow across banks without disclosing existing debt.
▪ A PCR may also help raise India’s rank in the global ease of doing business index.
▪ The committee has suggested the registry should capture all loan information
and borrowers be able to access their own history. Data is to be made available to
stakeholders such as banks, on a need-to-know basis. Data privacy will be
protected.

{QUESTION NUMBER} 89
{TAGS} Government Schemes and programmes
With reference to ‘Dam Rehabilitation & Improvement Project (DRIP), which of the
following statement/s is/are correct?
1. It has been launched under Ministry of Drinking water and sanitation
2. It was launched in 2012 by Central Water Commission

Have a querry?
Drop us a mail at – hello@civilstap.com or call us at 8146207241
110
Revise your past 1.5 years of current affairs with our most comprehensive Test Series on Current Affairs

Select the correct answer using the code given below:


{OPTION 1} 1 only
{OPTION 2} 2 only
{OPTION 3} Both 1 and 2
{OPTION 4} Neither 1 nor 2
{RIGHT ANSWER} 2
{EXPLANATION} Recent Context:
Dam Rehabilitation and Improvement Project Scheme has now been extended
till June 2020 with total cost of Rs 3466 crore.

About Dam Rehabilitation and Improvement Project:


▪ Dam Rehabilitation and Improvement Project (DRIP) is a state sector scheme
with central component to improve safety and operational performance of
selected dams, along with institutional strengthening with system wide
management approach.
▪ The project was launched in 2012 by Central Water Commission (CWC) under
Ministry of Water Resources, River Development & Ganga Rejuvenation
with assistance from World Bank.
▪ Originally the scheme was scheduled for six years with closure in June 2018
with total original cost of Rs. 2100 crore.

{QUESTION NUMBER} 90
{TAGS} Indices
Which of the following gives ‘Global Peace Index’ ranking to the countries of the
world?
{OPTION 1} World Economic Forum
{OPTION 2} UN Human Rights Council
{OPTION 3} Institute for Economics and Peace
{OPTION 4} International Monetary Fund
{RIGHT ANSWER} 3
{EXPLANATION} Recent context:
India fetched 136th rank among 163 countries on the 2018 Global Peace Index
that was released on June 7, 2018 by the Institute of Economics and Peace
(IEP), an Australian think tank.
▪ Iceland managed to retain its position at top as the most peaceful country in the
world.
▪ Iceland fetched 1st rank on the index, a position it has held since 2008, followed
by New Zealand and Austria at 2nd and 3rd position.
▪ Portugal and Denmark rounded up the top five most peaceful rankings.
▪ Syria remains the least peaceful country in the world, a position it has held for the
past five years.
▪ Afghanistan, South Sudan, Iraq and Somalia round up the list of five least
peaceful countries.

Have a querry?
Drop us a mail at – hello@civilstap.com or call us at 8146207241
111
Revise your past 1.5 years of current affairs with our most comprehensive Test Series on Current Affairs

{QUESTION NUMBER} 91
{TAGS} Government Rules and Regulations
Consider the following statements regarding Eco Sensitive Zones:
1. These are also called Ecologically Vulnerable Areas.
2. These are regulated by the respective state governments.
3. These zones are mentioned in the Environment (Protection) Act, 1986
4. Elephant Corridors come under such zones
Which of the above statement(s) is/are incorrect?
{OPTION 1} 1 and 3 only
{OPTION 2} 4 only
{OPTION 3} 4 only
{OPTION 4} All of the above
{RIGHT ANSWER} 4
{EXPLANATION}
Recent Context:
The draft re-notification of the Eco Sensitive Zone (ESZ) of Bannerghatta National
Park (BNP) by the Centre has come as a shock to many.

▪ Eco-Sensitive Zones (ESZs) or Ecologically Fragile Areas (EFAs) are areas notified by
the Ministry of Environment, Forests and Climate Change (MoEFCC)
around Protected Areas , National Parks and Wildlife Sanctuaries.
▪ The purpose of declaring ESZs is to create some kind of “shock absorbers” to the
protected areas by regulating and managing the activities around such areas. They
also act as a transition zone from areas of high protection to areas involving lesser
protection.
▪ They are regulated by central government through Ministry of Environment, Forests
and Climate change (MoEFCC). The Environment (Protection) Act, 1986 does not
mention the word “Eco-Sensitive Zones”.
▪ Section 3(2)(v) of the Act, says that Central Government can restrict areas in which
any industries, operations or processes or class of industries, operations or processes
shall not be carried out or shall be carried out subject to certain safeguards.

Have a querry?
Drop us a mail at – hello@civilstap.com or call us at 8146207241
112
Revise your past 1.5 years of current affairs with our most comprehensive Test Series on Current Affairs

{QUESTION NUMBER} 92
{TAGS} Defence in news
Consider the following statements regarding “Kamorta”:
1. It is a class of Indian Navy anti-submarine warfare stealth corvettes to be built in
India
2. It is a village in the Nicobar district of Andaman and Nicobar Islands.
Which of the above statement(s) is/ are correct?
{OPTION 1} 1 only
{OPTION 2} 2 only
{OPTION 3} Both 1 and 2
{OPTION 4} Neither 1 or 2
{RIGHT ANSWER} 4
{EXPLANATION} Recent Context:
▪ India and Russia have sealed a $500 million deal for construction of 2 missile frigates
of Talvar Class for the Indian Navy.
▪ These are a class of guided missile frigates designed and built by Russia for the Indian
Navy.
▪ They are modified Krivak III class frigates from Russia.
▪ The Kamorta-class corvettes or Project 28 are a class of anti-submarine warfare
corvettes currently in service with the Indian Navy
▪ They are the first anti-submarine warfare stealth corvettes to be built in India.
▪ Kamorta or Kalatapu is a village on the Kamorta Island, in the Nicobar district of
Andaman and Nicobar Islands.

Have a querry?
Drop us a mail at – hello@civilstap.com or call us at 8146207241
113
Revise your past 1.5 years of current affairs with our most comprehensive Test Series on Current Affairs

{QUESTION NUMBER} 93
{TAGS} Indices
The Public Sector Undertakings (PSU) index is:
{OPTION 1} listed on BSE and consists of all PSU stocks in the BSE 500 index
{OPTION 2} listed on NSE and consists of all PSU stocks in the NIFTY 50 index
{OPTION 3} listed on BSE and consists of all PSU stocks in the NIFTY 50 index
{OPTION 4} listed on BSE and consists of all PSU stocks in the BSE 30 index
{RIGHT ANSWER} 1
{EXPLANATION}
Context: BSE PSU index is likely to slip into the red under Narendra Modi govt first
since its inception in 1999.
▪ BSE Ltd. launched "BSE PSU Index" on 4 June 2001

▪ The S&P BSE SENSEX (S&P Bombay Stock Exchange Sensitive Index), also called
the BSE 30 or simply the SENSEX, is a stock market index of 30 well-established and
financially sound companies listed on Bombay Stock Exchange.
▪ The NIFTY 50 index represents the weighted average of 50 Indian company stocks in
12 sectors and is one of the two main stock indices used in India, the other being
the BSE sensex.

{QUESTION NUMBER} 94
{TAGS} Awards
United Nations Environment Programme (UNEP) has awarded Asia Environment
Enforcement Award 2018 to which of the following?
{OPTION 1} World Wildlife Fund for conservation of endangered species
{OPTION 2} Wildlife Crime Control Bureau for work done in combating
transboundary environmental crime
{OPTION 3} Food and Agriculture Organization for its efforts in defeating hunger
{OPTION 4} Both a and b
{RIGHT ANSWER} 2
{EXPLANATION}
▪ Context: UNEP has awarded Wildlife Crime Control Bureau (WCCB), MoEFCC with
Asia Environment Enforcement Awards, 2018 for excellent work done by the Bureau
in combating transboundary environmental crime.

Have a querry?
Drop us a mail at – hello@civilstap.com or call us at 8146207241
114
Revise your past 1.5 years of current affairs with our most comprehensive Test Series on Current Affairs

▪ These awards publicly recognize and celebrate excellence in enforcement by


government officials and institutions/teams combating transboundary
environmental crime in Asia.
▪ The awards are given to outstanding individuals and/or government
organizations/teams that demonstrate excellence and leadership in enforcement of
national laws to combat transboundary environmental crime in one of the following
eligibility criteria areas: collaboration; impact; innovation; integrity and gender
leadership.
▪ WCCB has been conferred this award in Innovation category.
▪ This is also the second time in a row the awards are being given by UNEP to India.

{QUESTION NUMBER} 95
{TAGS} Awards
Consider the following statements regarding Model Code of Conduct:
1) It has been given a statutory status.
2) It can come into action before polls are announced.
Which of the above statements is/are correct?
{OPTION 1} 1 only
{OPTION 2} 2 only
{OPTION 3} None is correct
{OPTION 4} Both are correct
{RIGHT ANSWER} 2
{EXPLANATION} Recent Context: Post dissolution of Jammu Kashmir Assembly
by the Governor, implementation of MCC by EC is under discussion.
▪ These are the guidelines issued by the Election Commission of India for conduct of
political parties and candidates during elections mainly with respect to speeches,
polling day, polling booths, election manifestos, processions and general conduct.
MCC does not have any statutory status and is only a moral code.
▪ The Model Code of Conduct comes into force immediately on announcement of the
election schedule by the commission. The Code remains in force till the end of the
electoral process.
▪ Its aim is to ensure free and fair elections.
▪ The poll panel had recently decided that in states where assemblies are dissolved
prematurely, the model code will come into force immediately. The poll code, which
bars the caretaker government and the party in power from taking policy decisions,
otherwise comes into force the day polls are announced by the EC.
▪ Telangana was the first state where the model code of conduct had come into effect
before the assembly polls were announced

Have a querry?
Drop us a mail at – hello@civilstap.com or call us at 8146207241
115
Revise your past 1.5 years of current affairs with our most comprehensive Test Series on Current Affairs

{QUESTION NUMBER} 96
{TAGS} International news
The Regional Integrated Multi Hazard Early Warning System (RIMES) was in news
recently. Consider the following statements regarding RIMES:
1) It is an inter governmental body registered under the United Nations.
2) It seeks to establish an early warning system for cyclones and earthquakes only.
Which of the above statement/s is/are correct?
{OPTION 1} 1 only
{OPTION 2} 2 only
{OPTION 3} Both 1 and 2
{OPTION 4} Neither 1 nor 2
{RIGHT ANSWER} 1
{EXPLANATION}
• It is an international and intergovernmental institution, owned and managed by
its Member States, for the generation and application of early warning
information.
• It evolved from the efforts of countries in Africa and Asia, in the aftermath of the
2004 Indian Ocean tsunami, to establish a regional early warning system within a
multi-hazard framework.
• It was established in 2009, and was registered with the United Nations later in
2009. It operates from its regional early warning center located in Thailand.
• Services - Weather, Climate and Hydrological Research and Development,
Earthquake & Tsunami Watch Provision, Capacity Building in End-to-End Early
Warning.

{QUESTION NUMBER} 97
{TAGS} Terms in news
Consider the following statements related to Particularly Vulnerable Tribal Groups
(PVTGs):
1) These are the tribes within the ST category which are more vulnerable among
the tribal groups.
2) These were earlier known as Primitive Tribal Groups
3) These are identified and approved by the Central Ministry of Tribal Affairs
Which of the above statement/s is/are correct?
{OPTION 1} 1 only
{OPTION 2} 1 and 2 only
{OPTION 3} 1 and 3 only
{OPTION 4} 2 only
{RIGHT ANSWER} 2
{EXPLANATION}
Explanation:

Have a querry?
Drop us a mail at – hello@civilstap.com or call us at 8146207241
116
Revise your past 1.5 years of current affairs with our most comprehensive Test Series on Current Affairs

▪ Context: Particularly vulnerable tribal group (earlier known as Primitive tribal group)
is a government of India classification created with the purpose of enabling
improvement in the conditions of certain communities with particularly low
development indices
▪ The Dhebar Commission (1960-1961) stated that within Scheduled Tribes there
existed an inequality in the rate of development.
▪ During the fourth 5 Year Plan a sub-category was created within Scheduled Tribes to
identify groups that considered to be at a lower level of development.
▪ 75 tribal groups have been categorized by Ministry of Home Affairs as Particularly
Vulnerable Tribal Groups (PVTG)s. The Ministry of Tribal Affairs implements the
Scheme of “Development of Particularly Vulnerable Tribal Groups (PVTGs)”
exclusively for them.

{QUESTION NUMBER} 98
{TAGS} Terms in news
Consider the following statements regarding “Constitution Day”:
1) It is celebrated in order to mark the coming into effect of the Constitution of
India.
2) It is celebrated on 26th Nov every year.
Which of the above statement/s is/are correct?
{OPTION 1} 1 only
{OPTION 2} 2 only
{OPTION 3} Both 1 and 2
{OPTION 4} Neither 1 nor 2
{RIGHT ANSWER} 2
{EXPLANATION}
▪ Constitution Day (National Law Day), also known as Samvidhan Divas, is celebrated
in India on 26 November every year to commemorate the adoption of Constitution
of India.
▪ The Constituent Assembly of India adopted the Constitution of India on 26 November
1949 and it came into effect on 26 January 1950.
▪ Previously this day was celebrated as Law Day.

{QUESTION NUMBER} 99
{TAGS} Important Bills, Acts and Amendments
Consider the following statements:
1) The 73rd Constitutional Amendment Act, 1992 extends to the whole of India.
2) Election Commission of India has the power to oversee the establishment of
electoral rolls as well as organize elections for legislative assembly and council
in Jammu and Kashmir.
Which of the above statement/s is/are incorrect?
{OPTION 1} 1 only

Have a querry?
Drop us a mail at – hello@civilstap.com or call us at 8146207241
117
Revise your past 1.5 years of current affairs with our most comprehensive Test Series on Current Affairs

{OPTION 2} 2 only
{OPTION 3} Both 1 and 2
{OPTION 4} Neither 1 nor 2
{RIGHT ANSWER} 1
{EXPLANATION}
▪ The legal authority to conduct elections in Jammu and Kashmir is drawn from
provisions in both the Constitution of India and the Constitution of Jammu and
Kashmir.
▪ Both constitutions give power to the ECI to oversee the establishment of the
electoral rolls as well as organize the elections to both the Legislative Assembly and
Legislative Council in J&K.
▪ Panchayat elections in J&K are held according to the provisions in the Jammu and
Kashmir Panchayati Raj Act 1989
▪ The 73rd Amendment 1992 added a new Part IX to the constitution titled “The
Panchayats” covering provisions from Article 243 to 243(O); and a new Eleventh
Schedule covering 29 subjects.

{QUESTION NUMBER} 100


{TAGS} Science and Tech
RFID tags are being increasingly used these days for identifying and tracking objects.
What is the full form of RFID?
{OPTION 1} Radio Frequency Identification Device
{OPTION 2} Real Feedback Identification Device
{OPTION 3} Radio Feedback Identification Device
{OPTION 4} None of the above
{RIGHT ANSWER} 1

Have a querry?
Drop us a mail at – hello@civilstap.com or call us at 8146207241
118

Potrebbero piacerti anche